Sales Compiled Digests

September 16, 2017 | Author: racerchique | Category: Assignment (Law), Taxes, Deed, Mineral Rights, Lawsuit
Share Embed Donate


Short Description

Download Sales Compiled Digests...

Description

INTRODUCTION Buenaventura vs CA (2003) FACTS:  Defendant spouses Leonardo Joaquin and Feliciana Landrito are parents of co-defendants Fidel, Tomas, Artemio, Clarita, Felicitas, Fe, and Gavino.  They are also the parents of plaintiffs Consolacion, Nora, Emma, and Natividad.  A deed of sale was executed by the defendant spouses in favor of their co-defendant children.  However, such deed of sale is sought to be declared null and void by the plaintiffs.  Plaintiffs argue that: 1. There was no actual consideration 2. Even assuming there was consideration, the properties are more than 3-fold times more valuable than the measly sums appearing therein. 3. the sale was the result of a deliberate conspiracy to unjustly deprive the rest of the compulsory heirs of their legitime.  RTC: ruled in favor of the defendants and dismissed the complaint. On the grounds that: 1. plaintiffs do not have a valid cause of action against defendants since there can be no legitime to speak of prior to the death of their parents. 2. legitime is computed as of the time of the death of the decedent.  CA: affirmed the decision of the RTC ISSUE: I. W/N the Deeds of Sale are void for lack of consideration HELD: I. DEED OF SALE VALID. 1. A contract of sale is not a real contract, but a consensual contract. 2. As a consensual contract, a contract of sale becomes a binding and valid contract upon the meeting of the minds as to price. 3. If there is a meeting of the minds of the parties as to the price, the contract of sale is valid, despite the manner of payment, or even the breach of that manner of payment. 4. It is not the act of payment of price that determines the validity of a contract of sale. 5. Payment of the price has nothing to do with the perfection of the contract. 6. Failure to pay the consideration is different from lack of consideration. 7. Petitioners do not have any legal interest over the properties. Their rights over the properties are merely inchoate and vests only upon their parents death. Gaite vs Fonacier (1961) Facts:







Fonacier owned 11 iron lode mineral claims, known as the Dawahan Group, located in Camarines Norte. He appointed Gaite as his attorney-in-fact to enter into contracts with individual or juridical persons for the exploration and development of the mining claims. Gaite in turn executed a general assignment conveying the development of the mining claims into the Larap Iron Mines, a single proprietorship owned by him. Then he started the development of those mining claims. Fonacier decided to revoke the authority granted by him to Gaite, and Gaite assented, subject to certain conditions. They entered into a contract, where Gaite transferred to Fonacier, for P20k, all his rights and interests on the roads and facilities in the claims, plus the right to use the business name “Larap Iron Mines.” Gaite also transferred to Fonacier, for P75k, all this rights and interests over the tons of iron ore that he already extracted from the mineral claims. P10k of this was paid upon signing and the contract stated that “the balance of P65k will be paid from and out of the first letter of credit covering the first shipment of iron ores and of the first account derived from the local sale of iron ore made by Larap.” To seucre the payment, Fonacier delivered to Gaite a surety bond. Gaite wanted another bond, so Fonacier executed a second one, but it provided that the liability of the surety company would attach only when there had been an actual sale of iron ore for an amount of no less than P65k, and that the liability of said surety company would automatically expire on Dec 1955. Up to Dec 1955, when the bond expired with respect to the surety company, no sale of the tons of iron ore had been made by Larap, nor had the P65k balance of the price of said ore

been paid to Gaite by Fonacier and his sureties, the second bond automatically expired. And when Fonacier and his sureties failed to pay, Gaite filed the present complaint against them for the payment of the P65k balance. ○ The defendants set up the defense that the obligation sued upon by Gaite was subject to a condition that the amount of P65k would be payable out of the first letter of credit covering the first shipment of iron ore and/or the first amount derived from the local sale of the iron ore by Larap, and that up to the time of the filing of the complaint no sale of the iron ore had been made. Therefore, the obligation was not due and demandable yet. ○ The lower court held in favor of Gaite, and the defendants were ordered to pay the P65k. The lower court held that the oblig of the defendants to pay Gaite was one with a term: that it would be paid upon the sale of sufficient iron ore, such sale to be effected on or before Dec 1955, and that as the latter failed to put up a good and sufficient security after the bond expired, the oblig became due and demandable. Issue: Is the obligation of Fonacier to pay Gaite the P65k an obligation with a period or term and not one with a suspensive condition? Held: Obligation was subject to a suspensive period or term. Lower court decision affirmed. – A contract of sale is normally commutative and onerous. Not only does each of the parties assume a correlative obligation (the seller to deliver and transfer ownership of the thing sold and the buyer to pay the price), but each party anticipates performance by the other from the very start. While in a sale the obligation of one party can be lawfully subordinated to an uncertain event, so that the other understands that he assumes the risk of receiving nothing for what he gives, it isn’t in the usual course of business to do so. Hence, the contingent character of the obligation must clearly appear. Nothing is found in the record to evidence that Gaite desired or assumed to run the risk of losing his right over the ore without getting paid for it, or that Fonacier understood that Gaite assumed any such risk. This is proven by the fact that Gaite insisted on a bond to guarantee payment of the P65k. – Plus, to subordinate the oblig to pay the remaining P65k as condition precedent would be tantamount to leaving the payment at the discretion of the debtor, for the sale or shipment could not be made unless steps were taken to sell the ore. – Fonacier has forefeited the right to compel Gaite to wait for the sale of the ore before receiving payment of the balance of P65k, because of their failure to renew the bond or else replace it with an equivalent guarantee. CELESTINO CO & CO. VS. COLLECTOR OF INTERNAL REVENUE Contract of sale vs. Contract for a piece of work Facts: – Celestino Co & Co. is a registered copartnership doing business under the trade name of Oriental Sash Factory, which makes sash, windows and doors – For years, they paid percentage taxes of 7% on gross receipts (for manufacturers), but in 1952 they began to claim that they should be paying only 3% under sec 191 of the National Revenue Code (for contractors) – They contend that Oriental Sash Factory does not manufacture ready-made doors, sash and windows for the public but only upon special order of its select customers and That the things, had they not been ordered, would not have existed. – They filed a petition w/ the Bureau of Internal Revenue and appealed to the CTA, which both held that they did not come under the purview of Sec. 191. Issue: whether Celestino Co & Co. is a manufacturer or a contractor? Held: manufacturer, in the business of selling goods, not services – Celestino Co & Co. habitually makes sash, windows and doors, as it has represented in its stationery and advertisements to the public. – The fact that windows and doors are made by it only when customers place their orders, does not alter the nature of the etablishment, for it only accepted such orders as called for the employment of such materials as it ordinarily manufactured or was in a position habitually to manufacture. – They do not serve special customers only or confine its services to them alone. Anyone who has the ability to pay may have such things manufactured by them.

– –

That the doors and windows must meet desired specifications is neither here nor there. If the specifications do not happen to be of the kind they habitually manufacture, they would not accept the order and no sale is made. It is only when this factory accepts a job that requires the use of extraordinary or additional equipment, or involves services not generally performed by it---ir thereby contracts for a piece-of-work---filling special orders within the meaning of Art. 1467. The orders, however, were not shown to be special.

Commissioner of Internal Revenue vs. Engineering Equipment and Supply Company 1. 2. 3.

4.

EESC is an engineering and machinery firm and is engaged, among others, in the design and installation of central type air conditioning system, pumping plants and steel fabrications. One Juan dela Cruz wrote the Collector, now Commissioner of CIR, denouncing EESC for tax evasion and failing to pay the correct percentage taxes due in connivance with foreign suppliers. EESC was likewise denounced to the Central Bank for alleged fraud in obtaining its dollar allocations. Revenue Examiners reported and recommended to the Collector that Engineering be assessed for P480K+ for not declaring its importations which are subject of tax under Section 185 of the Tax Code (30% of the gross selling price). Assessment was revised and raised to P916K+ inclusive of 25% and 50% surcharges. EESC appealed with the Court of Tax Appeals (CTA) and reduced liabilities to P740K+. Commissioner not satisfied filed appeal to the Court; EESC filed with CTA a motion for recon.

Issue: whether or not EESC is a manufacturer of air conditioning units under Sec. 185, or a contractor under Sec. 191 of the Tax Code Held: Contractor. 1. Contract of sale vs. contract of furnishing services, labor, and materials

a.

2.

3. 4.

Test: whether the thing transferred is one not in existence and which never would have existed but for the order of the party desiring to acquire it, or a thing which would have existed and has been the subject of sale to some other persons even if the order had not been given. If the article ordered by the purchaser is exactly as such as the plaintiff makes and keeps on hand for sale to anyone, and no change or modification of it is made at defendant’s request, it is a contract of sale, even though it may be entirely made after, and in consequence of, the defendants order for it. We find that EESC did not manufacture air conditioning units for sale to the general public, but imported some items which were used in executing contracts entered into by it. EESC undertook negotiations and execution of individual contracts for the design, supply, and installation of air conditioning units of the central type. EESC definitely did not and was engaged in the manufacture of air conditioning units but had its services contracted for the installation of a central system. EESC should be held liable to pay taxes prescribed in Section 190. This compensating tax is not a tax on the importation of the goods but tax on the use of imported goods not subject to sales tax. (30% compensating tax but without the 50% markup)

Issue: fraud Held: 1. The Commissioner charged Engineering with misdeclaration of the imported air conditioning units and parts or accessories thereof so as to make them subject to a lower rate of percentage tax (7%) under Section 186 of the Tax Code, when they are allegedly subject to a higher rate of tax (30%) under its Section 185(m). This charge of fraud was denied by Engineering but the Court of Tax Appeals in its decision found adversely and said" ... We are amply convinced from the evidence presented by respondent that petitioner deliberately and purposely misdeclared its importations. This evidence consists of letters written by petitioner to its foreign suppliers, instructing them on how to invoice and describe the air conditioning units ordered by petitioner. ... 2. Despite the above findings, however, the Court of Tax Appeals absolved Engineering from paying the 50% surcharge prescribe by Section 183(a) of the Tax Code by reasoning out as follows: The imposition of the 50% surcharge prescribed by Section 183(a) of the Tax Code is based on willful neglect to file the monthly return within 20 days after the end of each month or in case a false or fraudulent return is willfully made, it can readily be seen, that petitioner cannot legally be held subject to the 50% surcharge imposed by Section 183(a) of the Tax Code. Neither can petitioner be held subject to the 50% surcharge under Section 190 of the Tax Code dealing on compensating tax because the provisions thereof do not include the 50% surcharge. Where a particular provision of the Tax Code does not impose the 50% surcharge as fraud penalty we cannot enforce a non-existing provision of law notwithstanding the assessment of respondent to the contrary. Instances of the exclusion in the Tax Code of the 50% surcharge are those dealing on tax on banks, taxes on receipts of insurance companies, and franchise tax. However, if the Tax Code imposes the 50% surcharge as fraud penalty, it expressly so provides as in the cases of income tax, estate and inheritance taxes, gift taxes, mining tax, amusement tax and the monthly percentage taxes. Accordingly, we hold that

petitioner is not subject to the 50% surcharge despite the existence of fraud in the absence of legal basis to support the importation thereof. 3. The communications of EESC to its foreign importers are strongly indicative of the fraudulent intent of Engineering to misdeclare its importation of air conditioning units and spare parts or accessories thereof to evade payment of the 30% tax. And since the commission of fraud is altogether too glaring, We cannot agree with the Court of Tax Appeals in absolving Engineering from the 50% fraud surcharge, otherwise We will be giving premium to a plainly intolerable act of tax evasion. 4. Since the imported air conditioning units-and spare parts or accessories thereof are subject to the compensating tax of 30% as the same were used in the construction business of Engineering, it is incumbent upon the latter to comply with the aforequoted requirement of Section 190 of the Code, by posting in its books of accounts or notifying the Collector of Internal Revenue that the imported articles were used for other purposes within 30 days. ... Consequently; as the 30% compensating tax was not paid by petitioner within the time prescribed by Section 190 of the Tax Code as amended, it is therefore subject to the 25% surcharge for delinquency in the payment of the said tax. WHEREFORE, the decision appealed from is affirmed with the modification that Engineering is hereby also made liable to pay the 50% fraud surcharge.

Quiroga v Parsons Hardware Co. Quiroga and Parsons, both merchants in Manila, executed a contract between themselves for the “exclusive sale of Quiroga beds in the Visayan Islands.” The provisions of the contract essentially stipulated that Parsons would order the beds by the dozen, and would make an allowance of 25% of the invoiced prices. Parsons was also bound to pay for the received beds no later than 60 days after shipment. The matters of dispute center on two issues: 1) Parsons’ obligation to charge customers no higher than the invoiced prices, and 2) order the beds strictly by the dozen and in no other manner. Was this contract one of agency, or one of purchase and sale? The contract is one of purchase and sale. In order to classify a contract, due regard must be given to its essential clauses. In the contract in question, what was essential is that Quiroga was to furnish Parsons with the beds which the latter might order, at the price stipulated, and that the defendant was to pay the price in the manner stipulated. The contract exhibited the features of a contract of purchase and sale. The rule to remember is that when the supposed “agent” has to pay the “principal” the price of the goods, the contract ceases to be one of agency. In agency, the agent surrenders the amount received from the sale of goods, as well as the unsold goods themselves. In this case, it did not matter how many were sold; payment had to be made for delivered beds 60 days upon delivery. Thus, Parsons is not subject to the contract terms in question since it, as purchaser, acquires ownership of delivered goods. SONNY LO vs KJS ECO-FORMWORK SYSTEM PHIL, INC.

• •







Lo, doing business under the name San’s Enterprises, ordered scaffolding equipments from KJS worth P540,425.80. Lo paid a downpayment of P150,000 and the balance was to be paid in 10 monthly installments. KJS delivered the scaffoldings to Lo, who paid the first two installments. However, his business encountered financial difficulties and he was unable to settle his obligation despite oral and written demands. Lo and KJS executed a Deed of Assignment, whereby Lo assigned to KJS his receivables in the amount of P335,462.14 from Jomero Realty Corporation. The agreement also stipulated: “The ASSIGNOR further agrees and stipulates as aforesaid that the said ASSIGNOR, his heirs, executors, administrators, or assigns, shall and will at times hereafter, at the request of said ASSIGNEE, its successors or assigns, at his cost and expense, execute and do all such further acts and deeds as shall be reasonably necessary to effectually enable said ASSIGNEE to recover whatever collectibles said ASSIGNOR has in accordance with the true intent and meaning of these presents.” When KJS tried to collect the said credit from Jomero, it refused to honor the Deed of Assignment because it claimed that Lo was also indebted to it. KJS sent a letter to Lo demanding payment but he refused claiming that his obligation had been extinguished when they executed the Deed of Assignment. KJS filed an action for recovery of a sum of money against Lo with the RTC, which dismissed the complaint on the ground that the assignment of credit extinguished the obligation. However, the CA held that the Deed of Assignment did not extinguish the

obligation of Lo. Issue: W/N the Deed of Assignment extinguished Lo’s obligation. NO, he failed to comply with his warranty.



In dacion en pago1, as a special mode of payment, the debtor offers another thing to the creditor who accepts it as equivalent of payment of an outstanding debt. The undertaking really partakes in one sense of the nature of sale – the creditor is really buying the thing or property of the debtor, payment for which is to be charged against the debtor’s debt. • The assignment of credit, which is in the nature of a sale of personal property, produced the effects of a dation in payment, which may extinguish the obligation. However, as in any other contract of sale, the vendor or assignor is bound by certain warranties. Paragraph 1 of Article 1628 of the Civil Code provides: The vendor in good faith shall be responsible for the existence and legality of the credit at the time of the sale, unless it should have been sold as doubtful; but not for the solvency of the debtor, unless it has been so expressly stipulated or unless the insolvency was prior to the sale and of common knowledge. • Lo, as assignor, is bound to warrant the existence and legality of the credit at the time of the sale or assignment. When Jomero claimed that it was no longer indebted to Lo since the latter also had an unpaid obligation to it, it essentially meant that its obligation to Lo has been extinguished by compensation. As a result, KJS alleged the non-existence of the credit and asserted its claim to Lo’s warranty under the assignment. Lo was therefore required to make good its warranty and pay the obligation. • Furthermore, Lo breached his obligation under the Deed of Assignment as he did not “execute and do all such further acts and deeds as shall be reasonably necessary to effectually enable said ASSIGNEE to recover whatever collectibles said ASSIGNOR has in accordance with the true intent and meaning of these presents.” By warranting the existence of the credit, Lo should have ensured its performance in case it is found to be inexistent. He should be held liable to pay to KJS the amount of his indebtedness Judgment Affirmed.

II - PARTIES TO A SALE Paragas vs. Heirs of Dominador Balacano (2005) FACTS:  Gregorio and Lorenza Balacano owned Lots 1175-E and 117-F.  Spouses Balacano had 3 children, namely Domingo, Catalino, and Alfredo.  Lorenza died during Dec. 11, 1991, while Gregorio died on July 28, 1996.  Prior to Gregorio’s death, he was admitted in Veteran’s General Hospital in Nueva Vizcaya, and later transferred to Veteran’s Memorial Hospital in QC, until he died.  It was alleged that Gregorio, barely a week prior to his death, sold the 2 lots to spouses Rudy and Corazon Paragas.  The said sale appeared in a deed of absolute sale notarized by Atty. De Guzman.  The spouses Paragas then sold a portion of one of the lots to Catalino.  Domingo’s children filed a complaint for the annulment of the sale against Catalino and the spouses Paragas.  Domingo’s children argue that: 1. Grandfather Gregorio was seriously ill at the time of the execution of the deed of sale. 2. Gregorio’s consent was vitiated. 3. That the lots form part of the conjugal partnership properties of Gregorio and Lorenza.  Spouses Paragas moved to dismiss the complaint, arguing that 1. Plaintiffs do not have a cause of action and have no legal ground for the annulment of the deed of sale.

1 Requisites: (1) There must be the performance of the prestation in lieu of payment (animo solvendi) which may consist in the delivery of a corporeal thing or a real right or a credit against the third person; (2) There must be some difference between the prestation due and that which is given in substitution (aliud pro alio); (3) There must be an agreement between the creditor and debtor that the obligation is immediately extinguished by reason of the performance of a prestation different from that due.

2.





Gregorio and Paragas already agreed on the sale of the Lots, that the execution of the Deed of Sale was merely a confirmation of the said agreement. RTC: declared the deed of sale null and void, and the lots were CP properties. Grounds: 1. Gregorio was ill 2. Deed of sale was improperly notarized. 3. Atty. De Guzman explanations regarding the erroneous entries on the actual place and date of execution of the deed of sale were justifications for a lie. (He testified that the deed was only a “confirmation” of a previous agreement bet. Gregorio and Paragas). 4. Rudy Paragas refused or failed to testify about the signing of the deed of sale. CA: affirmed, with modifications: lots were estate of Gregorio

ISSUE: I. W/N the Deed of Sale is null and void. HELD: I. DEED OF SALE NULL AND VOID 1. Gregorio, died due to complications caused by cirrhosis of the liver, had been fighting the said disease for a month. 2. Due to his condition, there are serious doubts at to whether he could read, or fully understood the contents of the deed of sale. 3. There are no conclusive evidence that show that the evidence of the deed were sufficiently explained to Gregorio before he affixed his signature. 4. Art. 24 of the NCC provides that in all contractual, property or other relations, when one of the parties is at a disadvantage on account of his moral dependence, ignorance, indigence, mental weakness, tender age or other handicap, the courts must be vigilant for his protection.

calimlim-Canullas vs. Fortun (1984) FACTS:  Petitioner Mercedes Calimlim-Canullas and Fernando Canullas were married in 1962 and had 5 children.  They lived in a house on the residential lot in question.  After Fernando’s father died in 1965, Fernando inherited the said land.  In 1978, Fernando abandoned his family and lived with private respondent Corazon Daguines.  Both were convicted of concubinage by the CFI.  In 1980, Fernando sold the said land to Daguines.  In the document of sale, Fernando described the house as “also inherited by me from my deceased parents.”  Unable to take possession of the lot and house, Daguines filed a complaint against Mercedes.  Mercedes however, claimed that the sale is null and void for reason that the said house and land were conjugal properties and she had not given her consent to the sale.  CFI: declared Daguines as the lawful owner of the house and lot. ISSUE: I. W/N the construction of a conjugal house on the exclusive property of the husband gave the land the character of conjugal property. II. W/N the sale of the lot together with the house and improvements was valid HELD: I. LAND CONJUGAL PROPERTY 1. NCC provides that “buildings constructed at the expense of the partnership during marriage on land belonging to one of the spouses also pertain to the partnership, but the value of the land shall be reimbursed to the spouse who owns the same. 2. Therefore, both the land and the house belonged to the conjugal partnership, but the cp is indebted to the husband for the value of the land. 3. Husband’s power to alienate conjugal property must be with wife’s consent. 4. Mercedes did not consent to the sale. II. SALE NULL AND VOID. 1. Sale was contrary morals and public policy.

2. 3.

Sale was made by a husband in favor of a concubine after he had abandoned his family and left the conjugal home. NCC provides that contracts whose cause, object, or purpose is contrary to law, morals, good customs, public order, or public policy are void and inexistent from the very beginning.

Rubias vs Batiller (1973) Facts:



– – – –

Francisco Militante claimed that he owned a parcel of land located in Iloilo. He filed with the CFI of Iloilo an application for the registration of title of the land. This was opposed by the Director of Lands, the Director of Forestry, and other oppositors. The case was docked as a land case, and after trial the court dismissed the application for registration. Militante appealed to the Court of Appeals. Pending that appeal, he sold to Rubias (his son-in-law and a lawyer) the land. The CA rendered a decision, dismissing the application for registration. Rubias filed a Forcible Entry and Detainer case against Batiller. In that case, the court held that Rubias has no cause of action because the property in dispute which Rubias allegedly bought from Militante was the subject matter of a land case, in which case Rubias was the counsel on record of Militante himself. It thus falls under Article 1491 of the Civil Code. (Hence, this appeal.)

Issue: Whether the sale of the land is prohibited under Article 1491. Held: YES. Article 1491 says that “The following persons cannot acquire any purchase, even at a public or judicial auction, either in person or through the mediation of another…. (5) Justices, judges, prosecuting attorneys, clerks of superior and inferior courts, and other officers and employees connected with the administration of justice, the property and rights in litigation or levied upon an execution before the court within whose jurisdiction or territory they exercise their respective functions; this prohibition includes the act of acquiring by assignment and shall apply to lawyesr, with respect to the property and rights which may be the object of any litigation in which they may take part by virtue of their profession.” The present case clearly falls under this, especially since the case was still pending appeal when the sale was made. Issue: Legal effect of a sale falling under Article 1491? Held: NULL AND VOID. CANNOT BE RATIFIED. Manresa considered such prohibited acquisitions (which fell under the Spanish Civil Code) as merely voidable because the Spanish Code did not recognize nullity. But our Civil Code does recognize the absolute nullity of contracts “whose cause, object or purpose is contract to law, morals, good customs, public order or public policy” or which are “expressly prohibited or declared void by law” and declares such contracts “inexistent and void from the beginning.” The nullity of such prohibited contracts is definite and permanent, and cannot be cured by ratification. The public interest and public policy remain paramount and do not permit of compromise or ratification. In this aspect, the permanent disqualification of public and judicial officers and lawyers grounded on public policy differs from the first three cases of guardians agents and administrators (under Art 1491). As to their transactions, it has been opined that they may be “ratified” by means of and in “the form of a new contract, in which case its validity shall be determined only by the circumstances at the time of execution of such new contract.” In those cases, the object which was illegal at the time of the first contract may have already become lawful at the time of the ratification or second contract, or the intent, or the service which was impossible. The ratification or second contract would then be valid from its execution; however, it does not retroact to the date of the first contract. Decision affirmed.

Phil Trust Co. vs Roldan (1956) Facts:







Mariano Bernardo, a minor, inherited properties from his father, deceased. Included in these were 17 parcels of land located in Bulacan. Socorro Roldan was appointed his guardian. (Socorro was the stepmother of Mariano.) In the guardianship proceedings, Socorro filed a motion asking for authority to sell as guardian the 17 parcels of land for P14.7k to Dr. Fidel C. Ramos, her brother-in-law, the purpose of the sale being allegedly to invest the money in a residential house which the minor wanted to have. The motion was granted. Socorro then sold the parcels of land to Ramos for P14.7k. But the very next day, Ramos sold the land to Socorro for P15k. And eventually, Socorro sold 4 out of the 17 parcels to another party, reserving to herself the right to repuchase. The Phil Trust Company replaced Socorro Roldan as guardian. And this present case seeks to undo what she did, claiming that it was void for falling under Art 1459 which prohibits the guardian from purchasing “either in person or through the mediation of another” the property of her ward. The CFI held that Art 1459 was not controlling because there was no proof that Ramos had previously agreed to buy the parcels from Socorro for her benefit. The CFI upheld the contracts but allowed the minor to repurchase all the parcels by paying P15k. The CA affirmed the judgment, adding that the minor knew the particulars thereof and approved the transaction.

Issue: Valid sale? Held: NO. NULL AND VOID. As guardianship is a trust of the highest order, the trustee cannot be allowed to have any inducement to neglect his ward’s interest. Whenever the guardian acquires the ward’s property through an intermediary, he violates the provisions of Art 1459 and such transaction and subsequent ones emanating therefrom shall be annulled. The transactions show that Socorro bought the 17 parcels on the day following the sale to Ramos. There may not have been a previous agreement between her and Ramos to the effect that the latter would buy the lands for her, but the stubborn fact remains that she acquired her protégé’s properties, through her brother-in-law. That she planned to get them for herself at the time of selling them to Ramos may be deduced from the very short time between the two sales. Also, the third sale is void because Socorro could pass no title to the third party. The annulment carries with it the obligation of Socorro to return the 17 parcels of land together with the fruits and the duty of the minor, through his guardian to repay P14.7k with legal interest. Fabillo vs IAC (1991) Facts:







Justina Fabillo bequeathed to her brother Florencio a house and lot in San Salvador, Palo, Leyte, and to her husband a piece of land in Pugahanay, Palo, Leyte. After Justina’s death, Florencio filed a petition for the probate of said will. The court approved the project of partition but said that the ownership of the land of Florencio be litigated and determined in separate proceedings. So two years later, Florencio asked Atty. Murillo to assist him in recovering the San Salvador property. Murillo asked him for 40% of the money value of the house and lot as a contingent fee in case of success. Murillo and Florencio then entered into a contract: Florencio agreed to pay Murillo, in case of success, the sum equivalent to 40% of whatever benefit Florencio may derive from such cases. Also, if the house and lot or a portion thereof is just occupied by Florencio or his heirs, Murillo shall have the option of either occupying or leasing to any interested party 40% of the house and lot. Murillo, pursuant to the contract, filed a case against Justina’s husband to recover the San Salvador property. The case was terminated when the court, upon the parties’ compromise agreement, declared Florencio the lawful owner of the San Salvador and Pugahanay property. So Murillo then proceeded to exercise ownership over 40% of said properties and installed a tenant in the Pugahanay property. Eventually, Florencio claimed exclusive rights over the properties and refused to give Murillo the share of the produce of the properties. Murillo then filed a complaint, asking that he be declared owner of 40% of the two properties. Florencio asked that the contract be declared null and void, plus that they had vitiated consent.





The lower court: There was insufficient evidence to prove that the consent was vitiated. Ordered Florencio to pay 40% of the net produce of the property. Declared Murillo as owner of 40% of both the properties. IAC affirmed. Note: The case is being carried on by Florencio and Murillo’s heirs.

Issue: Stipulation valid? Held: YES. A contingent fee does not fall under prohibition in Art 1491 par 5. While Art 1491 par 5 prohibits lawyers from acquiring by purchase the properties and rights which are the objects of litigation in which they may take part by virtue of their profession, this prohibition applies only if the sale of the assignment of the property takes place during the pendency of the litigation involving the client’s property. A contract between a lawyer and his client stipulating a contingent fee is not covered by said prohibition. The payment of such fee is not made during the pendency of the litigation but only after judgment has been rendered in the case handled by the lawyer. As long as the lawyer does not exert undue influence and no fraud is committed, a contract for contingent fee is valid and enforceable. However, a careful look at the contract shows that the parties intended 40% of the value of the properties as Murillo’s contingent fee. Plus, the stipulation on Murillo having the option to occupy or lease to any interested party 40% of the house and lot is declared vague. The ambiguity should be resolved against Murillo because it was he who drafted the contract. IAC decision reversed. Florencio’s heirs ordered to pay Murillo’s heirs the amount of P3k as his contingent fee.

III – SUBJECT MATTER PUP V. CA –

Firestone Ceramics (Firestone) entered into a lease contract w/ the National Development Corporation (NDC) for a portion of its property in Sta. Mesa. 2 more lease agreements were entered into for NDC’s 4-unit pre-fabricated reparation steel warehouse and 6-unit prefabricated reparation steel warehouse. Such agreements provided for extenion of the terms of the lease.



Firestone requested for an extension, which was granted w/ the condition that in the event NDC "with the approval of higher authorities, decide to dispose and sell these properties including the lot, priority should be given to the LESSEE”  first refusal



Firestone again requested for an extension, but the communications were unacknowledged. FIRESTONE's predicament worsened when rumors of NDC's supposed plans to dispose of the subject property in favor of petitioner Polytechnic University of the Philippines (PUP) came to its knowledge. Forthwith, FIRESTONE served notice on NDC conveying its desire to purchase the property in the exercise of its contractual right of first refusal. The PUP-NDC agreement revolved around Memorandum No. 24 issued by Pres. Aquino ordering the transfer of the property to PUP.





PUP and NDC claim that there was no sale between them: ○ The right of first refusal invoked was limited to the warehouse and not the lot ○ The courts supposedly created a contract to sell b/w the parties. It argued that the "court cannot substitute or decree its mind or consent for that of the parties in determining whether or not a contract (has been) perfected between PUP and NDC ○ NDC posits that the transaction did not amount to a sale considering that "ownership of the property remained with the government." Petitioner NDC introduced the novel proposition that if the parties involved are both government entities the transaction cannot be legally called a sale.

Issues: 1. w/n there was a contract of sale b/w PUP and NDC 2. w/n Firestone should be allowed to exercise its right of refusal Held: 1.



YES Aside from the fact that the intention of NDC and PUP to enter into a contract of sale was clearly expressed in the Memorandum Order No. 214, a close perusal of the circumstances

– –

of this case strengthens the theory that the conveyance of the property from NDC to PUP was one of absolute sale, for a valuable consideration, and not a mere paper transfer as argued by petitioners. there is not just one party involved in the questioned transaction. Petitioners NDC and PUP have their respective charters and therefore each possesses a separate and distinct individual personality All elements of asale were present: ○ Consent  Memorandum No. 214: “WHEREAS, PUP has expressed its willingness to acquire said NDC properties and NDC has expressed its willingness to sell the properties to PUP” ○ Consideration  the cancellation of NDC's liabilities in favor of the National Government in the amount of P57,193,201.64 constituted the "consideration" for the sale ○ PUP asserted its ownership over the property by posting notices within the compound advising residents and occupants to vacate the premises

1. YES -the right of first refusal is an integral and indivisible part of the contract of lease and is inseparable from the whole contract. , it is not correct for petitioners to insist that there was no consideration paid by FIRESTONE to entitle it to the exercise of the right, inasmuch as the stipulation is part and parcel of the contract of lease making the consideration for the lease the same as that for the option Atilano vs Atilano Facts:







Atilano I bought from Villanueva Lot 535 of the then municipality of Zamboanga cadastre. Atilano I had the lot subdivided into five parts (referred to as Lots A-E from hereon). He then sold Lot E in favor of his brother, Atilano II. Lots B-D were sold to other persons. Atilano I kept Lot A for himself. When he died, title of the lot passed to Ladislao Atilano. Atilano II and his children then obtained the transfer of certificate of title over Lot E in their names as co-owners. When they decided to have the land resurveyed, they discovered that they were actually occupying Lot A and not Lot E, as referred to in the deed, while the land which remained in the possession of the vendor (Atilano I) and which passed to his successor Ladislao Atilano was Lot E and not Lot A. [They basically switched around lots without their knowledge.] Atilano II passed away. His heirs filed the present action. They claimed that they had offered to surrender to Atilano I’s heirs (defendants) the possession of Lot A and demanded in return Lot E, but that the defendant’s had refused to accept the exchange. (That’s understandable since Lot E is much bigger than Lot A.) The defendants, on the other hand, claim that the reference to Lot E in their deed of sale was an involuntary error and that the intention of the parties was to convey the lot correctly identified as Lot A. ○ The trial court held in favor of the plaintiffs.

Issue: What is binding, the intent of the parties or the lots named in the deeds? Held: Intent. Plaintiffs ordered to execute a deed of conveyance of Lot E in favor of the defendants, and the latter are ordered to give Lot A to them. When one sells or buys real property, one sells or buys the property as he sees it, in its actual setting and by its physical metes and bounds, and not by the mere lot number assigned to it in the certificate of title. In this case, the portion correctly referred to as Lot A was already in the possession of the vendee, Atilano II, even before the sale in his favor. In like manner, Atinalo I had his house on Lot E. The two brothers continued in possession of the respective portions for the rest of their lives, obviously ignorant of the initial mistake. The real issue here is not adverse possession, but the real intention of the parties to that sale. From all the facts and circumstances, it shows that the object was that portion where the vendee was already residing, and where his heirs continued to reside thereafter; namely, Lot A, and that its designation as Lot E in the deed of sale was a simple mistake. The Civil Code provides a remember for such a situation by means of reformation of the instrument. In this case, the deed of sale executed need no longer be reformed. The parties have retained possession of their respective properties and all they should do is to execute mutual deeds of conveyance.

LONDRES V CA –







Filomena vidal (mother of the petitioners) sold 2 parcels of land (Lots 1320, 1333) to Consolacion and Julian Alovera Petitioners seek for the declaration of nullity of the contract, because the validity fo the Absolute Sale is in doubt due to alleged tampering. (+just compensation vs. DPWH) ○ The cadastral lot number of the second lot mentioned in the Absolute Sale was altered to read Lot 1333 when it was originally written as Lot 2034. Lot 2034 was also written in the copy of the Records Mgt. and Archives Ofc. The Aloveras explained that Julian was in good faith and that he was deaf and dumb, so he was placed at a disadvantageous position. When they discovered that the Absolute Sale indicated Lot 2034, they went back to Filomena, who made the correction. However, the copy of the notary remained unchanged. TC decided in favor of the Aloveras: ○ The description in the Absolute Sale corresponds to Lot 1333.

○ ○

The Absolute Sale states that the lot is in Brgy. Baybay, where Lot 1333 is situated. Lot 2034, on the other hand, can be found in Brgy. Culasi. Plus, there was no evidence that pet’s family owned Lot 2034 at any time.

Issue: w/n the Absolute Sale is valid Held: YES: the true object of the sale is Lot 1333 – The correction was made to reflect the true object of the sale, Lot 1333. – Petitioners rely on the technical descriptions of Lots 1320 and 1333 that were issued by the Bureau of Lands on November 8, 1988. When private respondents and Filomena executed the sale, they based the description of the two lots on the tax declarations of Filomena.. What really defines a piece of land is not the area mentioned in its description, but the boundaries therein laid down, as enclosing the land and indicating its limits. In this case, the boundaries of the two lots are sufficiently designated in the Absolute Sale, leaving no room to doubt the identity of the objects of the sale. – Lot 2034 does not fit the description of the 2nd parcel in the Absolute Sale. – when one sells or buys real property, one sells or buys the property as he sees it, in its actual setting and by its physical metes and bounds, and not by the mere lot number assigned to it in the certificate of title. As long as the true intentions of the parties are evident, the mistake will not vitiate the consent of the parties, or affect the validity and binding effect of the contract between them. ○ evidence shows that the designation of the second parcel of land sold as Lot 2034 was merely an oversight or a typographical error. The intention of the parties to the Absolute Sale became unmistakably clear when private respondents, as vendees, took possession of Lots 1320 and 1333 in the concept of owners without the objection of Filomena, the vendor. – Even if the notarized copy indicated the wrong lot, the intent of the parties must prevail. Non-compliance w/ formal requirements does not adversely affect the validity of the contract or the rights and obligations of the parties. MELLIZA vs. CITY OF ILOILO (1968) Sales – Part 3 of Outline (Subject Matter)

1. 2. 3.

Julian Melliza during her lifetime owned, 3 parcels or residential land in Iloilo City. (Lots 2, 5, and 1214). Total area of Lot 1214 was 29, 073 sq. meters. Julian Meliiza donated to the Municipality of Iloilo 9,000 sq. meters of Lot 1214 to serve as site for the municipal hall. The donation was however revoked because of inadequacy to meet the requirements of the Arellano Plan. Subsequently ,Lot 124 was divided to A and B. Still later, B was further subdivided into 1, 2, 3. Lot 1214-B-1 (4,563 sq. m) became as Lot 1214-B; Lot 1214-B-2 (6,653 sq. m) became as Lot 1214-C; Lot 1214-B3 (4,135 sq. m) became Lot 1214-D

4.

5.

6.

7.

8. 9.

Julian Melliza sold her remaining interest to Remedios Villanueva who acquired title to the land. Villanueva transferred her rights to the portion of the land to Pio Sian Melliza who also obtained title. But there was annotation at the back of Pio’s certificate that: a. (a) that a portion of 10,788 square meters of Lot 1214 now designated as Lots Nos. 1214-B-2 and 1214-B-3 of the subdivision plan belongs to the Municipality of Iloilo as per instrument dated November 15, 1932.... On August 24, 1949 the City of Iloilo, which succeeded to the Municipality of Iloilo, donated the city hall site together with the building thereon, to the University of the Philippines (Iloilo branch). The site donated consisted of Lots Nos. 1214-B, 1214-C and 1214-D, with a total area of 15,350 square meters, more or less. Sometime in 1952, the University of the Philippines enclosed the site donated with a wire fence. Pio Sian Melliza thereupon made representations, thru his lawyer, with the city authorities for payment of the value of the lot (Lot 1214-B). No recovery was obtained, because as alleged by plaintiff, the City did not have funds On December 10, 1955 Pio Sian Melliza filed an action in the Court of First Instance of Iloilo against Iloilo City and the University of the Philippines for recovery of Lot 1214-B or of its value. CFI: dismissed complaint of Pio Melliza; instrument already executed by Melliza included in the conveyance of Lot 1214-B. Ca: affirmed CFI decision

Issue: whether or not the conveyance by Juliana Melliza to Iloilo municipality included that portion of Lot 1214 known as Lot 1214-B. HELD:

1.

It should be stressed, also, that the sale to Remedios Sian Villanueva — from which Pio Sian Melliza derived title — did not specifically designate Lot 1214-B, but only such portions of Lot 1214 as were not included in the previous sale to Iloilo municipality (Stipulation of Facts, par. 5, Record on Appeal, p. 23). And thus, if said Lot 1214-B had been included in the prior conveyance to Iloilo municipality, then it was excluded from the sale to Remedios Sian Villanueva and, later, to Pio Sian Melliza.

Issue: true intention of the parties 1. First of all, there is no question that the paramount intention of the parties was to provide Iloilo municipality with lots sufficient or adequate in area for the construction of the Iloilo City hall site, with its avenues and parks. For this matter, a previous donation for this purpose between the same parties was revoked by them, because of inadequacy of the area of the lot donated. 2. Secondly, reading the public instrument in toto, with special reference to the paragraphs describing the lots included in the sale, shows that said instrument describes four parcels of land by their lot numbers and area; and then it goes on to further describe, not only those lots already mentioned, but the lots object of the sale, by stating that said lots are the ones needed for the construction of the city hall site, avenues and parks according to the Arellano plan. If the parties intended merely to cover the specified lots — Lots 2, 5, 1214-C and 1214-D, there would scarcely have been any need for the next paragraph, since these lots are already plainly and very clearly described by their respective lot number and area. 3. The requirement of the law that a sale must have for its object a determinate thing, is fulfilled as long as, at the time the contract is entered into, the object of the sale is capable of being made determinate without the necessity of a new or further agreement between the parties (Art. 1273, old Civil Code; Art. 1460, New Civil Code). The specific mention of some of the lots plus the statement that the lots object of the sale are the ones needed for city hall site, avenues and parks, according to the Arellano plan, sufficiently provides a basis, as of the time of the execution of the contract, for rendering determinate said lots without the need of a new and further agreement of the parties. 4. Furthermore, Pio Sian Melliza, from the stipulation of facts, was the notary public of the public instrument. As such, he was aware of its terms. Said instrument was also registered with the Register of Deeds and such registration was annotated at the back of the corresponding title certificate of Juliana Melliza. From these stipulated facts, it can be inferred that Pio Sian Melliza knew of the aforesaid terms of the instrument or is chargeable with knowledge of them; that knowing so, he should have examined the Arellano plan in relation to the public instrument Exhibit "D"; that, furthermore, he should

5.

have taken notice of the possession first by the Municipality of Iloilo, then by the City of Iloilo and later by the University of the Philippines of Lot 1214-B as part of the city hall site conveyed under that public instrument, and raised proper objections thereto if it was his position that the same was not included in the same. The fact remains that, instead, for twenty long years, Pio Sian Melliza and his predecessors-in-interest, did not object to said possession, nor exercise any act of possession over Lot 1214-B. Applying, therefore, principles of civil law, as well as laches, estoppel, and equity, said lot must necessarily be deemed included in the conveyance in favor of Iloilo municipality, now Iloilo City.

Yu Tek & Co. v Gonzalez Yu Tek and Gonzalez entered into a contract of sale in which the former would pay the latter P3,000 and would in turn be bound to deliver 600 piculs of sugar (of the first and second grade) within a period of three months. Failing to do so within the specified time would result in the recission of the contract, the refund of the P3,000, as well as a penalty of P1,200 which Gonzalez would have to pay. Yu Tek proved before the court that three months had passed with no delivery of sugar. Gonzalez contended that a stipulation was verbally agreed upon, saying that the sugar was to come from the crops which he raised from his plantation. Gonzalez further contended that the crop failed miserably, and thus the delivery could not be possibly effected. The SC struck this down by saying that parol evidence which adds totally new stipulations which are not at all alluded to in the written contract cannot be entertained by the courts. Given that the sugar subject of the contract could not be particularly designated or physically segregated, is there still a perfected contract of sale? Yes there is, under the New Civil Code. Such contract is perfected upon the meeting of the minds between the parties as regards the subject matter of the contract, which should determinate or at least determinable. In the present case, the sugar can be considered a generic thing. Following the principle of “genus never perishes,” Gonzalez cannot now claim to be released from his obligation of delivery since it is, under the contract, impossible to lose. Generic things can always be replaced in fulfilling an obligation. He could have done so and effected a valid delivery of 600 piculs of sugar, but failed to do so within the three months agreed upon. Therefore, he is liable for the refund of the purchase price as well as the P1,200 penalty. JOHANNES SCHUBACK & SONS PHIL. TRADING CORP vs CA Sometime in 1981, Ramon San Jose (doing business under PHILIPPINE SJ INDUSTRIAL TRADING) contacted Schuback Hamburg through the Philippine Consulate General in Germany because he wanted to purchase MAN bus spare parts and he was referred to petitioner, their trading partner in the Philippines. San Jose submitted to petitioner a list of the parts he wanted to purchase with specific part numbers and description and petitioner replied with a letter with a quotation on the items. Dec 17, petitioner submitted its formal offer containing the item number, quantity, part number, description, unit price and total. Dec 24, San Jose informed petitioner of his desire to avail of the prices of the parts at that time and enclosed a Purchase Order w/c contained the item number, part number and description. He promised to submit the quantity per unit he wanted to order on December 28 or 29. Dec 29, San Jose personally submitted the quantities he wanted to Mr. Reichert, General Manager of petitoner, w/c were written in ink by San Jose in the same Purchase Order previously submitted at the bottom of which, San Jose wrote in ink above his signature: "NOTE: Above P.O. will include a 3% discount. The above will serve as our initial P.O.” Petitioner ordered the items from Schuback Hamburg, who in turn ordered the items from NDK, a supplier of MAN spare parts in Germany. Schuback Hamburg sent petitioner a proforma invoice to be used by San Jose in applying for a letter of credit. An order confirmation was later sent by Schuback Hamburg to petitioner w/c was forwarded to and received by San Jose. Petitioner reminded San Jose to open the letter of credit to avoid delay in shipment and payment of interest, who replied, that he was having difficulty with that. In the meantime, Schuback Hamburg received invoices from NDK for partial deliveries and even paid NDK.

Petitioner again reminded San Jose of his order and advised that the case may be endorsed to its lawyers. San Jose replied that he did not make any valid Purchase Order and that there was no definite contract between him and plaintiff. Schuback Hamburg issued a Statement of Account to petitioner charging plaintiff 30% cancellation fee, storage and interest charges in the total amount of DM 51,917.81 w/c was deducted from their account with Schuback Hamburg. Demand letters were sent to San Jose to no avail. Thus, petitioner filed the case. TC: in favor of petitioner CA: reverse decision of TC; no meeting of the minds as to the price ISSUE: W/N a contract of sale has been perfected between the parties. •







A contract of sale is perfected at the moment there is a meeting of minds upon the thing which is the object of the contract and upon the price (Art 1475 NCC) A formal offer was made when petitioner submitted its proposal containing the item number, quantity, part number, description, the unit price and total to San Jose. On Dec 24, San Jose confirmed to purchase on the indicated prices and even issued a Purchase Order. However, it did not contain the quantities per unit but he merely bound himself to submit the quantities a week after, as they were confirmed later on Dec 29. A binding contract of sale existed upon issuance of the purchase order & not upon the confirmation of the buyer of the quantities covered by the order. Perfection did not take place on Dec 29 but on Dec 24. Although the quantity to be ordered was made determinate only on Dec 29, quantity is immaterial in the perfection of a sales contract. What is of importance is the meeting of the minds as to the object and cause, which from the facts disclosed, show that as of Dec 24, these essential elements had already occurred. The opening of a letter of credit in favor of Schuback Hamburg is only a mode of payment & it is not among the essential requirements of a contract of sale (Art 1305 & 1474 NCC) and therefore does not prevent the perfection of the contract between the parties.

PETITION GRANTED. REINSTATE ORDER OF TC.

IV – CONSIDERATION / PRICE Mapalo v. Mapalo Mate v CA Josie Rey and Inocencio Tan went to Fernando Mate’s residence, where Rey (being a cousin of Mate’s wife) appealed to Mate to help her stave off prosecution by Tan. It appears that Rey had issued several rubber checks in favor of Tan which amounted to over P4,000,000. Fearing prosecution under BP 22, Rey convinced Mate to cede to Tan his three lots in Tacloban City which would later on be repurchased by Rey for Mate (as in a pacto de retro sale). Mate agreed to sign the Deed of Sale with Right of Repurchase, on the condition that, among other things, the rate of interest would be 5% per month. It was also demanded by Mate that although the titles would be delivered to Tan, the sale would not be registered in the Register of Deeds. In consideration of this, Rey issued two checks in Mate’s favor (1.4 M for the selling price and 420 K for the interest). Sometime later, Mate deposited these checks in two different banks (he had both a Metrobank and a UCPB account) and (surprise surprise) the checks were dishonored for being drawn against a closed account. Mate tried to track Rey down but she had long gone into hiding. Mate now impugns the pacto de retro sale stating, amongst other things, that the sale was void for lack of consideration. Was there valid consideration in the sale? Yes, there was. While Mate did not actually get the 1.4 M from Tan (the vendee), he had in his possession a postdated check of Rey in an equivalent amount precisely to repurchase the two lots on or before the time provided (six months). He even got another check for 420 K representing the interest. There is absolutely no basis for Mate to file a complaint against Tan to annul the sale on the ground of lack of consideration, invoking his failure to encash the checks. His cause of action was to file a BP 22 case against Rey, which he did. But the filing of the criminal cases was

itself a tacit admission by Mate that there was a consideration of the pacto de retro sale. He is, in effect, stopped from questioning the same.

(Note: Mate also alleged that there existed a condition upon the sale, a stipulation saying that if the checks ended up being dishonored, the sale would be annulled. That’s total BS. Tan was already poised to file criminal cases against Rey for the issuance of past worthless checks. It wouldn’t be logical for Tan to agree to this supposed stipulation since he would, in the event of the checks being rejected, still be left unserved.) Ong v. Ong P1.00 consideration AND OTHER VALUABLE CONSIDERATIONS Imelda Ong executed for and in consideration of P1.00 and other valuable considerations, a Quitclaim Deed in favor of a minor, Sandra Maruzzo where she transferred all her rights over ½ of an undivided portion of a parcel of land. Subsequently, she revoked said Quitclaim and donated the whole property to her son, Rex Ong Jimenez. Maruzzo filed for recovery of ownership/possession and the nullification of the Deed of Donation. Ong contends that the Quitclaim Deed was void as it is equivalent to a Deed of Donation, where acceptance by the donee is required and at the time, Maruzzo was still a minor who was incapable of accepting. TC: Quitclaim = Deed of Sale. There was valid conveyance. Ong appealed and contended that the P1.00 consideration was not consideration at all to amount to a sale. CA affirmed, saying that it is usual practice in deeds of conveyance to place a nominal amount although there is a more valuable consideration given. Issue: w/n there was valid consideration Held: YES, the conveyance to Maruzzo is valid. The conveyance was for and in consideration of P1.00 and other valuable considerations paid by Maruzzo through her representative. It was not just the P1.00 that was the consideration. The execution of a deed conveying ownership of a realty is in itself prima facie evidence of the existence of valuable consideration, the party alleging lack of consideration has the burden of proving such allegation. Bagnas v. CA Republic v Phil. Resources Macario Apostol, president of Phil. Resources, submitted the highest bids for two auctions held by the Bureau of Prisons, one for 100 tons of Palawan almaciga and another for three million board feet of logs. In both cases, he was not able to pay the full bid price, leaving outstanding obligations totaling more than P65,000. In payment thereof and without the company’s knowledge or consent, he surrendered certain goods (e.g. G.I. sheets, black sheets, M.S. plates, round bars and G.I. pipes) to the Bureau in an attempt to settle the said obligations. Upon discovery of this, Phil. Resources demanded the return of the goods from the Bureau but the latter refused. Can the delivery of these goods be considered payment for the outstanding balance of Apostol’s obligations? It is for the Court to decide. The Civil Code provides that price “is always paid in terms of money and the supposed payment being in kind, it is no payment at all.” However, the same article also provides that the purchaser may pay “a price certain in money or its equivalent” which means that payment of the price need not be in money. It is therefore within the province of the Court to decide whether the goods submitted are sufficient as payment. Should the ruling be in the affirmative, it will of course be considered that such is to the detriment of Phil. Resources, assuming that it can prove its ownership over the goods in question. The law, in this case, will certainly protect Phil. Resources’ interests, and it will have adequate legal remedies to resort to should they incur loss as a result of Apostol’s payment.

VELASCO vs CA







Lorenzo Velasco & Magdalena Estate, Inc. entered into a contract of sale involving a lot in New Manila for 100K.The agreement was that Lorenzo would give a down payment of 10K (as evidenced by a receipt) to be followed by 20K (time w/in which to make full down payment was not specified) and the balance of 70K would be paid in installments, the equal monthly amortization to be determined as soon as the 30K had been paid. Lorenzo paid the 10K but when he tendered payment for 20K, Magdalena refused to accept & refused to execute a formal deed of sale. Velasco filed a complaint for damages. Magdalena denied having any dealings/contractual relations w/ Lorenzo. It contends that a portion of the property was being leased by Lorenzo’s sister-in-law, Socorro Velasco who went to their office & they agreed to the sale of the property (30K down payment, 70K on installments+9% interest). Since Socorro was only able to pay 10K, it was merely accepted as deposit & on her request, the receipt was made in the name of Lorenzo. Socorro failed to complete the down payment & neither has she paid the 70K. It was only 2 years after that she tendered payment for 20K & by then, Magdalena considered their offer to sell rescinded. According to Lorenzo, he had requested Socorro to make the necessary contracts & he had authorized her to make negotiations w/ Magdalena on her own name, as he doesn’t understand English. He also uses as evidence the receipt to prove that there already had been a perfected contract to sell as the annotations therein indicated that earnest money for 10K had been received & also the agreed price (100K, 30K dp & bal in 10 yrs) appears thereon. To further prove that it was w/ him & not w/ Socorro that Magdalena dealt with, he showed 5 checks drawn by him for payment of the lease of the property.

W/N there was a consummated sale? NO •

• •

The minds of the parties did not meet in regard to the matter of payment. It is admitted that they still had to meet and agree on how & when the down payment & installments were to be paid. Therefore, it cannot be said that a definite & firm sales agreement between the parties had been perfected. The definite agreement on the manner of payment of the purchase price is an essential element in the formation of a binding & enforceable contract of sale. The fact that Velasco delivered to Magdalena the sum of 10K as part of the down payment that they had to be pay cannot be considered as sufficient proof of the perfection of any purchase & sale agreement between the parties under Art 1428, NCC.

V – FORMATION OF CONTRACT OF SALE OPTION CONTRACT CARCELLER vs CA



Respondent State Investment House, Inc. (SIHI) owns 2 parcels of land in Cebu and it entered into a lease contract w/ option to purchase with Carceller at a monthly rental of 10K for a period of 18 months. The lease contract stated that should Carceller exercise his privilege to purchase, he would have to pay 1.8M (360K down payment & balance to be paid over 60 months w/ 24% interest/annum). • 3 weeks before the expiration of the lease contract, SIHI notified Carceller of the impending termination of the lease & of the short time w/c he could still validly exercise the option. • Carceller wrote a letter to SIHI requesting for a 6 month extension of the lease contract alleging that he needed to raise sufficient funds to be able to exercise his option. 14 days after the expiration of the lease, SIHI informed Carceller that his request was disapproved and instead, offered to lease the property for 1 year at 30K/month and also informed him that the property would be offered for sale to the general public. • Four days later, Carceller notified SIHI of his decision to exercise the option to purchase & made arrangements for the payment of the 360K d/payment. SIHI responded saying that the option period had already lapsed & asked him to vacate w/in 10 days & to pay rental & penalties. • Carceller filed a complaint for specific performance & damages against SIHI. RTC: SIHI to execute deed of sale in favor of Carceller. CA: Affirmed but purchase price should be based on prevailing market value. W/N Carceller should be allowed to exercise the option to purchase despite the alleged delay of SIHI in giving the required notice to him?



An option is a preparatory contract in w/c one party grants to the other, for a fixed period & under specified conditions, the power to decide w/n to enter into a principal contract. It binds the party who has given the option not to enter into a principal contract w/ any other person during the period designated. It is a separate agreement distinct from the contract w/c the parties may enter into upon the consummation of the option. • Carceller’s letter to SIHI was fair notice of his intent to exercise the option, despite the request for the extension of the lease contract. He asked for the extension so that he may be able to raise sufficient funds to exercise his option to buy. Furthermore, if Carceller is not granted the option, it will cause him damage as he had already introduced considerable improvements of the property. • Even thought the option was exercised beyond the option period, it is still a substantial compliance w/ the exercise of the option. Carceller had already given notice of his intention to purchase the property w/in the option period when he asked for the extension. It was only after Carceller’s request for extension was denied that he notified SIHI of his desire to exercise the option formally. The delay of 18 days was neither “substantial” nor “fundamental” and did not amount to a breach that would defeat the intention of the parties when they executed the lease contract w/ option to purchase AFFIRMED but remanded to the RTC to determine the fair market value of the property at the time the option was exercised. Tayag v Lacson A group of farmers/tillers, who farmed on the land owned by the Lacson family, entered into separate Deeds of Assignment with Tayag. According to the Deeds, each of the farmers would waive in favor of Tayag his right to the actual area of land being tilled, provided that there be no legal impediment prejudicing the smooth transfer of lawful ownership of the property in Tayag’s name. The going price would be P50.00 per square meter of land actually being tilled by each farmer. Tayag then started paying various sums to each of the farmers, to be taken as partial payments under the Deed’s terms. To further expedite the process, Tayag called a meeting to work out the implementation of the terms of their separate agreements. However, the farmers wrote back to Tayag, saying that they weren’t planning to attend the meeting, and that they were planning to sell their rights and interests over the land to the Lacson family instead. They also claimed that they were tricked into agreeing on the Deeds, and that they received the various sums from Tayag in the form of loans, not as payments under the Deeds. In his response, Tayag claimed that the farmers were being coerced by the Lacsons to renege on their previous commitments to him and that the Deeds must be honored in the interest of law. The trial court ruled in favor of Tayag (granting him injunctive relief) but the CA reversed, and enjoined the trial court from proceeding with the civil case. Can the Lacsons be enjoined from the disposal or encumbrance of the property? the petitioner was burdened to establish the following: (1) a right in esse or a clear and unmistakable right to be protected; (2) a violation of that right; (3) that there is an urgent and permanent act and urgent necessity for the writ to prevent serious damage No. They were not parties to the Deeds of Assigment between Tayag and the farmers. (1) As duly registered owners of the property, the Lacsons have the right to enjoy and dispose of the property without any other limitations than those established by law. They may enter into contracts covering their property with another under such terms and conditions as they may deem beneficial provided they are not contrary to law, morals, good conduct, public order or public policy. (2) In addition, the Deeds stated that such right in favor of Tayag would be available only if there existed no legal obstacle to his acquisition of the property. There is no showing in the petitioner’s complaint that the respondents had agreed to sell their property, and that the legal impediments to the agreement no longer existed. The petitioner and the defendants-tenants had yet to submit the Deeds of Assignment to the Department of Agrarian Reform which, in turn, had to act on and approve or disapprove the same. In this case, the defendants-tenants-subtenants, under the deeds of assignment, granted to the petitioner not only an option but the exclusive right to buy the landholding. But the grantors were merely the defendants-tenants, and not the respondents, the registered owners of the property. Not being the registered owners of the property, the defendants-tenants could not legally grant to the petitioner the option, much less the “exclusive right” to buy the property. He never even came close to having a claim to the land; there is no right to speak of. It must also be noted that Tayag’s supposed scheme is truly revolting to the conscience. For the measly sum of fifty bucks per square meter, he’d be depriving the farmers/tillers of their interests over the land they work so hard to cultivate. Issue: W/N the deeds of assignment executed are perfected option contracts? Held: NO 1. 2. 3.

An option contract by which the owner of the property agrees with another person that he shall have the right to buy his property at a fixed price within a certain time. It imposes no binding obligation on the person holding the option, aside from the consideration for the offer. Until accepted, it is not, properly speaking, treated as a contract.

4.

An option contract is a separate and distinct contract from which the parties may enter into upon the conjunction of the option.

Villamor v CA – consideration = diff. b/w 70 (price agreed upon to be paid) minus the prevailing market price of the property. Thus, the option could not be withdrawn anytime before the lapse of the period. – Macaria sold ½ of her lot to Sps. Villamor. That this was bought only because she promised to sell her other half at 70. = consideration – Contract of sale perfected when the Villamors accepted the offer.

SANCHEZ V. RIGOS –

Sanchez and Rigos entered into an Option to Purchase. Here, Rigos agreed, promise and committed to Sanchez a parcel of land in Nueva Ecija for 1,510, provided that Sanchez exercise his right to purchase the property within 2 years. Failure to do so within the stipulated period would deem the option terminated and elapsed.



Rigos rejected several tenders of payment by Sanchez, which prompted the latter to deposit the amount in the court and file the case.



Defense: 2nd par. Of 1479: the contract was a unilateral promise to sell, and it being unsupported by any valuable consideration, is null and void. Thus, such an agreement is not binding.



Sanchez: 1st par of 1479: the contract is reciprocally demandable. TC: Rigos must accept and sell the property to Sanchez, using 1324 as basis.



w/n Rigos could refuse the tender of payments, since the Option to Purchase was not supported by any valuable consideration? Which provision governs, 1324 or 1479? Rigos should not have refused the payments, as a valid contract of sale was created when Sanchez exercised his right to purchase within the stipulated period and before withdrawal of the option.



Applicable provision: 2nd par of 1479









The Option was not reciprocally demandable, as it did not impose upon Sanchez the obligation to purchase the property. It merely granted him an option to buy, as clearly indicated in the caption “Option to Purchase.” There is nothing in the contract to indicate that Rigos’ agreement to sell is supported by a consideration distinct from the stipulated price. There has been a conflict in the interpretation of Articles 1324 and 1479, which provides: Southwestern Sugar & Molasses v. Atlantic Gulf & Pacific Co., in essence, states that a unilateral promise to but or to sell, even if accepted, is only binding if supported by a consideration. Thus, the option can still be withdrawn, even if accepted. The case makes a distinction b/w 1324 and 1479, in that the former is the general rule and the latter applies specifically to a promise to buy and sell. But the court recognized that later cases state that such an option, although not binding as a contract in itself for lack of a separate consideration, nevertheless generated a bilateral contract of purchase and sale upon acceptance. ○ More favored view, as reconciles the two provisions.

Vasquez vs. CA (1991) 1. 2. 3.

4.

Spouses Martin Vallejera and Apolonia Olea (respondents) filed this action against Spouses Cipriano Vasquez (petitioners herein) seeking to redeem a lot sold by respondent spouses. Said lot was registered in the name of Vallejera and Olea and the same lot was leased by them to Cipriano and Vasquez (1966-1969). After the execution of the lease, Cipriano and Vasquez took possession of the lot, up to now and devoted the same to the cultivation of sugar. Respondent spouses sold the lot to the petitioners under a Deed of Sale which was duly ratified and notarized. Petitioners got hold then of a TCT. Along with the Deed of Sale is a separate instrument (Right to Repurchase) was executed by them granting the respondent spouses such right to repurchase lot for 12k. (also ratified and notarized) However, after securing petitioners’ title, respondent spouses sold the same lot to Benito Derrama. Upon protestations of defendant, the said second sale was cancelled after the payment of 12 k by petitioners to Derrama.

5.

Petitioner spouses resisted this action for redemption on the premise that the Right to Repurchase is just an option to buy as such is not supported by a considereation distinct from the price, making it not binding to them.

6.

LC: against Cipriano and Vasquez; MR: denied also; CA: in favor of Vallejo and Olea

Issue: w/n there was a valid right to repurchase Held: NONE

1.

2.

3.

4. 5. 6.

It is clear that the right to repurchase was not supported by consideration distinct from the price. The rule is that the promisee has the burden of proving such consideration. Unfortunately, Vallejera and Olea, promisees in the right to repurchase failed to prove such consideration. The record does not show that Vallejera and Olea accepted the right to repurchase. Disagrees with the LC that Vallejera and Olea accepted the right to repurchase as evidenced by the annotation and registration of the same on the back of the TCT in the name of respondent spouses. As such, the Vasquez spouses are estopped from disregarding it. The annotation and registration of the right to repurchase at the back of the TCT of the petitioners cannot be considered as acceptance of the right to repurchase. Its only purpose is to bind purchasers of such registered land. In effect, the annotation of the right to repurchase found at the back of the certificate of title over the subject land of respondent spouses served as a notice of the existence of such unilateral promise of the petitioners to resell the same to respondent spouses. The respondents did not sign the offer. Acceptance should be made by the promisee, in this case, the respondent spouses and not the promisors, the petitioners herein. The respondent spouses ineffectual acceptance then of the option to buy validated the petitioners’ refusal to sell the parcel which can be considered as a withdrawal of the option to buy. Since the transaction between the petitioners and respondents was not a sale with a right to repurchase, the respondents cannot avail of conventional redemption. Petition granted.

Nietes v. CA – accepted – Dr. Garcia entered into a Contract of Lease w/ Option to Buy for 5 yrs., with the option period coinciding with the lease period, for the Angeles Educational Institute – Before the lapse of the period, he tried to rescind the contract for supposed violations by Nietes (like not using the original name of the school). – There was acceptance well within the period, before Garcia tried to rescind. – Pmt of 2K+ , for which a receipt was issued by Garcia, acknowledging it as advance payment for the purchase – Letter by Nietes’ lawyer in reply to the rescission letter saying that they are exercising theit right – The acceptance need not be coupled with payment. Nietes v CA Nietes and Garcia entered into a Contract of Lease with Option to Buy, involving a school in Pampanga which Garcia owned. It was a five-year lease at 5k per year, payable in three installments. However, instead of paying in that manner, Nietes instead made smaller payments, albeit still amounting to the total of 25k – plus an excess of more than 2k paid within the period of lease. Garcia later on sent a letter to Nietes, wishing to rescind the contract for a number of reasons (mainly centered on subpar maintenance). Nietes replied by saying that he never violated any of the terms set forth in the lease contract, and that moreover, he was exercising his option to buy and ready to pay the balance of the purchase price at any time. He then made the necessary deposit of funds (about 84k) at the Agro-Industrial Development Bank. Garcia contends that since no full payment had yet been made, Nietes could not be validly exercising his option to buy. Was full payment of the purchase price necessary for a valid exercise of the option to buy? No. First of all, Garcia is in estoppel. When one of the final payments had been made which actually exceeded the 25k rent, Garcia stated in the receipt that such payment had been made “as advance pay for the school, the Contract of Lease being paid.” In that regard, Garcia was actually acknowledging and assenting to Nietes’ intention to buy the school, as evidenced by his payments exceeding the rent. Also, in the case of an option to buy, the creditor may validly and effectively exercise his right by merely advising the debtor of the former’s decision to buy and expressing his readiness to pay the stipulated price, provided that the same is available and actually delivered to the debtor upon execution and delivery by him of the corresponding deed of sale. In other words, an option to buy is governed by provisions on reciprocal obligations. All the buyer needs to do is to make known his intention to buy and express his capacity to pay the sale price, and in return, the seller needs to execute and deliver the deed of sale. Besides, nowhere was it stipulated that Nietes had to make full payment just to exercise his option. Ang Yu Asuncion vs Court of Appeals –



Ang Yu Asuncion and Keh Tiong leased, for more than 50 years and religiously paying the rent, residential and commercial spaces in Binondo, owned by the Cu Unjiengs and Jose Tan. On several occasions, the lessors informed Ang Yu that they are offering to sell the premises and are giving them priority. Ang Yu asked the lessors to put their offer in writing, which they said they would but they never actually put it in writing. Ang Yu filed the case, claiming that the Cu Unjiengs failed to specify the terms and conditions of the offer to sell and that information was received that they were about to sell the property, so Ang Yu wants to compel the lessors to sell the property to them.

** RTC: The Cu Unjiengs’ offer to sell was never accepted by Ang Yu, for the reason that the parties didn’t’ agree upon the terms, but nevertheless, should the Cu Unjiengs offer their property for sale at a price of P11M or lower, Ang Yu will have the right of first refusal. ○ CA: Affirmed with modification. There will still be a right of first refusal whether the price is lower or above P11M. – The Cu Unjieng spouses executed a deed of sale, selling the property to Buen Realty, for P15M. Buen Realty filed a case asking Ang Yu to vacate, but Ang Yu claimed that Buen bought the land while it was under lis pendens. ○ RTC: Buen Realty’s title is set aside as having been executed in bad faith. Cu Unjieng spouses ordered to sell the property to Ang Yu for P15M. ○ CA: Reversed. Order to sell is without effect. Issue: Are the Cu Unjiengs bound to sell the property to Ang Yu and co.? Held: NO. The stages of a contract of sale are negotiation, perfection, and consummation. Until the contract is perfected, it cannot, as an independent source of obligation, serve as a binding juridical relation. A negotiation is formally initiated by an offer. An imperfect promise (politacion) is merely an offer. Thus, at any time prior to the perfection of the contract, either negotiating party may stop the negotiation, and the offer may be withdrawn. If a period is given to the offeree within which to accept the offer, the following rules govern: 1. If the period is not founded upon or supported by a consideration, the offeror is still free and has the right to withdraw the offer before its acceptance. ○

2.

If the period has a separate consideration, a contract of “option” is deemed perfected and it would be a breach of that contract to withdraw the offer during the agreed period. However, that option is an independent contract by itself, and it is to be distinguished from the projected main agreement (subject matter of the option) which is obviously yet to be concluded. If the optionerofferor withdraws the offer before its acceptance (exercise of the option) by the optionee-offeree, the latter may not sue for specific performance on the proposed contract (“object” of the option) since it has failed to reach its own stage of perfection. The optioner-offeror, however, renders himself liable for damages for breach of the option. Here, the first decisions (**) only granted a right of first refusal. In a right of first refusal, while the object might be made determinate, the exercise of the right, however, would be dependent not only on the grantor’s eventual intention to enter into a binding juridical relation with another, but also on terms, including the price, that obviously are yet to be firmed up. Prior thereto, it can at best be so described as merely belonging to a class of preparatory juridical relations governed not by contracts. The breach of right of first refusal cannot justify corresponding an issuance of a writ of execution under a judgment, nor would it sanction an action for specific performance without negating the indispensable element of consensuality. If Ang Yu is aggrieved by the failure to honor the right of first refusal, the remedy is not a writ of execution on the judgment, but an action for damages. Ang Yu Asuncion vs. CA (1994) FACTS:  Ang Yu Asuncion, et al., (plaintiffs) are tenants/lessees of residential and commercial spaces owned by the Cu Unjieng.  The Cu Unjiengs informed the plaintiffs that they are offering to sell the said premises and are giving them priority.  During the negotiations, the Cu Unjiengs offered a price of P6M, but the plaintiffs counter offered P5M.  Plaintiffs then asked the Cu Unjiengs to put their offer in writing, which the latter agreed.  Plaintiffs then asked that the terms and conditions of the offer to sell be specified, but the Cu Unjiengs did not reply.  Thus, plaintiffs filed a complaint to compel the Cu Unjiengs to sell the property to them.  RTC: offer to sell was never accepted, bec. parties never agreed on the terms and conditions of the proposed sale. However, the court held that the plaintiffs had a right of first refusal in case the property was to be sold at a price lower than P11M.  CA: affirmed the decision of RTC, with modications on that, due to the economy today, right of first refusal should also be made available to plaintiffs if the price is in excess of P11M.  Later, the Cu Unjiengs executed a Deed of Sale in favor of Buen Realty and Development Corporation (defendant), transferring the property to the latter for P15M.  Plaintiffs filed a Motion of Execution praying that the CA ruling be implemented.  RTC: granted the said motion, ordered defendant to execute the Deed of Sale in favor of plaintiffs, in recognition of their right of first refusal.  CA: set aside the said order of the lower court. ISSUE: I. W/N plaintiffs’ right of first refusal be recognized and the property should be sold in their favor. HELD: I. NO 1. Until the contract is perfected, it cannot, as an independent source of obligation, serve as a binding juridical relation.

2. 3. 4. 5. 6.

7. 8.

A contract is perfected when a person (seller), obligates himself, for a price certain, to deliver and to transfer ownership of a thing or right to another (buyer). In the law on sales, the so-called “right of first refusal” is an innovative juridical relation. It cannot be deemed a perfected contract of sale under the NCC. An option or an offer would require, among other things, a clear certainty on both the object and the cause or consideration of the envisioned contract. In a right of first refusal, while the object might be made determinate, the exercise of right, however, would be dependent not only on the grantor’s eventual intention to enter into a binding juridical relation with another but also on terms, including the price, that obviously are yet to be later firmed up. It can be best so described as merely not by contracts buy by, among other laws of general application, the pertinent scattered provisions of the NCC on human conduct. The remedy of plaintiffs is not a writ of execution on the judgment, since there is none to execute, but an action for damages in a proper forum for the purpose.

RIGHT OF FIRST REFUSAL Equatorial Realty Development vs Mayfair Theater



Carmelo owned a parcel of land in Manila. He leased it to Mayfair for a term of 20 years, for use as a motion picture theater. Two years later, Carmelo leased to Mayfair another portion of his property, also for 20 years. ○ Both contracts have the stipulation: “That if the lessor should desire to sell the leased premises, the lessee shall be given 30 days exclusive option to purchase the same. In the event, however, that the leased premises is sold to someone other than the lessee, the lessor is bound and obligated, as it hereby binds and obligates itself, to stipulate in the Deed of Sale thereof that the purchaser shall recognize this lease and be bound by all the terms and conditions thereof.



Mr. Pascal (of Carmelo) informed Yang (Mayfair’s president) that he wanted to sell the entire property, and that a certain Araneta was offering to buy the whole property for $1.2M. Pascal asked Yang if he was willing to buy the property for P6-7M.



Mayfair informed Carmelo that they wanted to purchase the entire property and reminded them of the stipulation in the lease, but Carmelo ignored the letter. Carmelo then sold its entire property to Equatorial for P11.3M.





Mayfair filed an action for specific performance and annulment of the leased premises to Equatorial.



Carmelo and Equatorial claimed: that it had informed Mayfair of its desire but that Mayfair had said it was only interested in buying the area under lease, which was impossible since the property was not a condominium, and that the option to purchase invoked by Mayfair is null and void for lack of consideration. ○ RTC: Dismissed Mayfair’s complaint. It reasoned that the option in the contract of lease was not supported by a separate consideration, and without a consideration, the option is not binding on Carmelo to sell the property to Mayfair. Cited Art 1479. Mayfair cannot compel Carmelo to comply with the promise unless Mayfair establishes the existence of a distinct consideration. Also, Art 1354 (Although the cause is not stated in the contract, it is presumed that it exists and is lawful unless the debtor proves the contrary), and consideration cannot be presumed, because when it comes to an option it is governed particularly by Art 1479, whereby the promissee has the burden of proving the existence of consideration. (This was the doctrine in the case of Sanchez.) ○ CA: The stipulation is a right of first refusal and not an option contract, which was the real intention of the parties. The stipulation is certain as to the object (the sale of the leased premises) but the price for which the object is ot be sold is not stated, so it isn’t an option contract. Also said that the right of first refusal was limited to the leased promises and not the entire property itself. Issue: Is the stipulation a right of first refusal or option contract? Held: Right of first refusal. The deed of option or the option clause in a contract, in order to be valid and enforceable, must, among other things, indicate the definite price at which the person granting the option is willing to sell. Cited case of Ang Yu Asuncion: An unconditional mutual promise to buy and sell, as long as the object is made determinate and the price is fixed, can be obligatory on the parties. An accepted unlitateral promise which specifies the thing to be sold and the price to be paid, when coupled with a valuable consideration distinct and separate from the price, is what may properly be termed a perfect contract of option, and this contract is legally binding. The provision is a right of first refusal, and as such, the requirement of a separate consideration has no applicability. An option is a contract granting a privilege to buy or sell within an agreed time and at a determined price, and it is a separate and distinct contract from that which the parties may enter into, and it must be supported by consideration. However, here the right of first refusal is an integral part of the contracts of lease.

There was a consideration for that right of refusal. The consideration is built into the reciprocal obligations of the parties. The consideration for the lease includes the consideration for the right of first refusal. Mayfair is in effect stating that it consents to lease the premises and to pay the price agreed upon, provided that the lessor should give it the right of first refusal. Carmelo actually acknowledged that Mayfair had the right of first refusal, because it informed Mayfair that it intended to sell the properties. The contract between Carmelo and Equatorial was entered into in bad faith. Since Mayfair has a right of first refusal, it can exercise the right only if the fraudulent sale is first set aside or rescinded. Deed of sale between Carmelo and Equatorial is rescinded. Carmelo is to return the purchase price to Equatorial, and Equatorial is ordered to return ownership of the land to Carmelo. Carmelo is ordered to allow Mayfair to buy the lots for P11.3M. PQUE KINGS ENT. V CA & SANTOS Right of first refusal  offer must be that which was made to the 3rd party

– – – – – – –

Santos owned 8parcels of land in Pque., which she leased to a Frederick Chua. Chua then assigned his rights to the property to Lee Ching Bing, who, in turn, assigned it to Pque. Kings. Said contract of lease contained a right of first refusal, which states that in case the Lessor decides to sell the properties, the lessee shall have the 1st option or priority to buy the properties subject of the lease. Santos then sold the properties to Raymundo for 5M. Upon knowledge of this, Pque Kings wrote a letter to Santos. In response to this, the property was recoveyed to her and Santos offered the property to Pque Kings for 15M. They had 10 days to make good of the offer. They offered to buy it at 5M. But Santos then sold the property to Santos For 9m. 2 days after this, she replied to Pque Kings, saying that the period has already lapsed and that they were not privy to the contract b/w her and Raymundo. Pque Kings: sale was in contravention of their right of first refusal, and there was a collusion b/w Santos and Raymundo (property was reconveyed immediately after their letter, same counsel, etc.) so that the property would be sold at a higher price than 5M.



Santos: deed of assignment did not include the assignment of the option to purchase. Plus, Pque Kings’ pmt of rentals to Raymundo was an acknowledgement of the latter’s status as the new owner. Raymundo: he is not privy to the contract, and thus, cannot bee held liable for it.



TC and CA: there was valid offers made by Santos, which they definitely refused.

Issues: is a right of first refusal enforceable by an action for specific performance?

1. 2. Held: 1.

2. – –

w/n the alleged breach of the right of first option or priority to buy states a valid cause of action. Is the grantee entitled to be offered the same terms as those given to a 3rd party?

YES. YES. The basis of the right of first refusal must be the CURRENT offer to sell of the seller and offer to purchase of any prospective buyer. Only after the optionee fails to exercise its right oof first priority under the same terms and within the period stipulated could the owner validly offer to sell the property to a 3rd person, again, under the same terms as offered to the optionee. To have full compliance with the contractual right of first option, the sale of the properties for 9M, the price for which they were finally sold to Raymundo, should likewise be offered to Pque. Kings. Such a rule applies even without a provision like “all things being equal” in the contract, as in the Bonnevie case.

Other pronouncements: 1. no cause of action under PD1517\ 2. Deed of Assignment included the option to purchase --- ALL rights were transferred 3. Raymundo privy to the contract of lease --- stepped into the shoes of the owner-lessor and assumed all the obligations under the lease contract.

Vazquez vs Ayala Corporation Facts: –

The spouses Vazquez entered into a Memorandum of Agreement (MOA) with Ayala Corp, with Ayala buying from the Vazquez spouses all their shares of stock in Conduit Development. The main asset of Conduit was a 49.9 hectare property in Ayala Alabang, which was then being developed by Conduit under a development plan.



– –





Under the MOA, Ayala was to develop part of the property, while the Vazquez spouses retained part of it (“Retained Area”). Ayala’s part contained 4 lots next to the “Retained Area”, and Ayala agreed to offer these lots for sale to the Vazquez spouses at the prevailing price at the time of the purchase. Paragraph 5.15 of the MOA states: “The buyer agrees to give the sellers first option to purchase four developed lots next to the ‘Retained Area” at the prevailing market price at the time of the purchase.” (An aside: There was trouble with the contractor of Conduit. Conduit’s contractor GP Construction hired a subcontractor, Lancer, who sued GP, Conduit and Ayala. Ayala ended up paying both Lancer and GP the total of P4M and the suit was terminated.) The Vasquez spouses believed that Ayala was obligated to sell the 4 lots to them within 3 years from the date of the MOA, so the spouses sent several “reminder” letters of the approaching deadline. However, no demand was ever made. Ayala finished the development of the four lots, and then offered to be sold to the spouses at the prevailing price in 1990. However, this was rejected by the Vasquez spouses, who wanted to pay at 1984 prices.

Issue: Whether Paragraph 5.15 of the MOA can properly construed as an option contract or a right of first refusal. Held: Right of first refusal. An option is a preparatory contract in which one party grants to another, for a fixed period and at a determined price, the privilege to buy or sell, or to decide whether or not to enter into a principal contract. It binds the party who has given the option not to enter into the principal contract with any other person during the period designated, and within that period, to enter into such contract with the one to whom the option was granted, if the latter should decide to use the option. In a right of first refusal, while the object might be determinate, the exercise of the right would be dependent not only on the grantor’s eventual intention to enter into a binding juridical relation with another but also on terms, including the price, that are yet to be firmed up. Here, in Paragraph 5.15, although it has a definite object (the sale of the subject lots), the period within which they will be offered for sale to petitioners and the price are not specified. The phrase “at the prevailing market price at the time of the purchase” connotes that there is no definite period within which Ayala is bound to reserve the subject lots for petitioners to exercise their privilege to purchase. Neither is there a fixed or determinable price at which the lots will be offered for sale. Furthermore, it is not supported by an independent consideration. And consequently, the offer may be withdrawn anytime by communicating the withdrawal to the other party. In this case, Ayala offered the lots for sale to petitioners at the price of P6,500/square meter, the prevailing market price for the property when the offer was made on June 1990. Insisting on paying for the lots of the prevailing market price in 1984 of P460/square meter, petitioners rejected the offer. Ayala even lowered it to P5,000 but again, petitioners rejected and made a counter offer of P2,000. Ayala then rejected that counter-offer. With this rejection, petitioners lost their right to purchase the subject lots. Paragraph 5.15 is a right of first refusal. Petitioners refused Ayala’s offer to sell, and so they have effectively waived their right to buy the lots.

EARNEST MONEY VILLONCO vs. BORMACHECO (1975) Unqualified acceptance, even if it contained request for changes; receipt of earnest money Francisco N. Cervantes and his wife, Rosario P. Navarra-Cervantes, are the owners of 3 lots in Buendia. Makati with a total area of 3,500 sq. m and were titled. The lots were mortgaged to the Development Bank of the Phil (DBP) as security for a loan of P441,000. The mortgage debt was fully paid on July 10, 1969. Cervantes is the president of Bormaheco, Inc., a dealer and importer of industrial and agricultural machinery. The entire lots are occupied by the building, machinery and equipment of Bormaheco, Inc. and are adjacent to the property of Villonco Realty Company. Negotiations for the sale of the said lots pursued through the intervention of Editha Perez as real estate broker. Bormaheco, Inc., through Cervantes, made a written offer dated February 12, 1964, to Romeo Villonco for the sale of the property. The written offer mentioned the ff: 1. That a deposit of P100,000.00 must be placed as earnest money on the purchase of the above property which will become part payment of the property in the event that the sale is consummated; 2.That this sale is to be consummated only after I shall have also consummated my purchase of another property located at Sta. Ana, Manila; 3. That if my negotiations with said property will not be consummated by reason beyond my control, I will return to you your deposit of P100,000 and the sale of my property to you will not also be consummated; and 4.That final negotiations on both properties can be definitely known after 45 days. The property mentioned in the letter was the land of the National Shipyards & Steel Corporation (Nassco), located at Punta, Sta. Ana, Manila. At the bidding, that land was awarded to Bormaheco, Inc., the highest bidder, for the price of P552,000. The Nassco Board of Directors in its resolution of February 18, 1964 authorized the General Manager to sign the necessary contract. Villonco Realty Company, through Teofilo Villonco, in its letter made a revised counter- offer for the purchase of the property. The counter-offer

was accepted by Cervantes which was in a way a reiteration only of Cervantes’ letter. The check for P100,000 was delivered by Edith Perez de Tagle to Bormaheco, Inc. and was received by Cervantes. In the voucher-receipt evidencing the delivery the broker indicated in her handwriting that the earnest money was "subject to the terms and conditions embodied in Bormaheco's letter". Then, unexpectedly, or 26 days after the signing of the contract of sale, Cervantes returned the earnest money, with interest. Cervantes cited as an excuse the circumstance that "despite the lapse of 45 days from February 12, 1964 there is no certainty yet" for the acquisition of the Punta property. Villonco Realty Company refused to accept the letter and the checks of Bormaheco, Inc. Cervantes sent them by registered mail. When he rescinded the contract, he was already aware that the Punta lot had been awarded to Bormaheco, Inc. Villonco filed a complaint and a notice of lis pendens was annotated at the back of the title of the lands. Bormacheco pleaded the defense that no perfection of sale occured. LC: in favor of petitioner spouses Issue: w/n there was a perfected contract of sale Held: YES 1. There was meeting of the minds upon the matter and consideration of the sale. Bormacheco’s acceptance of the part of the payment of 100k shows that the sale was conditionally consummated. The non-consummation of that purchase would be a negative resolutory condition. In fact, Bormacheco’s bid for the Sta. Ana property was already accepted by Naasco. 2. The agreement between the parties has a subject matter, the price, and the mode of payment and that part of the price was paid. Whenever earnest money is given, it shall be considered as part of the price and proof of the perfection of the contract of sale. 3. The 45 day period was merely an estimate and did not serve as deadline. And as stated above, the condition that Bormacheco should acquire the Sta. Ana property was fulfilled. 4. Cervantes alleged that he did not obtain the consent of his wife, the property being a conjugal one. Court held that such is not sustainable because if he did not obtain so, such would only make the sale voidable. 5. Under the facts of the case, it is evident that Bormacheco acted in gorss and evident bad faith in refusing to satisfy the valid and just demand of Villonco for specific performance. • •

The contract of sale is perfected at the moment of the meeting of the minds upon the thing which is the object of the contract and upon the price. From that moment, the parties may reciprocally demand performance, subject to the provisions of the law governing the form of contracts. Payment of earnest money was part of price and proof of perfection of contract

Concurring opinion of Barredo: 1. The actual contract between the parties is not the letter made by Cervantes but the counter-offer that was given by Villonco. The act of Cervantes in signing his conformity to the counter-offer resulted in a completely perfected sale

SPOUSES DOROMAL V. CA Not clear if the money was in the concept of earnest money

DALION vs CA •

Ruperto Sabesaje (relative of the spouses Dalion) sued to recover ownership over a parcel of land based on a private document of Absolute Sale that he alleged was executed in favor of him by the spouses. • Spouses denied the sale contending that the document was fictitious, his signature forged & the land is conjugal property w/c he & his wife acquired from Saturnina Sabaje. • Spouses also deny claims of Sebasaje that after executing the deed of sale, they had pleaded w/ him to be allowed to administer the land because they did not have any means of livelihood. • Spouses however admitted administering 5 parcels of land w/c belonged to Sabasaje’s grandfather who had already died but never received their 10% & 15% commission for sales of copra & abaca respectively. Sebasaje sued merely to harass, preempt & forestall their threat to sue for these unpaid commissions. TC: Dalion to deliver to Sebaseje the parcel of land & to execute formal deed of conveyance in a public document CA: Affirmed

W/N a public document is necessary for transfer of ownership? •

• •



Dalion impugns that even assuming genuineness of his signature & of the document, the sale is not valid as it is embodied in private document thus, did not convey title/right in question. Art 1358, par.1: “Acts & contracts w/c have for their object the creation, transmission, modification or extension of real rights over immovable property must appear in a public instrument” However, this necessity is only for convenience, not for validity or enforceability. It is not a requirement for the validity of a contract of sale of a parcel of land that this be embodied in a public instrument. A contract of sale is perfected by mere consent & no required form is required for its validity. Parties to a perfected contract of sale have the right to reciprocally demand performance (Art1475), and to observe a particular form if warranted (Art1357).  A sale of real property may be in a private instrument, but that contract is valid & binding between the parties upon its perfection. And a party may compel the other party to execute a public instrument embodying their contract affecting real rights once the contract appearing in a private instrument has been perfected. It was correct for TC to order Dalion to deliver the land & to execute formal deed of conveyance in a public document. When the sale is made through a public instrument, the execution thereof is equivalent to the delivery of the thing (Art1498). Delivery may either be actual (real) or constructive. Thus, delivery may be done by placing the vendee in control & possession of the land (real) or by embodying the sale in a public instrument (constructive).

DECISION AFFIRMED. STATUTE OF FRAUDS Secuya vs. Vda. De Selma (2000) FACTS:

     

     

Lot 5679 was originally sold to Caballero. Caballero then entered into an Agreement of Partition with Sabellona, whereby the latter will receive 1/3 of the portion of the whole property (Lot 5679). Sabellona then sold her portion to Secuya (predecessors of plaintiffs), by means of a private document which was lost. (Sabellona’s only heir admitted and confirmed said sale) Secuya, together with his bro and sis, took possession and cultivated the land. Superales, husband of Secuya’s sister, constructed his house on the said lot, and lived their continuously up to the present. (this was made with permission) Later, Secuya died. Selma, defendant-respondent, bought a portion of the whole Lot 5679; and later bought the remaining portion. The land in question, which was a portion of Lot 5679, is embraced and included within the boundary of the acquired land by Selma. Selma then filed a complaint with the Bgy. Captain asserting her ownership over the whole property. Selma argued that she bought the whole property from Aro, to whom the property was sold to by Caballero’s heirs, as evidenced by a notarized Deed of Sale. RTC: ruled in favor of Selma CA: affirmed decision of RTC.

ISSUE: I. W/N there was a valid transfer of 1/3 portion of Lot 5679 by Caballero to Sebellona, by virtue of the Agreement of Partition. HELD: I. NO 1. The Agreement is to be considered as an express trust, not a partition.

2.

In the said Agreement, Caballero held the portion specified as belonging to Sabellona when the application of ownership (granted by Director of Lands in favor of Caballero) was eventually approved and a sale certificate is to be issued in her name.

3.

However, Caballero never transferred the said lot to Sabellona during her lifetime. Instead, her heirs sold it to Aro (as argued by Selma). Plaintiffs did not even register the said Agreement with the Registry of Property or pay the requisite land taxes. Plaintiffs even insist that there was a sale between Sabellona and Secuya, which was embodied in a private document.

4. 5.

6.

As held before, while a sale of a piece of land appearing in a private deed is binding between the parties, it cannot be considered binding on 3rd persons, if it is not embodied in a public instrument and recorded in the Registry of Property.

Yuvienco v. Dacuycuy



this case refers to the Sotto property in Tacloban which was offered through a letter by Yuvienco (coursed through the attorney Gamboa) to Yao for 6.5M. Yao was given around 19 days to respond to the offer. A telegram was sent accepting the offer, after which Gamboa wrote to Yao that he would be arriving in Tacloban to negotiate the sale. The agreement as to payment was as such: (though not included in the memoranda)

P2,000,000.00 to be paid in full on the date of the execution of the contract; and the balance of P4,500,000.00 shall be fully paid within ninety (90) days thereafter;





But when Gamboa arrived Tacloban City bringing with him the prepared contract to purchase and to sell referred to in his telegram dated July 27, 1978 for the purpose of closing the transactions, however, to the complete surprise of plaintiffs, the defendant (except def. Tacloban City Ice Plant, Inc.) without giving notice to plaintiffs, changed the mode of payment with respect to the balance of P4,500,000.00 by imposing upon plaintiffs to pay same amount within thirty (30) days from execution of the contract instead of the former term of ninety (90) days Yuvienco moved to dismiss on the ground of unenforceability under the Statute of Frauds

Issues: 1. whether or not there was a perfected contract of sale 2. whether or not the claim alleged therein is unenforceable under the Statute of Frauds Held: 1.

NO



it should be noted that the Gamboa’s telegram was that he was going to Tacloban “To negotiate the details” and Yao King Ong's telegram simply says "we agree to buy property". It does not necessarily connote acceptance of the price but instead suggests that the details were to be subject of negotiation.



While Respondents maintain that what the telegram refers to as "details" to be "negotiated" are mere "accidental elements", not the essential elements of the contract, this cannot hold water because it was seen that negotiations have been going on for quite some time, but they have not come to a final agreement, and a meeting of the minds did not materialize. That respondents were all the time agreeable to buy the property may be conceded, but instead of "absolutely" accepting the "certain" offer — if there was one — of the petitioners, they still insisted on further negotiation of details.



it being doubtful whether or not, under Article 1319 of the Civil Code, the said letter may be deemed as an offer to sell that is "certain", and more, the Yao telegram is far from being an "absolute" acceptance under said article, there still there appears to be a cause of action alleged in Paragraphs 8 to 12 of the respondents' complaint, considering it is alleged therein that subsequent to the telegram of Yao, it was agreed that the petitioners would sell the property to respondents for P6.5 M, by paving P2 M down and the balance in 90 days and which agreement was allegedly violated when in the deeds prepared by Atty. Gamboa and taken to Tacloban, only 30 days were given to respondents.



BUT even if there seems to be a cause of action, such is still unenforceable under the Statute of Frauds.

1.

NO. There wasn’t sufficient compliance with the Statute of Frauds because the manner of payment was not included in the letters and telegrams.



Yuvienco initially argued against this motion and asked the court to reject the objection for the simple reason that the contract of sale sued upon in this case is supported by letters and telegrams annexed to the complaint and other papers which will be presented during the trial. This is not well taken. Yao having alleged that the contract is backed up by letters and telegrams, and the same being a sufficient memorandum, the complaint should be given a day in court and allowed to substantiate their allegations.



In other words, the entire case was not dismissed outright just because the contract was backed up by letters and telegrams. However, the central issue now (whether there is a cause of action because of the change in the manner of payment) is UNENFORCEABLE for not being included in said letters and telegrams.



It is nowhere alleged in said paragraphs 8 to 12 of the complaint that there is any writing or memorandum, much less a duly signed agreement to the effect that the price of P6,500,000 fixed by petitioners for the real property herein involved was agreed to be paid not in cash but in installments as alleged by respondents. In any sale of real property on installments, the Statute of Frauds read together with the perfection requirements of Article 1475 of the Civil Code must be understood and applied in the sense that the idea of payment on installments must be in the requisite of a note or memorandum therein contemplated.

Yao’s complaint dismissed. LIMKETKAI SONS MILLING, INCvs.CA • Phil Remnants Co constituted BPI as trustee to manage, administer & sell its real estate property including one in Pasig. Revilla, a real estate broker was given formal authority by BPI to sell the lot for 1K/sqm w/c was concurred in by the owners of Phil Remnants. • Revilla contacted Alfonso Lim of Limketkai Sons who agreed to buy the land. So the officials of Limketkai & Revilla were given permission to enter & view the property. • Revilla informed BPI that he had procured a buyer & 2 officials of Limketkai Sons (Alfonso Lim & Albino Limketkai) went to BPI & met w/ the VP (Albano) and Asst VP (Aromin) of BPI to confirm the sale. They asked that the price be reduced to 900/sqm but Albano stated the price to be 1,100/sqm. They finally agreed that the lot be sold at 1K/sqm to be paid in cash. (Authority to sell was on a first come first served basis & Limketkai Sons was the first comer) • Lim asked if it was possible to pay on terms. BPI’s officials said that there was no harm in trying to ask as the same had been allowed. However, it was understood that should the term payment be disapproved, then price shall be paid in cash. It was Albano who dictated the terms under w/c installments payments may be made & so, Lim wrote to BPI embodying the initial payment of 10% & the 90% w/in 90 days. • 3 days later, Limketkai Sons learned that its offer to pay on terms had been frozen. So Lim went to BPI to tender payment of 33M w/ Albano but he refused stating that the authority to sell the land had been w/drawn from his unit. The same check for 33M was tendered to BPI VP Bona, who also refused. • Limketkai Sons filed an action for specific performance w/ damages. In the course of trial, BPI informed the TC that it had sold the property to Nat’l Book Store (NBS) & the complaint was amended to include NBS. • TC: In favor of Limketkai. CA: reversed W/N bank officials were authorized to enter into the contract? YES • At the start of the transaction, broker Revilla himself had already full authority and was acting for & behalf of BPI. Notwithstanding this, Revilla saw it fit to bring BPI officials into the transaction. If BPI could give authority to licensed broker then there is no reason to doubt the authority to sell of the BPI officials whose precise job is to manage & administer real estate property. • Aromin, BPI’s Asst VP & Trust Officer acted in a perfectly natural manner & showed no indication that we was acting ultra vires. This shows that BPI held him out to the public as the officer routinely handling such transactions. Everything in the record points to the full authority of Aromin to bind the bank (made negotiations w/ Limketkai, etc) except for the self-serving memoranda or letters later produced by BPI that Aromin was an inefficient & undesirable officer and who in fact, was dismissed after he testified. MAIN ISSUE: W/N there was a perfected contract because of Limketkai’s request to pay on terms constituting it as a counter-offer & negotiations were still in progress at that point? PERFECTED CONTRACT • Negotiation/Preparation Stage: authority given by Phil remnants to BPI, authority given by BPI to broker Revilla, the offer to sell to Limketkai, the inspection of the property and the negotiations w/ Aromin & Albano • Perfection: Aromin & Albano, acting for BPI, agreed to sell & officers of Limketkai agreed to buy the lot for 1K/sqm. Aside from this, there was the earlier agreement between Limketkai & Revilla there was a concurrence of offer & acceptance on the object & the cause • The allegation of NBS that there was no concurrence of the offer & acceptance of the cause is belied by Aromin & Albano, the officials w/ whom the contract was perfected  the fact that the deed of sale still had to be signed & notarized does not mean that no contract had already been perfected • The sale of land is valid regardless of the form it may have been entered into. The requisite form under Art 458 is merely for convenience & failure to comply dos not affect its validity or binding effect. If the law requires a document or other special form, as in the sale of real property, the contracting parties may compel each other to observe that form once the contract has been perfected • STATUTE OF FRAUDS APPLICABLE because the sale involved real property. HOWEVER, under Art 1403, an exception to the unenforceability of the Statute of Frauds (when a written contract of sale is not necessary) is the existence of a written note or memorandum



evidencing the contract, w/c may be found in several writing, not only in one document. The memorandum is written evidence that such a contract was entered into. a written note/memorandum, embodying the essentials of the contract & signed by the party charged or his agents suffices to make the verbal agreement enforceable, taking it out of the operation of the statute While in this case there is no written contract, there are abundant notes & memoranda extant in the records evidencing the elements of a written contract. (e.g.: ExhPletter to Amorin authorizing the sale at 1K, giving 2% commission to the broken & instructing that sale be on cash basis; ExhBissued by BPI to Reviila authorizing him to sell property, etc.) Combining all these notes & memoranda, SC held that there is a perfected contract of sale. No particular form of language or instrument is necessary to constitute a memorandum or note in writing under the statute of frauds

W/N NBS is an innocent purchaser for value? NO, NBS IN BAD FAITH • NBS ignored the notice of lis pedens annotated on the title when it bought the lot. It was the willingness & design of NBS to buy property already sold to another party w/c led BPI to dishonor the contract w/ Limketkai. • BADGES OF FRAUD: 1. Sale was supposed to be done through authorized broken but BPI officials personally & directly took over this particular sale when a close friend became interested, 2. BPI Sen VP admitted that NBS Pres was his friend & had lunch meetings to discuss purchase of the lot 3. NBS offered 5m then 7M if Limketkai would drop the case & give up the lot 4. In an area abound w/ buildings, NBS had constructed only a warehouse w/c can easily be dismantled CA DECISION REVERSED, RTC DECISION REINSTATED. Ortega v. Leonardo Partial performance  EXC to Statute of Frauds –

– –

Ortega and Leonardo were both asserting their rights to a parcel of land in Malate for the purposes of purchase. Because of this, an investigation ensued, and Leonardo proposed to Ortega that the latter desist from her claim in exchange for the promise that if and when he succeeded in getting title to the lot, he would sell to her a portion thereof, provided (1) she paid for the surveying and subdivision of the lot and that (2) after he acquired title, she could continue holding the lot as tenant by paying monthly rental until said portion shall haven been segregated and the purchase price fully paid. The offer was accepted and Ortega desisted from her claim. She then caused the survey and segregation and paid the monthly rentals. However, when she tendered payment to defendant the purchase price which the latter refused to accept.

Issue: w/n the contract is unenforceable for not being written, as required by the Statute of Frauds? TC: unenforceable SC: the case falls under the exception, where there is partial performance. There were several circumstances indicating partial performance: 1. the relinquishment of the rights of continued possession 2. the building of improvements, which is considered to the the strongest and most unequivocal act of part performance 3. the tender of payment plus the surveying of the lot at Ortega’s expense and the payment of the rentals It would be fraud if Leonardo was permitted to oppose performance of his part after he was allowed or induced the former to perform in reliance upon the agreement.

CLAUDEL V. CA Statute of frauds applicable – –

– –

This case is basically a dispute over a land in Muntinlupa which was previously owned by Basilio Claudel. He declared the lot in his name and paid the realty taxes, which was passed on to his widow and son after his death. The dispute is between the HEIRS OF CECILIO and the SIBLINGS OF CECILIO. The HEIRS partitioned the land, and the SIBLINGS filed a case to cancel the titles and for reconveyance. The ground for this was that the land was supposedly sold to them prior to Cecilio’s death for P30, but this was only a verbal contract. The CFI dismissed the case because, among others, the SIBLINGS were not able to present any document evidencing the alleged sale, and the Statute of Frauds precludes admission of oral testimony on the sale of real property. Plus, the action has prescribed. CA reversed

Issue: 1. w/n the contract may be proven orally 2. w/n the prescriptive period for filing the action for cancellation and reconveyance should be counted from thesale or from the date of issuance of titles? Held: 1.

NO The rule of thumb is that a sale of land, once consummated, is valid regardless of the form it may have been entered into. For nowhere does law or jurisprudence prescribe that the contract of sale be put in writing before such contract can validly cede or transmit rights over a certain real property between the parties themselves. However, in the event that a third party, as in this case, disputes the ownership of the property, the person against whom that claim is brought can not present any proof of such sale and hence has no means to enforce the contract. Thus the Statute of Frauds was precisely devised to protect the parties in a contract of sale of real property so that no such contract is enforceable unless certain requisites, for purposes of proof, are met. Except under the conditions provided by the Statute of Frauds, the existence of the contract of sale made by Cecilio w/ his siblings cannot be proved. 2. YES The action is supposed to be based on an oral contract, which prescribes in 6 years. More than 6 years has lapsed. The reason that an implied trust in favor of the SIBLINGS OF CECILIO was established in 1972, when the HEIRS OF CECILIO executed a contract of partition over the said properties, is not well-taken. As been pointed out, the law recognizes the superiority of the torrens title. Above all, the torrens title in the possession of the HEIRS OF CECILIO carries more weight as proof of ownership than the survey or subdivision plan of a parcel of land in the name of SIBLINGS OF CECILIO.

ALFREDO v. BORRAS (2003) The owners of the subject lot in this case were petitioner spouses, Godofredo Alfredo and Carmen Limon Alfredo. The Subject Land is covered by an OCT. Private respondents, spouses Armando Borras and Adelia Lobaton Borras, filed a complaint for specific performance against Godofredo and Carmen before the RTC. Armando and Adelia alleged in their complaint that Godofredo and Carmen mortgaged the Subject Land for 7k with DBP. To pay the debt, Carmen and Godofredo sold the Subject Land to Armando and Adelia for 15k, the buyers to pay the DBP loan and its accumulated interest, and the balance to be paid in cash to the sellers. Armando and Adelia gave Godofredo and Carmen the money to pay the loan to DBP which signed the release of mortgage and returned the owner’s duplicate copy of OCT to Godofredo and Carmen. Armando and Adelia subsequently paid the balance of the purchase price of the Subject Land. Godofredo and Carmen then delivered to Adelia the owner’s duplicate copy of OCT, with the document of cancellation of mortgage, official receipts of realty tax payments, and tax declaration in the name of Godofredo. Godofredo and Carmen introduced Armando and Adelia, as the new owners of the Subject Land, to the Natanawans, the old tenants of the Subject Land. Armando and Adelia then took possession of the Subject Land. In January 1994, Armando and Adelia learned that hired persons had entered the Subject Land and were cutting trees under instructions of allegedly new owners of the Subject Land. Subsequently, Armando and Adelia discovered that Godofredo and Carmen had re-sold portions of the Subject Land to several persons. Armando and Adelia filed an adverse claim with the Register of Deeds of Bataan. Armando and Adelia discovered that Godofredo and Carmen had secured an owner’s duplicate copy of OCT after filing a petition in court for the issuance of a new copy. Armando and Adelia wrote Godofredo and Carmen complaining about their acts, but the latter did not reply. Thus, Armando and Adelia filed a complaint for specific performance. Petitioners asserted that the Subsequent Buyers were buyers in good faith and for value. TC: in favor of Armando and Amelia. RTC: there was perfected contract of sale; all the elements are present (object, purchase price, manner of payment); subsequent buyers not in GF. CA: affirmed RTC Issue: w/n there was perfected contract of sale Held: 1.

2.

3.

There was a perfected contract of sale. A contract is perfected once there is consent of the contracting parties on the object certain and on the cause of the obligation. The contract of sale has also been consummated because the sellers and buyers have performed their respective obligations under the contract. In a contract of sale, the seller obligates himself to transfer ownership of the determinate thing sold, and to deliver the same, to the buyer who obligates himself to pay a price certain to the seller. Ownership of the thing sold is transferred to the vendee upon its actual or constructive delivery. Godofredo and Carmen also turned over to Amando and Amelia the documents of ownership. Armando and Amelia paid the full purchase price as evidenced by the receipt. Subsequent buyers not in GF because they were aware of the lis pendens in the title. Also. The settled rules is when ownership or title passes to the buyer, the seller ceases to have any title to transfer to any 3rd person. If the seller sells the same land to another, the 2nd buyer hwo has actual or

4.

constructive knowledge of the prior sale cannot be a registrant in good faith. Such 2nd buyer cannot defeat the 1st buyer’s title. Petition denied.

Toyota Shaw vs CA Facts:



Sometime in June 1989, Sosa wanted to purchase a Toyota Lite Ace, and went to the Toyota Shaw office, where they talked to Bernardo, a sales representative of Toyota. Sosa told Bernardo that he needed the Lite Ace not later than June 17 because he, his family and a balikbayan guest would use it on the 18th to go to Marinduque. He claimed that if he didn’t arrive in his hometown without the new car, he would become a laughingstock. So Bernardo assured him that a car would be ready for pickup. Bernardo signed an agreement, titled “Agreements Between Mr. Sosa and Bernardo of Toyota Shaw.” It stated that the downpayment of P100k would be paid by Sosa, and the Lite Ace will be released on the 17th of June at 10am.



The next day, Sosa delivered the downpayment, and Bernardo printed a Vehicle Sales Proposal (VSP). It stated that payment is by “installment to be financed by B.A. Finance.” The balance would be P270k. The spaces provided for “Delivery Terms” were not filled up. It also contained the provisions: “This sale is subject to availability of unit. Stated price is subject to change without prior notice. Price prevailing and in effect at time of selling will apply.” However, when Sosa went to Toyota Shaw on June 17, according to Sosa, Bernardo made them wait for an hour and told them the car could not be delivered because “nasulot ang unit ng ibang malakas.”





Toyota claims that the car wasn’t delivered because of the disapproval by B.A. Finance of the credit financing application of Sosa.



Sosa wanted his downpayment refunded, which Toyota did (they issued him a check for the P100k). Then Sosa filed a complaint against Toyota for damages. ○ RTC: Ruled in favor of Sosa. He was awarded damages. ○ CA: Affirmed. Issue: Is this a perfected contract of sale? Held: NO.



“Agreements Between Sosa and Bernardo”: not a contract of sale. No obligation on the part of Toyota to transfer ownership of a determinate thing to Sosa and no correlative obligation on the part of the latter to pay a price certain appears on it. The provision on the downpayment of P100k made no specific reference to a sale of a vehicle. Nothing was mentioned about the full purchase price and the manner of payment in installments. It has already been ruled that a definite agreement on the manner of payment of the price is an essential element. Plus, Sosa did not even sign the document, and the title itself shows that he was not dealing with Toyota but with Bernardo (Toyota’s agent). At most, this document can be considered part of the negotiation stage.



VSP: Here, it stated that the balance of the purchase price was to be financed by B.A. Finance Corporation. But B.A. Finance disapproved Sosa’s application. (The Court chose not believe Sosa’s version that claimed that Bernardo said, “nasulot ang unit ng ibang malakas.”) Plus, the VSP was a mere proposal which was aborted in lieu of subsequent events. So the VSP created no demandable right in favor of Sosa for the delivery of the vehicle to him. In addition, no damages should be paid to Sosa. He claims that it was known to the people in his hometown that he would arrive with a Lite Ace, and because of its nondelivery he suffered humiliation, shame, sleepless nights. But at the bottom of this claim is nothing but misplaced pride and ego. He should not have announced his plan to buy a Lite Ace knowing that he might not be able to pay the full purchase price, so it was he who brought embarrassment upon himself.



CONSUMMATION AND PERFECTION Santos vs. Santos (2001) FACTS:  Spouses Jesus and Rosalia Santos owned a parcel of land, which had a four-door apartment administered by Rosalia who rented them out.  The spouses had 5 children, namely Salvador, Calixto, Alberto, Antonio, and Rosa.  The spouses executed a deed of sale of the properties in favor of their children Salvador and Rosa. (a TCT was issued)  Rosa then sold her share to Salvador. (another TCT was issued)  Despite the transfer of the property to Salvador, Rosalia continued to lease and receive rentals from the apartment units.  Later, Jesus died. Six years later, Salvador died. Shortly thereafter, Rosalia died.  Petitioner Zenaida Santos, wife of Salvador, demanded rent from a tenant of Rosalia.





  

Tenant refused, but Zenaida filed an ejectment suit against him, which eventually decided in Zenaida’s favor. Private Respondents, the Santos siblings, filed an action for reconveyance of property, alleging that:

a. b.

the two deeds of sale were simulated, therefore void.

a.

Art. 1477 – ownership of the thing sold is transferred to the vendee upon its actual or constructive delivery.

b.

Art. 1498 – when the sale is made through a public instrument, its execution is equivalent to the delivery of the thing subject of the contract.

c.

Applying these provisions, Salvador became the owner of the subject property by virtue of the 2 deeds of sale.

The two sales were executed to accommodate Salvador in generating funds for his business ventures and providing him with greater business flexibility. RTC: two deeds of sale are null and void for being fictitious or simulated. CA: affirmed decision. Zenaida argues:

ISSUE: I. W/N the execution of a public instrument is equivalent to delivery of the land. HELD: I. EXECUTION OF PUBLIC INST. NOT DELIVERY 1. Nowhere in the NCC does it provide that execution of a deed of sale is a conclusive presumption of delivery of possession. 2. The NCC merely said that the execution shall be equivalent to delivery. The presumption can be rebutted by clear and convincing evidence. 3. Presumptive delivery can be negated by the failure of the vendee to take actual possession of the land sold.

4. 5.

Although the spouses executed a deed of sale, they did not deliver the possession and ownership of the property to Salvador and Rosa. The original sellers retained their control and possession. Therefore, there was no real transfer of ownership.

Addison vs Felix and Tioco (1918) FACTS:

 

Addison sold to defendant Felix and Tioco (her husband) 4 parcels of land. Under the instrument executed: a. Felix is to pay P3000 as downpayment. b. To pay the balance of P2000 on July 1914, then P5000 30 days after the issuance to her of a certificate of title.

c.

To pay, within 10 years, P10 for each cocoanut tree in bearing and P5 for each such tree not in bearing, with the condition that the total price should not exceed P85,000.

d.

 

Felix was to deliver to Addison 25% of the value of the products that she might obtain from the 4 parcels of lands from the moment Felix takes possession of the lands until a TCT has been issued in her favor. e. That within 1 year from the date of the TCT is issued in Felix’ favor, she may rescind the contract. (everyone will return what they have given + interests) Addison filed a suit in CFI to compel Felix to make payment of the 1st installment. Felix’ defense:

a.





Addison had absolutely failed to deliver to her the lands that were the subject matter of the sale. NOTE: After the execution of the deed of sale, Addison designated to a representative of Felix only 2 of the 4 parcels of land. And these 2 lands were in possession of a 3rd person. Felix filed an application for the registration of the 4 parcels of land, which was dismissed for failure to present the required plans within the period required. RTC: ruled in favor of Felix; contract of sale be rescinded. Addison argues: a. Right to rescind not only did not exist from the moment of the execution of the contract up to 1 year after the registration of the land, but does not accrue until the land is registered. b. Right to rescind was subject to a condition, namely the issuance of the title.

ISSUE: I. W/N there was delivery of the thing sold HELD: I. NO DELIVER; FELIX WON!!! 

1. 2.

With respect to two of the parcels of land, Addison was not even able to show them to the purchaser; and as regards the other two, more than 2/3 of their area was in the adverse possession of a 3rd person. The thing is considered to be delivered when it is placed “in the hands and possession of the vendee.”

3. 4. 5.

6.

In order that symbolic delivery may produce the effect of tradition, it is necessary that the vendor shall have had such control over the thing sold. It is not enough to confer upon the purchaser the ownership and the right of possession. The thing sold must be placed in his control. The execution of a public instrument is sufficient for the purposes of the abandonment made by the vendor; but it is not always sufficient. The mere execution of the instrument was not a fulfillment of the vendor’s obligation to deliver the thing sold, and that from such non-fulfillment arises purchaser’s right to demand, as Felix demanded, the rescission of the sale and the return of the price.

Danguilan v. IAC and Melad (1988) – –

Domingo Melad owned 2 lots in Cagayan, a residential lot and a farm lot. Danguilan and Apolonia Melad both claim rights over the said lots.



Melad’s version: based on a Deed of Sale in 1943 signed by Domingo. This sale was supposedly entered into by her mother when she was just a kid, the payment of 80 for such sale earned by her mother at the Tabacalera Factory. She also claimed that she was the illegitimate daughter of Melad, who lived w/ them til he died. (This was later on found to be very suspicious) She explained that she vacated the place because Melad asked her permission to cultivate the land as her tenant who would deliver ½ of the harvest to her. But she filed a complaint since the deliveries have stopped.



Danguilan’s version: Donations in 2 private instruments were made by Domingo to him and his wife (domingo’s niece), in exchange for taking care of the old man, and cultivating his land and with the understanding that he would bury him upon his death.



It is important to note that the TC found that Danguilan was in open and continuous poss’n of the lots in question. Melad’s version that she considered Danguilan as a tenant was not given much weight.

1. w/n there was a valid donation, given that a donation must be made in a public instrument - YES. While truly donations, the conveyances where onerous donations as the properties were given to the petitioner in exchange for his obligation to take care of the donee for the rest of his life and provide for his burial. There was a fair exchange as Domingo died really old, which means that Dnaguilan practically farmed the land by himself. 2. –





Assuming that there was a valid sale to Melad, was the contract consummated? NO. Even assuming the validity of the deed of sale, the record shows that the private respondent did not take possn of the disputed properties and indeed waited until 1962 to file the action for recovery of the lands from the petitioner. If she did have poss’n, she transferred the same to Dnaguilan in 1946, by her own admission. She failed to show that she consummated the contract of sale by actual delivery of the properties to her and actual poss’n thereof in the concept of purchaser-owner. As ro the argument that symbolic delivery was effected through the deed of sale, the Code imposes upon the vendor the obligation to deliver the thing sold and such is done when it is placed in the hands and poss’n of the vendee. In order that symbolic delivery may produce the effect of tradition, control over the thing sold is necessary, and not just ownership and right of poss’n.

Danguilan has a better right to the lots. Pasagui vs. Villablanca (1975) FACTS:  Pasagui bought from co-defendants Bocar a parcel of agricultural land.  A deed of sale was executed, which was notarized and registered in the Registry of Deeds.  Later, defendants Villablanca took possession of the said property, harvesting coconuts, thus depriving Pasagui possession.  Thus, this prompted Pasagui to file a complaint with the CFI against the Villablancas to surrender possession.  Villablancas moved to dismiss the complaint, alleging that since it was a complaint for forcible entry, CFI had no jurisdiction.  RTC: dismissed complaint. MTC has jurisdiction. (at that time, Justice of the Peace) ISSUE: I. W/N action of forcible entry should be granted HELD: I. SHOULDN’T BE GRANTED; PASAGUI NOT IN POSSESSION. 1. Complaint does not allege that the plaintiffs were in physical possession of the land and have been deprived of that possession through force, intimidation, threat, strategy or stealth. 2. It is true that the execution of the deed of absolute sale in a public instrument is equivalent to delivery of the land subject of the sale. This presumptive delivery only holds true when there is no impediment that may prevent the passing of the property from the hands of the vendor into

3. 4. 5.

those of the vendee. It can be negated by the reality that the vendees actually failed to obtain material possession of the land subject of the sale. Pasagui did not physical possession of the land since its purchase. As a matter of fact, their purpose in filing the complaint is precisely to get possession of the property. Forcible entry should be denied because Pasagui had no prior physical possession of the property.

DY, JR. vs. CA (1991) 1. Wilfredo Dy purchased a truck and a farm tractor through financing extended by Libra Finance Investment Corp. Both truck and tractor were mortgaged to Libra as security for the loan 2. Perfecto DY, his brother, wanted to buy the tractor. Perfecto then wrote a letter to Libra for him to purchase such and assume the mortgage. Libra approved the request. And Wilfredo executed a deed of absolute sale. 3. The tractor was withheld by Libra because of Wilfredo’s failure to pay amortizations. Despite offer of Perfecto to pay the full amount, it could not be effected because Libra insisted also for the payment of the truck. A check was then issued to Libra to settle the full amount. 4. A civil case of Wilfredo was also pending in Cebu to recover a sum of money. The provincial sheriff was able to seize and levy on the tractor. The tractor was sold at the pubic auction where Gelac trading was the lone bidder. Gelac later sold it Antonio Gonzales 5. When the check was cleared, it was only then Perfecto learned about the purchase of Gelac. He filed then an action to recover the tractor against Gelac with the RTC of Cebu. RTC: in favor of petitioner; CA: reversed Issue: w/n there was a valid sale to Perfecto Held: VALID 1. Mortgagor who gave the property as security under a chattel mortgage did not part with the ownership over the same. He had the right to sell it although he was under obligation to secure the written consent of the mortgagee. And even if no consent was obtained form the mortgagee, the validity of the sale would still be effected. 2. Court sees no reason why Wilfredo cannot sell the subject tractor. The consent of Libra was even obtained in this case. The sale between the brothers was valid and binding even to the mortgagee. 3. Art. 1496 states that the ownership of the thing sold is acquired by the vendee from the moment it is delivered to him in any of the ways in 1497 to 1501. In this case, actual delivery cannot be made. However, there was constructive delivery already upon the execution of the public instrument and upon the consent and agreement of the parties. While it is true that Wilfredo was not in actual possession and control of the subject tractor, his right of ownership was not divested from his default. Neither could it be said that Libra was the owner because the mortgagee can not become the owner or convert and appropriate himself the property mortgaged. 4. When a third person purchases the mortgaged property, he automatically steps into the shoes of the original mortgagor. 5. The payment of the check was actually intended to extinguish the mortgage obligation so that the tractor could be released to the petitioner. The sale of the subject tractor was already consummated when it was levied upon the by the sheriff. Gelac even knew of the transfer of the property to Perfecto when it received summons. Even with that, they still continued to sell the subject tractor to Antonio.

POWER COMMERCIAL vs. CA (1997) 1. 2. 3. 4. 5.

6.

Power Commercial and Industrial Development Corporation, an industrial asbestos manufacturer, needed a bigger office space and warehouse for its products. For this, it entered into a contract of sale with the spouses Reynaldo and Angelita Quiambao (private respondents). Petitioner assumed, as part of the purchase price, the existing mortgage on the land. In full satisfaction, Reynaldo paid PNB around 79k. June 1 - Spouses again mortgaged said land to PNB to guarantee a loan of which was paid to spouses. Petitioner agreed to assume the payment of the loan. June 26 – Deed of Absolute Sale with Assumption of Mortgage was executed by the parties. On the same date, the manager of Power Commercial submitted to PNB said deed with formal application of the assumption of mortgage. Almost 7 months after said sale, PNB informed spouses that Power Commercial failed to submit papers necessary for the approval of the mortgage. Thus application for assumption was withdrawn. The outstanding balance deemed due and demandable. Power Commercial paid and sent a letter to PNB that there are people who are physically occupying their lot and it is their desire to eject them immediately. With that, they request that their assumption of mortgage be given favorable consideration. PNB replied and asked that Power Commercial remit payments to cover interest and at least part of the principal. Power Commercial then filed a case for rescission and damages before the RTC. Petitioner demanded return of payments it made on the ground that its assumption of mortgage was never approved. Subsequently, the mortgage was closed and was bought by PNB in the public auction.

7. 8.

TC: failure of respondent spouses to deliver actual possession to petitioner entitled the petitioner to rescind the sale. CA: reversed; deed of sale did not obligate the spouses to eject the lessees from the land as a condition of the sale, nor was the occupation by said lessees a violation of the warranty against eviction; no substantial breach to justify rescission

Issue: was there an effective symbolic delivery? (important issue for sales) Held: 1.

There was no express stipulation in the contract that spouses will be guaranteeing ejectment of the occupants although there was a proviso guaranteeing the peaceful possession by the buyer of the land.

2.

Most authorities consider transfer of ownership as the primary purpose of sale, delivery remains an indispensable requisite as our law does not admit the doctrine of transfer of property by mere consent. Civil Code provides that delivery can either be actual (1497) or constructive (1498-1501). Symbolic delivery as a species of constructive delivery, effects the transfer of ownership through the execution of a public document. But if, notwithstanding the execution of the instrument, the purchaser cannot have the enjoyment and material tenancy of the ting and make use of it himself or through another in his name, because such tenancy and enjoyment are opposed by the interposition of another will, then fiction yields to reality – the delivery has not been effected.

3.

In this case, delivery was effected through the execution of deed. The lot sold had been placed under the control of petitioner; thus, the filing of the ejectment suit was subsequently done. It signified that its new owner intended to obtain for itself and to terminate said occupants actual possession thereof. Prior physical delivery or possession is not legally required and the execution of the deed of sale is deemed equivalent to delivery. This deed operates as a formal or symbolic delivery of the property sold and authorizes the buyer to use the document as proof of ownership. Nothing more is required. Petition denied.

BEHN, MEYER & CO. vs YANGCO

• •

• •

Behn Meyer & Yangco entered into a contract of sale of 80 drums of “Carabao” Caustic Soda for P10,063. These were shipped from New York on board the steamship Chinese Prince, w/c was detained by British authorities & 71 drums of caustic soda were removed Yangco refused to accept delivery of the 9 remaining drums on the ground that they were in bad order & also refused the optional offer of Behn (waiting for remainder of the shipment until it arrived) or the offer to substitute the 71 drums of caustic soda of similar grade from Behn’s stock (not of the “Carabao” brand). Behn sold, for the account of Yangco, 80 drums of caustic soda realizing P6,352, w/c was deducted from the selling price of P10,063 making up the amount of damages claimed by Behn for alleged breach of contract. TC: For Yangco.

Issue on place of delivery: • A specification in a contract relative to the payment of freight can be taken to indicate the intention of the parties in regard to the place of delivery. If the buyer is to pay the freight, it is reasonable to suppose that he does so because the goods become his at the point of shipment. On the other hand, if the seller is to pay the freight, it can be inferred that it is the duty of the seller to have the goods transported to their ultimate destination & that title does not pass until the goods have reached their destination. Contract also provided for “c.i.f. Manila, pagadero against delivery of documents.” What does this mean? • In British contracts, “c.i.f.” stands for costs, insurance & freight. They signify that the price fixed covers not only the costs of the goods but the expense of freight & insurance to be paid by the seller. • In American mercantile contracts, the letters “F.O.B.” stands for Free on Board w/c means that the seller shall bear all expenses until the goods ae delivered where they are to be delivered “FOB” at the point of shipment or at the point of destination determines the time when property passes. Both the terms “cif” & “FOB” merely make rules of presumption w/c yield o proof of contrary intention to be ascertained by a consideration of all the circumstances. • In this case, the world “Manila” in conjunction with “c.i.f.” must mean that the contract price, covering costs, insurance & freight, signifies that delivery was to be made at Manila. If Behn Meyer had seriously thought that place of delivery was New York, it would not have gone through the trouble of making fruitless attempts to substitute the goods but would have permitted the entire loss of the shipment to fall upon Yangco. Issue on time of delivery: The contract provided: “Embarque: March, 1916” but the merchandise was shipped only on April 12, 1916. CONCLUSION: The contract was for 80 drums of “Carabao” caustic soda (costs, insurance & freight to be included) to be shipped during March 1916, to be delivered at Manila & paid for on delivery of the documents. The soda offered by Behn was not of the “Carabao” brand & the merchandise was not shipped on March but

April. Behn has not proved performance on its part of the conditions set forth in the contract as it failed to deliver the goods contracted for. THUS, Yangco was rescind the contract of sale because of a breach in substantial particulars going to the essence of the contract and so, Yangco is not liable. The vendee can demand the fulfillment of the contract but as this is shown to be impossible, is relieved of his obligation. JUDGMENT AFFIRMED. Pacific Vegetable Oil Corp. v. Singzon VALLARTA vs CA •

Cruz, offended party, & Vallarta are long time friends & business acquaintances. Cruz entrusted to Vallarta 7 pieces of jewelry. Month laterr, Vallarta decided to buy some items, exchanged 1 item for another & issued a postdated check (5K). Cruz deposited check but was dishonored as Valalrta’s account had been closed. • Vallarta promised to give another check but she didn’t & started avoiding Cruz. So, a criminal action was instituted against her. TC & CA: Vallarta guilty beyond reasonable doubt of estafa. SC: AFFIRMED. • To constitute estafa, act of postdating/issuing a check must be efficient cause of defraudation & should be prior to or simultaneous to act of fraud. Offender must be able to obtain the property because of the issuance of the check. • Transaction entered into by the parties was a “sale on approval” & not a “sale on return”. In SALE OF RETURN, ownership passes to buyer on delivery. Delivery as a mode of acquiring property must be in consequence of a contract. • In this case, ownership on the jewelry was made when the check was issued because it was only then that Vallarta, signified her acceptance to the seller, Cruz & the purchase price was agreed upon. (SALE ON APPROVAL) • Thus, when the check was issued, it was not for the payment of a pre-existing obligation. JUDGMENT AFFIRMED. Santa Ana, Jr vs Hernandez





The Santa Ana spouses owned a 115k square meter parcel of land in Bulacan. They sold 2 separate portions for P11k to Rosa Hernandez. After the sale (there were two other previous sales to different vendees of other portions of the land) the spouses caused the preparation of a subdivision plan. Rosa Hernandez, however, did not conform to the plan and caused the preparation of a different one. The spouses filed a complaint, claiming that Hernandez was occupying an excess of 17k square meters. Hernandez claimed that the alleged excess was part of the areas that she bought. ○ RTC: Ordered Hernandez to vacate the excess. ○ CA: Reversed. Hernandez is the owner of the whole of the lots in her own subdivision plan. Also declared this was a sale of real estate for a lump sum. Applied Art 1542. Found as fact that “the two parcels of land sold to Hernandez were identified by the conspicuous boundaries.” This was a long and continuous dike on the property that separated the lands in question from the others. ○ The spouses appealed, claiming that the recited area in the deed should be controlling and that the boundaries given in the deed are indefinite. They point out that in the deed, the southern boundary of one parcel is merely given as “lupang kasanib” and that the same concurs with the western boundary of the other parcel, which is recited as “lupang kasanib.”

Issue: Does Art 1542 apply? Held: YES. CA decision affirmed. – CA’s findings of fact cannot be questioned at this stage.

– –



The sale was of a definite and identified tract, a corpus certum, that obligated the vendors to deliver all the land within the boundaries, irrespective of whether its real area should be greater or smaller than what is recited in the deed. To hold the buyer to no more than the area recited on the deed, it must be made clear therein that the sale was made by unit of measure at a definite price for each unit. If the buyer intended to buy by the meter, he should have so stated in the contract. The ruling of the SC of Spain in construing Art 1471 of the Spanish Civil Code (copied verbatim in our Art 1542) is highly persuasive: As between the absence of a recital of a given price per unit of measurement, and the specification of the total area sold, the former must prevail and determines the applicability of the norms concerning sales for a lump sum. DOUBLE SALES

Naawan Community Rural Bank vs. CA

Naval v. CA Carillo vs Court of Appeals













Maria Gonzales filed a complaint against the spouses Priscilla and Jose Manio with the RT of Davao del Sur. Gonzales sought the execution of the deed of sale in her favor for the property she bought from Priscilla. Gonzales alleged that she paid P10k (as downpayment) on the P400k purchase price of the lot which actually belong to Aristotle, Priscilla’s son (Priscilla had an SPA from him). They also agreed that the balance would be paid within 3 months. But Priscilla did not execute the deed of sale after demands. So Gonzales filed an action for specific performance. The RTC held: In favor of Gonzales. The sheriff served a copy of the decision to Priscilla at midnight of August 4, 1990. The writ of execution, however, was not served upon the Manios because they could not be located, acc to the sheriff. Gonzales also filed a motion for the nullification of the owner’s duplicate certificate of title and asked that a new certificate be issued in her name. The RTC thus declared the owner’s duplicate certificate of title void, and issued a new TCT under Gonzales’ name. Then, herein respondents Maria Paz Dabon and Rosalina Dabon filed a petition before the CA for annulment of judgment and orders of the RTC. They claimed to have bought the lot from Aristotle Manio first. They allege that the judgment of the RTC was void because of lack of jurisd over their persons as the real parties in interest, and that they were fraudulently deprived of their right to due process. The CA held: The contract of sale between Gonzales and Priscilla was unenforceable because it was evidenced by a handwritten note which was vague as to the amount and which was not notarized, the trial court did not acquire jurisdiction over the indispensable parties, and the proceedings were attended with fraud. Gonzales claims that the CA should have applied the doctrine of double sale to settle the issue of ownership and declare her the true owner of the property.

Issue: Was there sufficient basis to annul the judgment? YES. First of all, the land actually belonged to Aristotle and not Priscilla, who was not acting in her own name. Accordingly, the failure to implead Aristotle renders all proceedings null and void. There are two grounds for annulment of judgment: extrinsic fraud and lack of jurisdiction. Extrinsic fraud is when a party has been prevented by fraud or deception from presenting his case. The overriding consideration is that the fraudulent scheme of the prevailing litigant prevented a party from having his day in court. The CA found that indices of fraud attended the RTC case: Gonzales deliberately excluded the Dabons despite knowing that they bought the land from Aristotle, the sheriff served the copy of the decision at midnight on a Saturday, the trial court ordered Gonzales to deposit the full payment but subsequently ordered the withdrawal, and there was no notice given to the person named in the certificate of title which Gonzales wanted annulled. Issue: Does Art 1544 (on double sales) apply here? NO. Gonzales claims that Art 1544 should be applied, and that she owns the land because she bought the lot on April 26, 1988, while the Dabons allegedly bought it on Oct 19, 1989. But the action for annulment of judgment, under the Rules of Court, does not involve the merits of the final order of the trial court. The issue of whether before the SC is a case of double sale is outside the scope of the present petition. The appellate court only allowed the reception of extraneous evidence to determine extrinsic fraud. To determine which sale was valid, review of evidence is necessary and this Court cannot do that. Petition denied. CA decision affirmed. Consolidated Rural Bank (Cagayan Valley), Inc vs CA



The Madrid brothers (Rizal, Anselmo, Gregorio, Filomeno, and Domingo) were registered owners of a parcel of land in Isabela. They subdivided the land under a subdivision plan, and one of the subdivision lots was Lot 7036-A-7. Rizal Madrid sold part of his share to Gamiao and Dayag, and Rizal’s brothers did not object. The deed of sale for this sale was not registered, but Gamiao and Dayag declared the property for taxation purposes.



Gamiao and Dayag then sold the southern part to Dela Cruz, and the northern part to Hernandez. Dela Cruz and Hernandez took possession of the land and cultivated it. Hernandez then donated his part to his daughter, while Dela Cruz’s children continued possession of their respective part.

– –



Then, in another deed of sale, the Madrid brothers conveyed all their rights and interests over Lot 7036-A-7 to Marquez. This deed of sale was registered. Marquez then subdivided the lot and mortgage part of it to Consolidated Rural Bank (CRB) to secure a loan of P100k, and another part to the Rural Bank of Cauayan (RCB) to secure a loan of P10k. CRB then foreclosed the land because Marquez failed to pay. Marquez then sold another part of the land (it wasn’t mortgaged) to Calixto.



The Heirs of Dela Cruz filed a case for reconveyance and damages of the southern portion of Lot 7036-A against Marquez, Calixto, RBC and CRB. ○ Marquez argues that apart from being the first registrant, he was a buyer in good faith and for value. He also argues that the sale by Rizal Madrid to Gamiao and Dayag was not binding upon him since it wasn’t registered. CRB also alleges that it was a mortgagee in good faith. ○ RTC: Marquez is the lawful owner of the lot and the mortgages are valid. Applied Art 1544.



CA: Reversed. The heirs of Dela Cruz are the lawful owners of the southern half portion and the daughter of Hernandez of the northern half portion. The deed of sale between the Madrid brothers and Marquez is void, and so are the mortgages.

Issue: Who has right over the property? Held: The Heirs of Dela Cruz, NOT Marquez. Art 1544 does not apply in this case. Art 1544 contemplates a case of double or multiple sales by a single vendor. More specifically, it covers a situation where a single vendor sold one and the same immovable property to two or more buyers. It is necessary that the conveyance must have been made by a party who has an existing right in the thing and the power to dispose of it. It cannot be invoked where the two different contracts of sale are made by two different persons, one of them not being the owner of the property. And even if the sale was made by the same person, if the second sale was made when such person was no longer the owner, because it had been acquired by the first purchaser in full dominion, the second purchaser cannot acquire any right. In the case at bar, the property was not transferred to several buyers by a single vendor. In the FIRST deed of sale, the vendors were Gamiao and Dayag. Their right to the property originates from their buying it from Rizal Madrid and his brothers all conformed to the sale. G & D declared the property for tax purposes too. On the other hand, the vendors in the LATER deed of sale were the Madrid brothers, but at that time they were no longer the owners since they had long before disposed of the property in favor of G & D. In a situation where not all the req’s are present in order to apply Art 1544, the principle of prior tempore, potior jure or simply “he who is first in time is preferred in right” should apply. In this case, the sale to the Heirs by G & D, who first bought it from Rizal Madrid, was before the sale by the Madrid brothers to Marquez. The Heirs had also possessed the property for some time. So the Heirs have a superior right. One can sell only what one owns or is authorized to sell, and the buyer can acquire no more than what the seller can transfer legally. In this case, since the Madrid brothers were no longer the owners of the subject property at the time of the sale to Marquez, Marquez did not acquire any right. Even if Art 1544 did apply here, Marquez was not a purchaser and registrant in good faith. Marquez knew at the time of the sale that the property was being claimed or “taken” by the Heirs. He testified that he did not even know who was in possession. The actual possession by people other than the vendor should put the purchaser upon inquiry. That also applies to CRB, since banks, are expected to exercise more care and diligence. Since they merely relied on the certificates of title and failed to ascertain the status of the mortgaged properties (which is supposed to be standard procedure), CRB is a mortgagee in bad faith. Carbonell vs CA –



– –

– –

Jose Poncio owned a parcel of land in Rizal covered by a TCT and subject to a mortgage in favor of Republic Savings Bank. Rosario Carbonell is Poncio’s cousin and lives in the adjoining lot. Emma Infante also lives adjacent to their lots. Carbonell and Infante offered to buy the lot from Poncio. Poncio was unable to keep up with the installments due on the mortgage so he approached Carbonell and offered to sell the lot excluding the house. Carbonell accepted. Poncio and Carbonell then made a document in the Batanes dialect, which read, “Contract for ½ lot which I bought from Jose Poncio.” It indicated that Poncio can live on the lot he sold to Carbonell for a year, without rent. When Carbonell arrived at Poncio’s house, however, Poncio told Carbonell that he could no longer proceed with the sale since he had already given the lot to Infante, and that he could not withdraw from his deal with Infante even if he had to go to jail. Infante then erected a wall around the lot with a gate. Carbonell registered an adverse claim on Feb 8, 1955. Infante’s deed of sale was registered on Feb 12, 1955. A TCT was issued to Infante with the annotation of Carbonell’s adverse claim. Carbonell then filed a complaint against Poncio and Infante asking that the land be declared in her favor.

Issue: Who has right over the land? Held: Carbonell has superior right. The TCT in favor of Infante is cancelled. – It is essential that the buyer must act in good faith in registering his deed of sale to merit the protection of the second paragraph of Art 1544 (it directs that ownership of immovable property should be recognized in favor of one who in good faith first recorded his right). If there is no inscription, what is decisive is prior possession in good faith. If there is inscription, prior registration in good faith is a pre-condition to superior title. – When Carbonell bought the lot, she was the only buyer and the title of Poncio was still in his name solely encumbered by bank mortgage duly annotated thereon. Carbonell was not aware of any sale to Infante as there was no sale to Infante then. Hence, Carbonell’s prior purchase of the land was made

in good faith. Her good faith subsisted and continued to exist when she recorded her adverse claim 4 days before Infante’s registration of deed of sale.



Infante was in bad faith: She refused to see Carbonell, and did not harbor any suspicion even when Poncio was not in possession of the mortgage passbook and his copy of the mortgage contract (because they were given to Carbonell). Also, the notice of the adverse claim was made prior to the registration of her sale, therefore she had knowledge of that prior sale.

Teehankee’s separate opinion: – Both buyers may be deemed to be purchasers in good faith at the dates of purchase. – The seller’s fraudulent act of informing the first buyer that he had wrongfully sold his property to another cannot be to the detriment of the first buyer. – The governing principle is first in time, first in right. Knowledge gained by the first buyer of the second sale cannot defeat the first buyer’s rights except when the second buyer first registers in good faith the second sale against of the first. Such knowledge of the first buyer does not bar her from availing of her rights under the law, among them, to register first her purchase as against the second buyer. Knowledge gained by the second buyer of the first sale defeats his rights, even if he is first to register the second sale, since such knowledge taints his registration with bad faith. – Before the second buyer can have priority over the first, he must show that he acted in good faith throughout (ignorance of the first sale and of the first buyer’s rights) from the time of acquisition until the title is transferred to him by registration or delivery of possession. The second buyer must show continuing good faith and innocence of the first sale until his contract ripens into full ownership. CHENG vs. GENATO (1998) 1. 2.

3. 4. 5. 6. 7. 8.

9.

Ramon Genato is the owner of 2 lots in Bulacan covered by a TCT. He entered into a contract with Da Jose spouses over the 2 lots. The contract was in a public instrument and was duly annotated. There was a condition stated in the annotation that full payment will be paid in within 30 days after payment of downpayment worth 50k. Da Jose spouses asked for an extension when stipulated time of payment came. Pending the effectivity of the extension period, without notice to the spouses, Genato executed an Affidavit to Annul the Contract to Sell (affidavit). But such was not annotated at the back of the titles right away. Subsequently, Cheng expressed interest in the lots. Genato showed the TCT’s and the annotations at the back and the Affidavit. Cheng issued a check for 50k upon the assurance that the contract with the Da Jose spouses will be annulled for which Genato issued a handwritten receipt. Genato deposited the check and Cheng kept on reminding him to register the affidavit. Genato then caused such registration. Genato met by coincidence in the registry the Da Jose spouses. The spouses were shocked to know about the affidavit because the extension they asked was still in effect. With that, Genato decided to continue the Contract he had with them. Genato advised Cheng of his decision of not pursuing the sale with him. Cheng sent a letter demanding of compliance as for him there was already a perfected contract of sale. Genato returned the check but was refused by Cheng. Nevertheless, the contract with the spouses still pushed through Cheng instituted a complaint for specific performance to compel Genato. LC held that the receipt issued by Genato to Cheng meant a sale and not just a priority or an option to buy. Also, there was a valid rescission by virtue of the affidavit and concluded that Cheng should be preferred over the spouses. However on appeal, the CA reversed the decision and that the prior contract to sell in favor of the spouses was not validly rescinded and that Cheng should pay damages for being in bad faith.

Issue: w/n the contract to sell with regard to the spouses was validly rescinded – There can be no rescission of an obligation that is still non-existent, the suspensive condition not having occurred yet – No default can be ascribed to the spouses since the period has not yet expired – Genato should have made notice to the spouses regarding the rescission but he did not do so – Execution by Genato of the affidavit to annul the contract is not called for. Even with or without the affidavit, their non-payment to complete the full downpayment ipso facto avoids their contract to sell, it being subject to a suspensive condition

– –

When a contract is subject to suspensive condition, its effectivity can take place only if anf when the event which constitutes the condition happens or is fulfilled. If the suspensive condition does not take place, the parties would stand as if the conditional obligation had never existed Thus, Cheng’s contention that the contract to sell between Genato and spouses was rescinded due to Genato’s unilateral rescission finds no support in this case

Issue: w/n Cheng’s contract with Genato was not just a contract to sell but a conditional contract of dale which gave him better rights, thus precluding the application of 1544 – Cheng was inconsistent in characterizing the contract he allegedly entered into – In fact, both courts below correctly held that the receipt which was the result of their agreement is a contract to sell









– –

Even if it be treated as conditional contract of sale, it did not acquire obligatory force since it was subject to suspensive condition that the earlier contract of the spouses be cancelled first – a condition never met, as Genato redeemed himself by maintaining the contract with the spouses A reading of 1544 is not apropos to the case. The following circumstance must occur: ○ That 2 or more sales transactions in issue must pertain to exactly the same subject matter, and must be valid sales transactions ○ The 2 or more buyers at odds over the rightful ownership of the subject matter must each represent conflicting interests ○ The 2 or more buyers at odds over the rightful ownership of the subject matter must each have bought from the very same seller These situations are obviously lacking. But even though, 1544 should still apply to the situation. FIRST IN TIME, FIRST IN RIGHT. Not only was the contract between the spouses first in time, it was also registered before the intrusion of Cheng The rule of 1544 for the 2nd buyer to be able to displace the 1st buyer are:

○ ○

That the 2nd buyer must show that he acted in good faith from the time of acquisition until title is transferred That 2nd buyer must show continuing good faith until contract ripens into full ownership

The knowledge gained by Cheng of the 1st transaction defeats his rights even if he is the first to register the transaction since he can be considered in bad faith Good faith must concur with registration for such prior right to be enforceable. The annotation made by the spouses on the titles more than satisy the requirement.

Issue: w/n it was error to hold him liable for damages – No, Genato in bad faith when he filed suite for specific performance knowing fully well that his agreement with Genato did not push through CA did not commit reversible error. Petition denied. MENDOZA vs. KALAW

1. 2.

3.

Agripino Mendoza presented to the CFI for the registration under the Torrens system, of a parcel of land. He alleged that he was the owner in fee simple for the reason that he purchased it from Federico Cañet. Accompanying the petition, there was a united plan containing technical description of the metes and bounds of said parcel of land. The oppositor, Primitivo Kalae, alleging that he was the owner of the same and he had acquired it also from Federico. Honorable James Ostrand held that Mendoza was owner in fee simple and ordered it registered in Mendoza’s name. Therefore this appeal.

Examination of evidence: 1. September 24, 1919 = conditional sale to Kalaw

2. 3. 4. 5.

November 8, 1919 = absolute sale to Mendoza Novemeber 12 = Mendoza took actual possession After Mendoza fenced and cleansed the lot, representative of Kalaw attempted to obtain possession. Kalaw tried to have it registererd but was denied to defect in description of property. Registry however made an anotacion preventiva

Held:

1. 2. 3.

It will be noted that while the absolute sale was subsequent to the conditional sale, Mendoza obtained possession of the property first. Mendoza actually paid his vendor the purchase price of the property, while the payment by the oppositor depended upon the performance of certain conditions in the contract of sale. A conditional sale, before the performance of the condition, can hardly be said to be a sale of property, especially where the condition has not been complied with. Neither can action preventiva obtained by Kalaw be said to have created advantage in his favor for such only protects the rights of the person securing it for a period of thirty days.

4.

In the case at bar, Mendoza had acquired an absolute deed in the land and had actually entered possession before the preventive precautionary. Under the provisions of the Mortgage Law, it could in no way affect the rights or interests of persons, acquired theretofore.

5.

Registration of Mendoza affirmed.

Adalin v CA Palanca and her siblings (the Kado siblings) owned a parcel of land in Cotabato City on which stood a commercial building which they were leasing to Loreto Adalin and two other people (Demetrio Adaya and

Magno Adalin). It was alleged that the lease agreement also included a 30-day option to buy the land if the Kados ever decided to sell it. The Kados commissioned Bautista to look for buyers, as they wanted to sell the property. Bautista offered it to Yu and Lim, who expressed their interest. It was arranged that Yu and Lim would buy two doors each while Loreto Adalin would buy the one door which he was already leasing, all for the price of 2.6M. Yu and Lim asked if the lessees were interested in buying the property but Palanca answered in the negative, saying that they had been offered to buy it for a long time and they had never been interested. It was then arranged that Yu, Lim, and Adalin would pay 300k as downpayment, the balance of which would come once the lessees were evicted from the property. All this was evidenced in a “Deed of Conditional Sale.” Palanca then wrote a letter to the lessees telling them that they had 30 days to vacate the building. She even filed a complaint for unlawful detainer against them. A few days later, both lessees told her that they were ready to exercise the option and buy the property they were occupying, totaling 600k each. The Kados then backed out from their dealings with Yu and Lim, and instead signed three different Deeds of Sale of Registered Land (for 1M, 3M, and 860K) in favor of the lessees (including BOTH Adalins). They then refunded the downpayment tendered by Yu and Lim, plus interest. The latter refused to accept it, and filed this case. The lower court ruled for the lessees, saying that the sale to Yu and Lim (and Adalin) was based on a condition (eviction) which was not fulfilled, hence the sale was never perfected. However, the CA reversed the decision and ruled in favor of Yu and Lim. Was the “conditional sale” executed by the Kados in favor of Yu and Lim voided as a result of the failure to evict the lessees? No. The SC held that it is important to look into the nuances of the contract to ascertain the parties’ true intentions. While it is true that, subject to the lessees’ eviction, the payment of the balance and the subsequent execution of the “Deed of Absolute Sale” were suspended by a condition, the CHOICE of who to sell the land to was NOT conditional anymore. It had been decided. It was absolute. The property was going to be sold to Yu and Lim. The lessees’ contention that they had a right to exercise their option likewise fails, for two reasons: 1) the option was never supported by a separate consideration, and 2) the lessees had already said that they couldn’t buy the land because they couldn’t afford it. They are thus stopped from questioning their own statements. In any case, the SC emphasized that Palanca’s acts smacked of bad faith, most particularly her double dealings with the lessees and the buyers (Yu and Lim). In fact, her filing of the unlawful detainer complaint against the lessees proves that she knew exactly what the contract between the Kados and the buyers was all about. For her to turn around and abandon her obligations to them was utterly unethical and distasteful. San Lorenzo Development Corporation v CA The Lus (Miguel and Pacita) owned two parcels of land in Laguna, which they sold to Babasanta for P15.00 per square meter. Babasanta made a downpayment of 50k, as well as other payments totaling 200k. In 1989, Babasanta wrote to the Lus and asked for a final deed of sale to be executed in his favor, so that he could pay the remaining balance. He also said that he had learned of the land being sold to someone else without his knowledge and consent. He demanded that the second sale be cancelled and a deed of sale to be issued in his favor. In response, Pacita Lu said that when the balance fell due, Babasanta asked for a price reduction, and when denied, backed out of negotiations altogether. She also said that she had returned the sum of 50k to him through Eugenio Oya. Babasanta filed for Specific Performance. The Lus alleged that Pacita Lu obtained loans from Babasanta which reached 50k, and after that point, without Miguel’s knowledge and consent, Pacita and Babasanta agreed to convert the contract into one of sale, selling the properties to Babasanta with the indebtedness of Pacita (50k) as downpayment. Babasanta had made payments totaling 200k, leaving a balance of 260k, which despite repeated demands, he would not pay. He then asked for a reduction but was denied. He then asked that the contract be reverted back into one of loan, making the Lus indebted to him for 200k. They purchased an Interbank Manager’s Check for 200k to show that they were ready to settle Pacita’s debt. After some time, SLDC intervened and stated that they had an option on the property supported by a separate consideration of more than 316k. They exercised the option and had a Deed of Absolute Sale with Mortgage issued in their favor. They further averred that they had no reason to suspect any defect since Babasanta’s claims were never annotated in the certificates of title (in short, IPV ang SLDC). The trial court applied Art. 1544 of the NCC and ruled that since both sales were unregistered, SLDC had the better right, since by having the public Deed executed, there was already symbolic delivery and SLDC had therefore taken possession of the land first. The CA reversed, saying that SLDC was not a purchaser in good faith. They reasoned that Pacita had obtained from SLDC a cash advance of 200k (presumably to settle her debt with Babasanta), and SLDC was thus put on inquiry of a prior transaction on the property. Babasanta also claimed that a notice of lis pendens was on the file with the RD, so SLDC could not have been an IPV. Does SLDC qualify as a buyer in good faith? Yes. It must be stressed that from the execution of the first deed up to the moment of transfer and delivery of possession of the lands to SLDC, it had acted in good faith. The subsequent notice of lis pendens has no effect whatsoever on the consummated sale between SLDC and the Lus. Babasanta argues that the registration of the notice served as constructive notice of their interest in the lands, but again, this registration was made LONG AFTER the sale to SLDC was consummated. Even assuming that this is material, Babasanta still cannot claim a superior right since he already had notice of a previous sale (as evidenced by his letter). He cannot therefore be considered a buyer in good faith either. The law, after all, speaks not only of one criterion. The

first is priority of entry in the registry of property; there being none, the second is priority of possession; and in the absence of both, the third is the date of the title. In any event, since SLDC acquired possession of the property in good faith in contrast to Babasanta, who neither registered nor possessed the property at any time, SLDC’s right is definitely the superior one. AGRICULTURAL & HOME EXTENSION DEVELOPMENT GROUP (AHDG) vs CA In 1972, Diaz and Mia sold a parcel of land to Gundran. The owner’s duplicate copy was given to Gundran but the sale was unregistered because of existing notices of lis pendens on the title. Gundran and AHDG entered into a joint venture agreement for the improvement & subdivision of the land. This agreement was also not annotated. In 1976, Diaz & Mia again sold the same property to Cabautan. By virtue of a court order, a new owner’s copy of the title was issued (they supposedly lost their copy). The notice of lis pendens was canceled and the deed of sale was recorded. A new TCT was issued in favor of Cabautan. In 1977, Gundran issued an action for reconveyance. Who has a better right to the property? Was Cabautan a purchaser in good faith? Since the 2nd sale was registered, the 2nd buyer has a better right to the property. Cabautan was not a purchaser in bad faith merely because of the notice of lis pendens. A purchaser in good faith is one who buys the property without notice that another person has an interest in the property and pays a full and fair price for the same at the time of the purchase or before he has notice of the claim or interest of some other person in the property. The TCT shows no annotation of any sale, lien, encumbrance or adverse claim on the property. When the property is registered under the Torrens system, registration is the operative act to convey or affect the land insofar as 3rrd persons are concerned. A person dealing with registered land is only charged with notice of the burdens on the property which are noted on the register or certificate of title. Even the annotation of lis pendens on the title to the property by 3rd parties does not place the buyer thereof in bad faith since these did not have the effect of establishing a lien or encumbrance on the property affected. Their only purpose was to give notice to 3rrd persons and to the whole world that any interest they might acquire in the property pending litigation would be subject to the result of the suit. ABRIGO vs DE VERA Villafania sold a house & lot in Pangasinan to Tigno-Salazar & Cave-Go. The sale became a subject of suit for annulment of documents between the vendor and the vendees. The RTC rendered judgment approving the Compromise Agreement submitted by the parties & Villafania was given 1 year to buy back the house & lot and failure to do so would mean that the previous sale in favor of Tigno & Cave shall remain valid & binding & Villafania would vacate w/o need of any demand. Villafania failed to buy back the property so Tigno & Cave declared the lot in their name. Unknown to Tigno & Cave, Villafania obtained a free patent over that particular property registered as an OCT w/c was later on cancelled by a TCT, also in her name. Tigno & Cave sold the property to the spouses Abrigo. In the same month, Villafania also sold the same property to de Vera, who registered the sale and a TCT was issued to them. De Vera filed an action for Forcible Entry & Damages against the Abrigos, but pursuant to an agreement between them, the case was dismissed, Instead, the Abrigos filed a case in the RTC for annulment of documents, injunction, restraining order & damagaes against de Vera & Villafania. RTC: For the Abrigos. CA: 1st ruled that Villafania had already transferred ownership to Tigno & Cave, thus the subsequent sale to de Vera was void but upon reconsideration, ruled for De Vera saying that she was in good faith & must be protected. W/N the deed of sale executed by Villafania in favor of de Vera is valid? W/N de Vera is a purchaser in value in good faith? Who between Abrigo & de Vera has a better title over the property? This case involves a double sale under Art1544 of the NCC. A double sale of immovables transfers ownership to (1) the first registrant in good faith; (2) then, the first possessor in good faith; and (3) finally, the buyer who in good faith presents the oldest title. Abrigo registered the sale under Act 3344. On the other hand, de Vera registered under the Torrens system. The Torrens system should prevail over that of Abrigo’s, which is not considered registered. The Abrigos cannot validly argue that they were fraudulently misled into believing that the property was unregistered. A Torrens title, once registered, serves as a notice to the whole world. All persons must take notice, and no one can plead ignorance of the registration. Article 1544 requires the second buyer to acquire the immovable in good faith and to register it in

good faith. Mere registration of title is not enough, good faith must concur. Also, under Sec 44 of PD 1529, every registered owner receiving a certificate of title pursuant to a decree of registration, and every subsequent purchaser of registered land taking such certificate for value and in good faith shall hold the same free from all encumbrances, except those noted and enumerated in the certificate. Thus, a person dealing with registered land is not required to go beyond the certificate of title to determine the condition of the property. Constructive notice to the second buyer through registration under Act 3344 does not apply if the property is registered under the Torrens system, as in this case. De Vera was a purchaser in good faith & for value as Villafania appeared to be the registered owner and she had examined the title in the Registry of Deeds and even went to the premises. When she bought the land, she had no notice the property was already sold nor that the property was under litigation. AFFIRMED. DAGUPAN TRADING VS. MACAM Sale to Macam executed BEFORE the registration of the land, and the conflicting sale was executed AFTER the registration.

– – –

Maron and his 7 siblings were pro-indiviso owners of a parcel of unregistered land in Binmaley, Pangasinan. Sept. 21, 1955: While their application for registration was pending, they executed 2 deeds of sale in favour of Macam. Oct. 14, 1955: One month after, an OCT was issued in the name of the Marons, free from all liens and encumbrances



Aug. 4, 1956: final judgment against Sammy Maron in favour of Mla. Trading and Supply, and levy was made upon whatever interest he had in the property. The notice of levy, certificate of sale and Sheriff’s final cert. of final sale were REGISTERED.



Mar. 1, 1958: Mla Trading sold to Dagupan Trading.

Issue: Who has a better right to the land? Held: Macam

2

The ordinary rules do not apply in the case. Here the sale in favor of appellee was executed before the land subject-matter thereof was registered, while the conflicting sale in favor of appellant was executed after the same property had been registered. The governing law would be last paragraph of Section 35, Rule 39 of the Rules of Court: upon the execution and delivery of the final certificate of sale in favor of the purchaser of land sold in an execution sale, such purchaser "shall be substituted to and acquire all the right, title, interest and claim of the judgment debtor to the property as of the time of the levy." To determine who has a better right, it is necessary to know the interest Sammy Maron had on the 1/8 portion at the time of the levy. Sammy Maron had none, because at the time of the levy, it was already conveyed to Macam. Consequently, subsequent levy made on the property for the purpose of satisfying the judgment rendered against Sammy Maron in favor of the Manila Trading Company was void. Needless to say, the unregistered sale and the consequent conveyance of title and ownership in favor of appellee could not have been cancelled and rendered of no effect upon the subsequent issuance of the Torrens title over the entire parcel of land. In addition, Macam already took poss’n of the land and introduced improvements on the land. CARRUMBA vs. CA 1.

2. 3. 4.

Spouses Amado Canuto (older brother of Benita) and Nemesia Ibasco by virtue of a Deed of Sale of Unregistered Land with Covenants of Warranty sold a parcel of land, partly residential, partly coconut land to spouses Amado Carumba and Benita Canuto. The referred deed of sale was never registered and the notary public was not then authorized. A complaint was filed by Santiago Balbuena against Amado Canuto and Ibasco. CFI rendered decision in favor of the Balbuena and sheriff issued a definite deed of sale to him which was registered. CFI finding after execution of the document Carumba had taken possession of the land, planting fruits and vegetables, declared him to be the owner under a consummated sale and held void the execution of levy made by the sheriff and nullified sale in favor of Balbuena. CA: Double sale. But Balbuena’s title was superior to that of his adversary since he was the first to register and he was in good faith.

HELD: Court disagrees.

2 Conflicting sales of UNREGISTERED LAND  prior sale + public, exclusive and continuous poss’n Conflicting sales of REGISTERED LAND  first to register the transaction

1.

1544 not applicable. Purchaser of unregistered land at sheriff’s execution sale only steps into the shoes of the judgment debtor and merely acquires the latter’s interest in the property sold as of the time the property was levied upon.

2.

While at the time of the levy does not clearly appear, it could have been made prior to April 1957 when the decision against the former owners of the land was rendered in favor of Balbuena. But the deed of sale in favor of Canuto had been executed 2 years before, and while only embodied in a private document, the same, coupled with the fact that Carumba had taken possession of the unregistered land sold, sufficed to vest ownership on Carumba.

3.

When the levy was made by the sheriff, the judgment debtor no longer had dominical interest nor any real right over the land that could pass to the purchaser at the execution sale. Land belongs to Carumba.

4.

CA decision reversed. CFI decision Radiowealth Finance Company v Palileo The Castros (Enrique and Herminia) sold to Palileo a parcel of unregistered coconut land situated in Surigao del Norte. The sale was evidenced by a deed, but was not registered in the Registry of Property. Since the execution of the deed, Palileo, through his mother (as administratrix or overseer), exercised acts of ownership over the land. He also paid the necessary taxes from 1971 onwards. In 1976, Enrique Castro lost a case to Radiowealth and to satisfy the judgment, the same piece of land was levied upon and sold at a public auction. A certificate of sale was executed by the Provincial Sheriff in favor of Radiowealth (the only bidder). A deed of final sale was also issued after the lapse of the period of redemption. Palileo filed an action for quieting of title. The trial court ruled for him. The CA affirmed. Who has a better right to the land? Radiowealth or Palileo? Palileo. It must be stressed that what is being dealt with here is a parcel of unregistered land. Under Act No. 3344, registration of instruments affecting unregistered lands is “without prejudice to a third party with a better right,” meaning that the mere registration of a sale in one’s favor does not give him any right over the land if the vendor was not anymore the owner of the land having previously sold the same to somebody else, even if the first sale was unrecorded. In the previous case of Carumba v CA, it was held that the purchaser of an unregistered land at an execution sale only steps into the shoes of the judgment debtor, and merely acquires the latter’s interest in the property sold as of the time the property was levied upon. Applying this, Radiowealth merely acquired Castro’s interest in the land which, since he had already sold the same to Palileo previously, was no interest at all. Therefore, Palileo has the better right to the land.

SALE BY NON-OWNER PAULMITAN vs. CA (1992) –







Agatona Paulmitan had two children: Pascual (he died and left 7 kids who are respondents in this case) and Donato (1 child, Juliana Fanesa). Agatona died and left two parcels of land (Lot 757 and Lot 1091). Her estate was unsettled. But then Donato extrajudicially adjudicated onto himself Lot 757, and a TCT was issued in his name. Then donato executed a Deed of Sale over Lot 1091 in favor of his daughter Juliana Fanesa. ○ Lot 1091 was forfeited and sold at a public auction for non-payment of taxes. Juliana Fanesa redeemed the property. The respondents (kids of Pascual) filed a case to partition the properties plus damages. RTC: The respondents, as descendants of Agatona, are entitled to ½ of Lot 1091 pro indiviso. The deed of sale to Juliana Fanesa is valid as to the one-half undivided portion of Lot 1091, while the remaining half belongs to the respondents. CA: Affirmed.

Issue: Does Juliana own the entirety of Lot 1091? Held: NO. Only ½ pro-indiviso share. – When Agatona died, she left two kids, Pascual and Donato. Art 777 says: “The rights to the succession are transmitted form the moment of the death of the decedent,” so the right of ownership of Pascual and Donato was vested in them when Agatona died. – But when Donato sold Lot 1091 to his daughter, he could only sell that portion which may be allotted to him upon termination of the co-ownership, because he co-owned the lot with his brother Pascual who died and left 7 kids, so the kids/respondents are the co-owners along with Donato. Donato’s sale to his daughter did not vest ownership in the entire land with his daughter, but



transferred only the seller’s pro indiviso share in the property and consequently made the buyer a co-owner of the land until it is partitioned. Juliana Fanesa claims that the owns the land by virtue of her redeeming it from the government. But the redemption did not terminate the co-ownership nor give her title to the entire land subject of the co-ownership. ○ But she can be reimbursed for half the redemption price she paid on behalf of her co-owners.

MINDANAO vs. YAP (1965) – Rosenda de Nuqui (widow of deceased Sotero Dionisio) and her son Sotero sold three parcels of land in favor of Yap. Included the sale were also buildings on the land as well as lab equipment, books, furniture and other fixtures used by two schools established on the properties (the Mindanao Academy and the Misamis Academy). The price was P100,700. – But Rosenda actually co-owned the land with many of her other children. And the lab equipment, buildings, books, etc were owned by the Mindanao Academy and the Misamis Academy. – Yap took possession of the properties and even changed the names of the schools to Harvardian Colleges. – The children who did not take part in the deed of sale filed two cases: one was for annulment of the sale and the other for rescission. Issue: What is the status of the sale? Held: NULL AND VOID. – The contract purported to sell properties of which the sellers were not the only owners. Plus the prestation involved in the sale was indivisible and therefore incapable of partial annulment, inasmuch as the buyer Yap, by his own admission, would not have entered into the transaction except to acquire all of the properties purchased by him. So there is no necessity to rule on the question of rescission since the contract itself is null and void. – The buyer and seller were both in bad faith. The return of the properties by the buyer is a necessary consequence of the decree of annulment. No part of the purchase price having been paid, as far as the record shows, there is no need for restitution (just return of the properties). Note: See Villanueva’s explanation in the book (his opinion is that the sale is NOT null and void because a seller may validly enter into a valid and binding contract of sale on properties which he entirely does not own; there is a diff between the perfect stage and the consummation stage). ESTOQUE vs. PAJIMULA (1968) BUCTON vs. GABAR (1974) SUN BROS. & CO. vs. VELASCO (1958) TAGATAC vs. JIMENEZ (1957)





Tagatac bought a car from Danielson and Kavarno Moters in California. Tagatac brought it to the Phils. One of her friends, Lee, introduced Warner Feist (aka Warner Levy) to her. Feist was posing as a very wealthy man; he let Tagatac believe that he was a millionaire, the manager of a corporation, the owner of two houses and cars, etc. He offered to buy the car for P15k, and she agreed. They signed a deed of sale and Feist paid the price to Tagatac thru a postdated check. Tagatac then tried to cash the check with PNB, but the bank refused to honor it and told her that Feist had no account and no funds with the bank. The car disappeared, and so did Feist. Feist managed to have the deed of sale notarized and the registration certificate of the car transferred in his name. He sold it to Sanchez, and (registration was transferred in his name). Sanchez sold the car to Jimenez (registration was transferred in his name). Jimenez then delivered the car to a car exchange on Taft to display it for sale, and transferred it to two people in order to facilitate the sale of the car, but they were unable to.



Tagatac found the car at the car exchange and filed a suit for the recovery of the possession of the car. She claims that Jimenez is a purchaser in bad faith. She also claims that acc to the ROC, there is a disputable presumption that a person found in possession of a thing taken in the doing of a recent wrongful act is the taker and the doer of the whole act. She also claims that Art 559 of the Civil Code applies here.

Issue: Who has valid title over the car? Held: Jimenez. – The ROC presumption does not apply because the car was not stolen and Jimenez came into possession of it 2 months after Feist swindled Tagatac. – Jimenez is NOT a purchaser in bad faith. ○ When he acquired the car, he had no knowledge of any flaw in the title. – Art 559 does NOT apply. Art 559 says that although possession of movable property acquired in good faith is equivalent to a title, one who has lost any movable or has been unlawfully deprived thereof, may nevertheless recover it form the person in possession of the same. ○ It only seems that Tagatac was unlawfully deprived. But this case doesn’t fall under Art 559. There was valid transmission of ownership from Tagatac to Feist, considering that there was a contract of sale between them. Remember, failure to pay the price or the issuance of a check for the price w/o funds does not affect the validity of the transfer. – The contract between Tagatac and Feist, because of the fraud and deceit, is voidable. Being voidable, it is susceptible of either ratification or annulment. If ratified: action to annul it is extinguished. If annulled: parties are restored to their situations before the contract and there is mutual restitution. – However, as long as there is no action taken by the party entitled (he picks neither annulment nor ratification), the contract of sale remains valid and binding. The title that Feist acquired was defective and voidable, but at the time he sold the car to Sanchez his title had not been avoided and Sanchez therefore conferred a good title on the latter, provided Sanchez bought the car in good faith, for value, and w/o notice of the defect of Feist’s title. There is no proof that Sanchez acted in bad faith, so it’s safe to assume he acted in good faith. Jimenez thus bought the car in good faith, for value, and w/o notice of any defect in Sanchez’s title, hence he acquired a good title to the car. Good title means an indefeasible title to the car as against the original owner. – Even under the rules of equity, Jimenez has the better title. Both Tagatac and Jimenez are innocent parties. But it was through Tagatac’s gullibility that Feist swindled her. EDCA PUBLISHING vs. SANTOS (1990)



– –



a person identifying himself as Prof. Jose Cruz placed an order by telephone w/ EDA for 406 books, payable on delivery. The invoice was prepared and the books were delivered, for which Cruz issued a personal check drawn against Phil. Amanah Bank amounting to 8,995. Cruz then sold 120 books to Santos, who after verifying the seller’s ownership from the invoice he showed her, paid him 1,700. EDA became suspicious and made inquiries w/ De la Salle and found out that he had no more account w/ the bank. A trap was set and it was discovered that he was really Tomas de la Pena. EDCA, together w/ the police, then forced their way into Santos’ store and threatened Santos w/ prosecution for buying stolen property then seized the books w/o warrant.

Issues:

1. – –

Who is the lawful owner of the books? Santos Art. 559: “the possession of a movable property in good faith is equivalent to a title.” EDCA’s contentions: ○ that Santos has not established their ownership bec. they have not produced a receipt  not required, as the provision itself dispenses w/ further proof. ○ That Santos was in bad faith  Santos was in good faith. Santos first ascertained the ownership from the EDCA invoice showing that they have been sold to Cruz,

who said he was selling them for a discount for he was in financial need. It is hardly bad faith for any one in the business of buying and selling books to buy them at a discount and resell them.

1. – –



Whether EDCA was unlawfully deprived of the books because the check in payment therefore was dishonored?  NO EDCA: that because the impostor acquired no title to the books because of the failure of consideration that nullified the contract of sale, he could not have validly transferred them to Santos. Non-pmt. Only creates a right to demand pmt. Or to rescind the contract or to criminal prosecution, in the case of bouncing checks. But absent any stipulation, delivery will effectively transfer ownership to the buyer who can in turn transfer it to another. Actual delivery having been made, Cruz acquired ownership over the books which he could then validly transfer to Santos. The fact that he had not yet paid for them to EDCA was a matter b/w him and EDCA and did not impair the title acquired by Santos to the books.

CRUZ vs. PAHATI (1956) 1.

2. 3. 4. 5.

The automobile in question was originally owned by Northern Motors which later sold it to Chinaman Lu DAg. Lu Dag  Jesusito Belizo  Jose Cruz Belizo is a dealer of 2nd hand cars. Belizo offered Cruz to sell the automobile for him claiming to have a buyer for it. Cruz agreed. Cruz’ certificate of registration was missing, therefore, Cruz wrote to Bureau of Public Works. This letter was delivered to Belizo and the automobile was turned over to him on Belizo’s pretext that he was going to show it to a prospective buyer. The letter was falsified and controverted into an authorized deed of sale in favor of Belizo by erasing a portion. After some time, Belizo sold the car to Pahati. This shows then a conflict of rights of 2 persons who claims to be the owner of the property

Issue: Who is the owner of the automobile? Held: 1.

Law applicable: Art. 559 of CC a. The possession of movable property acquired in good faith is equivalent to a title. Nevertheless, one who has lost any movable or has been unlawfully deprived thereof, may recover it, from the person in possession of the same. b. If the possessor of a movable lost or of which the owner has been unlawfully deprived, has acquired in good faith at a public sale, the owner cannot obtain its return without reimbursing the price paid therefor 2. Art. 1505 a. “where goods are sold by a person who is not the owner, thereof, and who does not sell them under authority or with the consent of the owner, the buyer acquires no better title to the goods than the seller had, unless the owner of the goods is by his conduct precluded from denying the seller’s authority” 3. Cruz has a better right in this case. 4. Where one of 2 innocent parties must suffer by a fraud perpetrated by another, the law imposes the loss upon the party who, by his misplaced confidence, has enabled the fraud to be committed 5. in this case, Bulahan should bear the loss because of the confidence he reposed in Belizo which enabled the latter to commit the falsification Judgment of LC: reversed Suntay owns a 3-carat diamond ring valued at 5.5K, w/c she delivered to Sison for sale on commission. Sison executed & delivered a receipt. Sison didn’t return the ring so Suntay made demands but she couldn’t comply cos the ring was already pledged by Sison’s niece, Melia, to Dizon’s pawnshop for 2.6K. Sison delivered to Suntay the pawnshop ticket. Suntay filed a case for estafa gainst Sison. Suntay wrote a letter to Dizon asking for the delivery of the ring, to w/c Dizon refused so Suntay filed the present action w/ the provisional remedy of replevin. LC: executed writ of replevin & ruled for Suntay. CA: Affirmed.

W/N the owner of a movable unlawfully pledged by another can recover? YES Art 559 CC:The possession of movable property acquired in good faith is equivalent to title but one who has lost any movable or has been unlawfully deprived may recover it from the person in possession of the same. The only exception that the law allows is when there is acquisition in good faith of the possessor at a public sale in w/c case the owner cannot obtain its return w/o compensation Dizon asserts that Suntay is in estoppel. This cannot hold. He is engaged in a business requiring prudence to ascertain w/n an individual offering in pledge is authorized to do so. He has only himself to blame. Teehankee, concurring: The phrase “unlawfully deprived” in Art559 is used in a general sense & not specifically as in deprivation by robbery or theft. It extends to all cases where there has been no valid transmission of ownership, including a depositary or lessee who has sold the same. The owner in such case is undoubtedly unlawfully deprived of his property & may recover from a possessor in good faith. The contention that the owner may recover only when there is a conviction of the “embezzler from the pawnshop” cannot stand. This adds a requirement not in the article & unduly prejudices the victim.

Roman vs. Grimalt (1906) FACTS:  Roman filed a complaint against Grimalt for the payment of the purchase price of a certain schooner (a vessel).  Roman alleged that he and Grimalt, through Pastor, verbally agreed for the sale of a schooner for the amount of P1500 to be paid on installments.  However, when the schooner was about to be delivered, it was destroyed due to a severe storm.  A couple of days later, Roman was demanding the purchase price of the said schooner.  Grimalt argued that accepted the offer of sale on condition that the title papers were found to be satisfactory, also that the vessel was in a seaworthy condition.  However, upon examination of the documents by a notary public, it was found out that it was insufficient to show ownership and to transfer title.  Grimalt further argued that he accepted the offer because Roman promised that he would perfect the documents (to show that he was the owner). ISSUE: I. W/N there was a perfected sale. II. W/N Grimalt should shoulder the loss of the schooner. HELD: I. NO PERFECTED SALE; ROMAN TO SHOULDER THE LOSS 1. The sale of the schooner was not perfected and the Grimalt did not consent to the execution of the deed of transfer for the reason that the title of the vessel was in the name of one Paulina Giron and not in the name of Pedro Roman, the alleged owner. 2. Roman promised, however, to perfect his title to the vessel, but he failed to do so. The papers presented by him did not show that he was the owner of the vessel. 3. If no contract of sale was actually executed by the parties the loss of the vessel must be borne by its owner and not by a party who only intended to purchase it and who was unable to do so on account of failure on the part of the owner to show proper title to the vessel and thus enable them to draw up the contract of sale.

4. The defendant was under no obligation to pay the price of the vessel, the purchase of which had not been concluded. 5. It follows, therefore, that article 1452 of the Civil Code relative to the injury or benefit of the thing sold after a contract has been perfected and articles 1096 and 1182 of the same code relative to the obligation to deliver a specified thing and the extinction of such obligation when the thing is either lost or destroyed, are not applicable to the case at bar. 6. The first paragraph of article 1460 of the Civil Code and section 335 of the Code of Civil Procedure are not applicable. These provisions contemplate the existence of a perfected contract which can not, however, be enforced on account of the entire loss of the thing or made the basis of an action in court through failure to conform to the requisites provided by law.

Lawyer’s Cooperative Publishing Co. vs Tabora (1965)









Tabora bought from the Lawyer’s Cooperative Publishing Co. one complete set of American Jurisprudence plus one set of AmJur Index for the total price of P1,675. Tabora made a partial payment of P300 (left a balance of P1,300). The books were delivered to his law office. The contract stated: “Title to and ownership of the books shall remain with the seller until the purchase price shall have been full paid. Loss or damage to the books after delivery to the buyer shall be borne by the buyer. A fire broke out at midnight of the day the books were delivered, and they were destroyed. Tabora informed the company and they even gave him volumes of the Phil Reports as goodwill. But since Tabora failed to pay the balance, Lawyer’s Coop filed the present action against him. Tabora claims: The books were lost through force majeure and he cannot be held responsible for the loss. He claims that since it was agreed that the ownership of the books shall remain with the seller until after the payment of the purchase price, it is the seller who should bear the loss.

Issue: Who shall bear the loss? Held: Tabora. – The contract actually states that “Loss or damage to the books after delivery to the buyer shall be borne by the buyer.” – Art 1504 actually sanctions such stipulation: “Where delivery of the goods has been made to the buyer or to a bailee for the buyer, in pursuance of the contract and the ownership in the goods has been retained by the seller merely to secure performance by the buyer of his obligations under the contract, the goods are at the buyer’s risk form the time of such delivery.” – Tabora cannot claim that since the books were destroyed by fire w/o any fault on his part he should be relieved from the resultant obligation, under the rule that an obligor should be held exempt from liability when the loss occurs thru a fortuitous event. This is because the rule only holds true when the oblig consists in the delivery of a determinate thing and there is no stipulation holding him liable even in case of fortuitous event. That doesn’t apply in the present case because the obligor bound himself to assume the loss after the delivery of the goods to him; there is stipulation in the contract.

REMEDIES IN CASE OF BREACH VISAYAS SAWMILL COMPANY V. CA, RJH TRADING –







– –

Visayas Sawmill entered into a Purchase and Sale of Scrap Iron Agreement w/ RJH Trading, where it undertook the obligation to sell such scrap iron. RJH was obligated to open a n irrevocable and unconditional LC not later than May 15, 1983 at the Consolidated Bank and Trust Company in the sum of 250K. VSC, in turn, will furnish RJH free of charge cargo trucks to haul the weighed materials from Cawitan to the TSMC wharf. VSC then allowed RjH’s men to dig and gather scrap iron within its premises. But later on, VSC alledgedly ordered RJH to desist in view of the case filed against the latter by one Pursuelo. RJH, in turn, contends that VSC sent a telegram canceling the contract of sale because of failure of the latter to comply w/ the conditions thereof. A LC was then opened on may1 w/ BPI Ayala, and on May 26, VSC received a letter advice from BPI Dumaguete, for the account of Armaco-marsteel Allow, which was to expire on July 24, 1983 without recourse. RJH then sent telegrams to VSC stating that the case was dismissed and demanding compliance w/ the deed of sale. TC and CA: cancellation was not in order. The contract was perfected, and delivery has already been made when VSC allowed the digging within its premises, with RJH having control and possession. Automatic rescission, tehn, cannot take place because delivery had already been made. In addition, the 11-day delay in the opening of the LC cannot be considered a substantial breach of the agreement.

Can VSC rescind the sale?  YES – Art. 1593: “With respect to movable property, the rescission of the sale shall of right take place in the interest of the vendor, if the vendee, upon the expiration fo the period fixed for the delivery of the thing, should not have appeared to receive it, or having appeared, he should not have tendered the price at the same time, UNLESS a longer period has been stipulated for its payment.” – What obtains in the case is a MERE CONTRACT TO SELL, not a contract of sale. The obligation to sell is subject to a POSITIVE SUSPENSIVE CONDITION, the opening of the irrevocable LC. Thus, there was to be no actual sale until the opening of such LC. And the failure of RJH to comply w/ the positive suspensive condition cannot even be considered a breach but simply an event that prevented the obligation of vSC to convey title from acquiring binding force. ○ The LC was not in accordance w/ the stipulation in the contract on 3 counts:  It was not opened by RJH, but by a corp. which is not a party to the contract  Not opened w/ the bank agreed upon  Not irrevocable and unconditional, for it is w/o recourse and to expire on a specific date. – Consequently, the oblig. of VSC did not arise and cannot be compelled by specific performance. On the contrary, VSC may totally rescind the contract based on Art. 1597: “Where the goods have nto been delivered to the buyer, and the buyer has repudiated the contract of sale, or has manifested his inability to perform his obligations, thereunder, or has committed a breach thereof, the seller may totally rescind the contract of sale by giving notice of his election so to do to the buyer. – On the issue of delivery, such was not made and the consent to dig did not amount to control and possession of the goods. ROMERO DISSENT: – the contract was a contract of sale, with all the essential requisites present in the case at bar. Non-payment is immaterial to the transfer of the right of ownership, unless ownership has been reserved by the seller until full payment has been made, which was not done here. The mere insertion of the price and mode of pmt. will not necessarily make it a contract to sell.

– –

The breach was not substantial, as in other cases, 20 days was not ruled to be a ground for rescission. S=None of the defects in the letter of credit would serve to defeat the object of payment. Delivery has been made, as decided by the LCs.

CRUZ V. FILINVEST INVESTMENT & FINANCE CORP. –

– –

– – – –

Cruz purchased on installments one Isuzu diesel bus from Far East Motor Corp. for 44K. Cruz executed a PN and a chattel mortgage on the said vehicle. As there was no downpayment, Cruz had to give an additional security and Reyes executed an additional real estate mortgage on her land and building. At that time, said land and building were mortgaged to the DBP for loans obtained by Reyes. Far East then indorsed the PN to Filinvest Investment and assigned all rights to the chattel and real estate mortgage, with due notice of the assignment to the plaintiffs. Cruz defaulted, paying only 500. Filinvest took steps to foreclose the chattel mortgage, and emerged as the highest bidder at 15K. This, however, was not sufficient to cover the expenses of the sale, the principal oblig., interests and attorney’s fees amounting to 43K. Filinvest, preparatory to foreclosing the REM in order to collect the deficiency, paid Reyes’ indebtedness to DBP. It then asked the Sheriff to take poss’n of the land. Reyes then sought the cancellation of the REM, which was not granted by FIlinvest, on the ground that it was w/o legal basis. The Sheriff then held in abeyance the sale of the land.

Issues:

1.

Whether defendant, which has already extrajudicially foreclosed the chattel mortgage executed by the buyer on the bus sold to him on installments, may also extrajudicially foreclose the real estate mortgage constituted by Mrs. Reyes on her own land, as additional security, for the payment of the balance of Cruz' Obligation, still remaining unpaid ?  NO

- ART. 1484. In a contract of sale of personal property the price of which is payable in installments, the vendor may exercise any of the following remedies: (1) Exact fulfillment of the obligation, should the vendee fail to pay; (2) Cancel the sale, should the vendee's failure to pay cover two or more installments; (3) Foreclose the chattel mortgage on the thing sold, if one has been constituted, should the vendee's failure to pay cover two or more installments. In this case, he shall have no further action against the purchaser to recover any unpaid balance of the price. Any agreement to the contrary shall be void.

– –





the established rule is that the foreclosure and actual sale of a mortgaged chattel bars further recovery by the vendor of any balance on the purchaser's outstanding obligation not so satisfied by the sale. Cruz’ contention: that what is being withheld from the vendor, by the proviso of Article 1484 of the Civil Code, is only the right to recover "against the purchaser", and not a recourse to the additional security put up, not by the purchaser himself, but by a third person To sustain appellant's argument is to overlook the fact that if the guarantor should be compelled to pay the balance of the purchase price, the guarantor will in turn be entitled to recover what she has paid from the debtor vendee (Art. 2066, Civil Code) ; so that ultimately, it will be the vendee who will be made to bear the payment of the balance of the price, despite the earlier foreclosure of the chattel mortgage given by him. Thus, the protection given by Article 1484 would be indirectly subverted, and public policy overturned. An extrajudicial foreclosure of a REM is contemplated in the said provision, contrary to Cruz’ argument. It is a proceeding by virtue of which one is enabled to lawfully recover the unsatisfied balance.

Foreclosure of the REM is barred by virtue of the prior foreclosure of the chattel mortgage. However, to the extent that she was benefited by such payment, Reyes should have been required to reimburse the Filinvest.

Ridad vs Filipinas Investment and Finance Corp. (1983) – – – – – – – – –

Ridad purchased from Supreme Sales and Development Corporation 2 Ford cars (for taxicab franchise). The parties agreed that the cars are to be paid in 24 monthly installments. To secure payment, Ridad issued a PN and a chattel mortgage over the 2 Ford cars plus a Chevrolet car. Later, Supreme Sales assigned its rights over the PN and CM to Filipinas investment. Ridad failed to pay, so Filipias investment foreclosed the CM. Filipinas investment was the highest bidder, and subsequently sold and conveyed the same to co-defendant Sebastian. Ridad filed an action for annulment of the contract bet. Filipinas and Sebastion. RTC: CM is null and void in so far as the taxicab franchise and the used Chevrolet car; sale at public auction concerning the taxicab franchise to be of no legal effect. CA: affirmed.

Issue: W/N the CM is valid in so far as the franchise and the subsequent sale thereof. Held: CM NULL AND VOID. 1. Under Art.1484 of the NCC, the vendor of personal property the purchase price of which is payable in installments, has the right, should the vendee default in the payment of two or more of the agreed installments, to exact fulfillment by the purchaser of the obligation, or to cancel the sale, or to foreclose the mortgage on the purchased personal property, if one was constituted. 2. Whichever the right the vendor elects, he cannot avail of the other, these remedies being alternative, not cumulative. 3. If the vendor avails himself of the right to foreclose his mortgage, the law prohibits him from further bringing an action against the vendee for the purpose of recovering whatever balance of the debt secured not satisfied by the foreclosure sale. 4. Filipinas Investment elected to foreclose its mortgage upon default by the plaintiffs in the payment of the agreed installments. Having chosen to foreclose the CM, and bought the purchased vehicles at the public auction as the highest bidder, it submitted itself to the consequences of the law as specifically mentioned, by which it is deemed to have renounced any and all rights which it might otherwise have under the promissory note and the CM as well as the payment of the unpaid balance.

Zayas vs Luneta Motor Company –

– –





Zayas purchased on installment a motor vehicle (Ford Thames Freighter) from Escaño Enterprises, dealer of Luneta Motor Company. The price was payable in 24 months at 12% interest per annum. The vehicle was delivered to Zayas, who paid the initial payment of P1k and executed PN’s in the amount of P7.9k for the balance. The PN stated the amounts and dates of payment of 26 installments. He also executed a chattel mortgage on the vehicle in the favor of Luneta. After he paid P3k, he was unable to pay the rest of the price, so Luneta extrajudicially foreclosed the chattel mortgage. The vehicle was sold at public auction with Luneta as highest bidder in the amount of P5k. Since the payments made by Zayas plus the P5k from the foreclosure wasn’t enough to cover the total of the PN, Luneta filed a civil case for the recovery of the balance. Zayas claims that the outstanding obligation had been discharged by sale in public auction of the motor vehicle, pursuant to Art 1484. ○ RTC and CA: Luneta’s complaint dismissed. ○ Luneta’s main allegation is that it was Escaño from which Zayas purchased the vehicle, and Escaño is a different entity from Luneta.

Issue: Was the obligation discharged, according to Art 1484? Held: YES. Art 1484 says, “In a contract of sale of personal property the price of which is payable in installments, the vendor may exercise any of the ff remedies: (3) Foreclose the chattel mortgage on the thing sold, if one has been constituted, should the vendee’s failure to pay cover two or more installments. In this case, he shall have no further action against the purchaser to recover any unpaid balance of the price. Any agreement to the contrary shall be void.” The established rule is that the foreclosure and actual sale of a mortgaged chattel bars further recovery by the vendor of any balance on the purchaser’s outstanding obligation not so satisfied by the sale. Issue: Was Escaño a distinct and independent entity? Held: NO. Escaño was a collecting agent as far as the purchase of the motor vehicle is concerned. The principal and agent relationship is clear. Even assuming that Escaño is a distinct and independent entity, the nature of the transaction as a sale of personal property on installment basis remains. Escaño actually assigned its rights vis-à-vis the sale to Luneta, and the nature of the transaction did not change at all. As assignee, Luneta had no better rights than Escaño under the same transaction. The transaction would still fall under Art 1484.

NONATO vs IAC Spouses Nonanto purchased a Volkwagen Sakbayan from People’s Car, Inc. on installment basis. To secure payment, Investor’s Finance Corp (respondents) executed a promissory note & chattel mortgage. People’s Car assigned its rights & interests over the note & mortgage to Investor’s. For failure of the Nonatos to pay 2/more installments despite demands, the car was repossessed. Despite reposession, Investor’s demanded from the Nonatos payment of the balance of the price. Finally, Investor’s filed a complaint for payment of the balance. The Nonatos defense is that when the company repossessed the car, it had effectively cancelled the sale & thus barred from exacting recovery of the unpaid balance. TC: Nonatos to pay Investor’s. CA: Affirmed. W/N a vendor or his assignee who had cancelled the sale of a motor vehicle for failure of the buyer to pay 2 or more of the stipulated installments may also demand payment of the balance of the purchase price. Art.1484  Sale of personal property in installments. Should the vendor/purchaser default in the payment of 2/more of the agreed installments, the vendor/seller has the option to avail of any of the 3 remedied: either to exact fulfillment by the purchaser OR to cancel the sale OR to foreclose the mortgage on the purchased personal property if one was constituted. These are alternative remedies not cumulative. The exercise of one would bar the exercise of the others. Accdg to the company the repossession was only for the purpose of appraising the car’s value & for storage and safekeeping pending full payment of the purchase price. The receipt issued by the company when it took possession states that the vehicle would be redeemed w/in 15 days. This could only mean that should Nonatos fail to redeem w/in the said period, the company would retain permanent possession of the vehicle, w/c it did in fact. Furthermore, even after the company had notified the Nonatos that the value of the car was not sufficient to cover the balance of the purchase price, there was no attempt at all on the part of the company to return the repossessed car. The acts performed were consistent with the conclusion that it had opted to cancel the contract of sale of the car. DISMISSED.

RIDAD vs FILIPINAS INVESTMENT & FINANCE CORP.

Ridad spouses purchased from Supreme Sales & Devpt Corp 2 brand new Ford Consul Sedans w/ accessories for 26K payable in 24 monthly installments. To secure payments, Ridad executed a promissory note & a deed of chattel mortgage on the 2 vehicles & another vehicle (a Chevrolet) & they’re franchise/certificate of public convenience for the operation of a taxi fleet. With Ridad’s consent, Supreme Sales assigned its rights, title & interest to Filipinas Investment. Due to the failure of Ridad to pay monthly installments, Filipinas foreclosed the chattel mortgage extrajudicially & at public auction of the 2 Ford Consuls, of w/c Ridad was not notified, Filipinas was the highest bidder & purchaser. Another auction sale was held involving the other properties and at the public auction, the taxi franchise was sold to Filinvest who later on sold it to Sebastian who then filed w/ the Public Service Commission an application for the approval of the sale in his favor. Ridad filed an action for annulment of the contract. LC: Chattel mortgage null & void as so far as the Chevrolet & taxi franchise is concerned. CA: Certified appeal to the SC. W/N the chattel mortgage is valid in so far as the franchise & its subsequent sale to Sebastian was concerned. Art.1484  Sale of personal property in installments. Should the vendor/purchaser default in the payment of 2/more of the agreed installments, the vendor/seller has the option to avail of any of the 3 remedied: either to exact fulfillment by the purchaser OR to cancel the sale OR to foreclose the mortgage on the purchased personal property if one was constituted. These are alternative remedies not cumulative. The exercise of one would bar the exercise of the others. The purpose of the law is to prevent mortgagees from seizing the mortgaged property, buying it at foreclosure sale for a low price then bringing suit against the mortgagor for a deficiency judgment, otherwise the mortgage would find himself w/o the property & still owing practically the full amount of the original indebtedness. Filipinas elected to foreclose its mortgage upon default in the payment of installments. Thus, it has renounced any and all rights w/c it might otherwise have under the promissory note & the chattel mortgage as well as the payment of the unpaid balance. As for the taxi franchise, it was ruled that should the vendor choose to foreclose the mortgage, he has to content himself w/ the proceeds of the sale at the public auction of the chattels w/c were sold on installment and mortgaged to him and cannot insist on the sale of the other properties as it would be equivalent to obtaining a writ of execution against the buyer concerning other properties w/c are separate and distinct from those w/c were sold on installment. This would be contrary to public police & the purpose of the law limiting the vendor’s right to foreclose the chattel mortgage only on the thing sold. AFFIRMED

Filinvest Credit Corporation vs CA (1989) FACTS:  Spouses Jose Sy Bang and Iluminada Tan (private respondents) were engaged in the sale of gravel produced from crushed rocks and used for construction purposes.

     





     

To increase their production, they engaged the services of Mercurio, the proprietor of Gemini Motor Sales, to look for a rock crusher which they could buy. Mercurio referred the spouses to the Rizal Consolidated Corporation which had with them such machinery. Oscar Sy Bang, a brother of PR, inspected the machine at the Rizal Consolidated's plant site. Satisfied with the machinery, PR signified their intent to purchase it. However, the machinery amounted to P550k The PR applied for financial assistance from the petitioner, Filinvest Credit Corporation. The petitioner agreed to extend to the PR financial aid on the following conditions: 1. that the machinery be purchased in the petitioner's name; 2. that it be leased (with option to purchase upon the termination of the lease period); 3. that the PR execute a real estate mortgage in favor of the petitioner as security for the amount advanced by the latter. A contract of lease of machinery (with option to purchase) was entered into by the parties. And to guarantee their compliance with the lease contract, the PR executed a real estate mortgage over two parcels of land in favor of the petitioner. PR, claiming that they had only tested the machine for a month, sent a letter-complaint to the petitioner, alleging that contrary to the 20 to 40 tons per hour capacity of the machine as stated in the lease contract, the machine could only process 5 tons of rocks and stones per hour. Based on such claim, PR stopped payment on the remaining checks they had issued to the petitioner. As a consequence of the non-payment by the PR, petitioner extrajudicially foreclosed the real estate mortgage. PR filed a complaint to enjoin the foreclosure, rescission of the contract of lease with option to purchase, and annulment of the real estate mortgage RTC: ruled in favor of PR CA: affirmed decision Filinvest’s argument: it merely financed the purchase and therefore any defect on the machinery should be addressed to Rizal Consolidated, the seller.

ISSUE: I. W/N the contract entered into bet. PR and Filinvest is a contract of lease or a sale. HELD: I. SALE (FILINVEST IS THE OWNER OF THE MACHINERY) 1. The nomenclature of the agreement cannot change its true essence, i.e., a sale on installments. 2. Upon completion of the payments, then the rock crusher, subject matter of the contract, would become the property of the PR. 3. This form of agreement has been criticized as a lease only in name. 4. The Court held that the contract of lease with option to buy is at times resorted to as a means to circumvent Art. 1484, particularly par. 3. 5. Through the set-up, the vendor, by retaining ownership over the property in the guise of being the lessor, retains, likewise, the right to repossess the same, without going through the process of foreclosure. 6. There arises therefore no need to constitute a chattel mortgage over the movable sold. 7. More importantly, the vendor, after repossessing the property and, in effect, cancelling the contract of sale, gets to keep all the installments-rentals already paid. Note: The Court ruled that the PR have defaulted on their contract with the finance company, and therefore dismissed the complaint of the PR.

DELTA MOTOR SALES vs. NIU KIM DUAN (1992) 1.

NKD purchased from Delta 3 units of Daikin air-conditioner worth 19,350. A deed of conditional sale was issued which states that: a. NKD would pay a downpayment of 774 and the balance of 18k+ shall be paid in 24 monthly installments b. title to properties would remain with Delta until the price is fully paid

c. if any 2 installments are not paid by NKD on their due dates, the the whole of the principal sum remaining unpaid would become due 2.

To secure the payment of the balance, NKD jointly and severally executed in favor of Delta a promissory note.

3.

The airconditioners were delivered and received. But after paying almost 7k, NKD failed to pay at least monthly installments. Delta tried to recover extrajudicially, but they failed to do so. A writ of replevin then was issued by the court and the payment of almost 7k was treated as rentals of the aircons.

Issue: Can Delta treat the payments as rentals? What are the remedies of Delta? Held: 1.

Yes. A stipulation in the contract that the installments paid shall not be returned to the vendee is valid as it is not unconscionable and is under Art. 1486. The amount NKD has paid only corresponds to 7 months. Since they admit using it already for 22 months, this means that they did not pay for 15 months thus used the aircons for free to the prejudice of Delta.

2.

The vendor in a sale of personal property payable in installments may exercise 1 of 3 remedies: a) exact the fulfillment of the obligation, should the vendee fail to pay b) cancel the sale upon the vendee's failure to pay 2 or more installments c) foreclose the chattel mortgage, if one has been constituted on the property sold, upon the vendee's failure to pay 2 or more installments * C option is subject to the limitation that the vendor cannot recover any unpaid balance of the price and any agreement to the contrary is void

The 3 remedies are alternative and not cumulative. In the case at bar, Delta filed a case to seek a judicial declaration that it had validly rescinded the deed of conditional sale (through the writ of replevin). Delta chose alternative B. Having done so, Delta is barred from exacting payment from NKD of the balance of the price. It cannot have its cake and eat it too. LC decision set aside.

BORBON II vs. SERVICEWIDE SPECIALISTS (1996) 1.

Daniel Borbon and Francisco Borbon signed a promissory note for the purchase of an Isuzu truck worth 122,856 from Pangasinan Auto, payable in 12 monthly installments.

2.

Panagasinan Auto assigned its rights to Filinvest Credit Corp. Filinvest assigned its rights over the PN and the chattel mortgage to Servicewide Specialists.

3.

Borbons failed to pay. Demand letter were sent but still no payment was made. Serviceiwde sued Borbons then. The defense of Borbons was that the vehicle delivered was an Isuzu crew cab instead of the truck.

4.

LC and CA: Servicewide Issue: Borbons merely seek a modification of the decision to delete the awards of liquidated damages and attorney's fees. Is this possible? Held: Yes 1.

When the assignee forecloses on the mortgage, there can be no further recovery of the deficiency, and the seller-mortgagee is deemed to have renounced any right thereto.

2.

Under 1484, the vendor-mortgagee or its assignees loses any right to “recover any unpaid balance of the price” and any “agreement to the contrary would be void”.

Any unpaid balance can only mean the deficiency judgment to which the mortgagee may be entitled to when the proceeds from the auction sale are insufficient to cover the full amount of the secured obligation which include interest on the principal, attorney's fees, expenses of collection, and the costs. Given the circumstances, the court strikes down the award for liquidated damages but uphold the grant of attorney's fees 3.

The remedies in 1484 are not cumulative but alternative and exclusive.

4.

Alternative obligations is a mere choice categorically and unequivocally made and then communicated by the person entitled to exercise the option concludes the parties. The creditor may not thereafter exercise any other option, unless the chosen alternative proves to be ineffectual or unavailing due to no fault on his part.

5.

Alternative remedies on the other hand is wherein the choice generally becomes conclusive only upon the exercise of the remedy.

FILIPINAS INVESTMENT & FINANCE V. RIDAD

– – –

– –

spouses Ridad bought from Supreme Sales & Dev’t Corp. a Ford Consul sedan for 13K in 24 equal monthly installments. Ridad executed a PN and a chattel mortgage on the car. For failure to pay 5 consecutive instalmments, Fil. Instituted a replevin suit for the seizure of the car OR the recovery of the unpaid balance in case delivery could not be effected. The car was seized by the Sheriff and Fil. Instituted extrajudicial foreclosure proceedings, the result of which was the sale of the car at the public auction to Fil. Inv. The Ridads were declared in default and were ordered to pay 500 as attorney’s fees and expenses relative to the seizure of the car. This order was then contested by Ridad: ○ Ridad: action was for pmt of unpaid balance of the purchase price w/ prayer for replevin. When the car was seized and foreclosed, it thereby renounced any right under the PN and the attorney’s fees and costs of suir. ○ Fil: the costs were incurred by viretue of the unjustificable failure and refusal oof the Ridads to comply w/ their oblig. What was prohibited under 1484 is the recovery of the unpaid balance of the purchase price by means of action OTHER THAN a suit for replevin. Plus, such was already stipulated in the PN and chattel mortgage contract.

Issue: w/n Fil. Investment is entitled to attorney’s fees and other expenses? Held: – 1484 applies. While the present action is one for replevin, it culminated in the foreclosure of the chattel mortgage and the sale at public auction. – Macondray: “any unpaid balance” refers to the deficiency judgment to which the mortgagee may be entitled to when the proceeds from the auction sale are insufficient to cover the full amount of the secured obligation which include interest on the principal,







attorney's fees, expenses of collection, and the costs. Were it the intention of the Leg. To limit the meaning tot eh unpaid balance of the principal, it would have so stated. In all proceedings for the foreclosure of a chattel mortgage, executed on chattels which have been sold on the installment plan, the mortgagee is limited to the property mortgaged, and is not entitled to attorney’s fees and costs of suit. Such is to prevent the vendor from circumventing the Recto law. EXC: mortgagee is not entitled to protection against PERVERSE MORTGAGORS. Where the mortgagor plainly refuses to deliver the chattel subject of the mortgage upon his failure to pay 2 or more installments or if he conceals the chattel to place it beyond the reach of the mortggagee, the mortgagee has not choice but to file an action for replevin to the end that he may recover immediate poss’n of the chattel and thereafter, enforce his rights in accordance w/ the contractual rel. b/w him and the mortgagor. It logically follows that the necessary expenses incurred in the prosecution should be borne by the mortgagor. Recoverable expenses would included expenses incurred in effecting seizure of the chattel and attorney’s fees.

TANJANLANGIT V. SOUTHERN MOTORS – –

– –



Tanjanlangit bought 2 tractors and a thresher from Southern Motors for 24K payable in installments, w/ an acceleration clause. They executed a PN in SM’s favor. Tajanlangit failed to meet any installment and they were sued for the amount of the PN. The Sheriff then levied ont eh machineries and farm implements and sold it at the public auction to SM. SM sought an alias writ of execution and the sheriff levied attachment on the rights and interests of the Tajanlangits in certain real properties w/ a view to another sale on execution. Tajanlangits instituted this action contending that they returned the implements to SM, and have thus, settled their accounts and SM REPOSSESSED THE MACHINEDS AND ARE COVERED BY THE PROHIBITION OF THE RECTO LAW.

w/n SM is covered by the Recto Law? No – SM elected to sue on the note EXCLUSIVELY, and thus chose to exact fulfillment of the oblig. to pay. In choosing to do so, it was not limited to the proceeds of the sale, on execution of the mortgaged good. – T contends that the machines were returned to and accepted by SM. Such was not proven bec. as the sheriff proves, the goods were deposited in the shop when the sheridd attached them in pursuance of the execution. Further, such an argument assumes that acceptance of the goods is w/ a view to cancellation of the sale. MACONDRAY VS. EUSTAQUIO

– –

Macondray sold to Eustaquio a De Soto car, sedan, payable in 12 monthly installments, w/ an acceleration clause. Eustaquio then executed a mortgage on the car in favor of Macondray. After paying the first installment, Eustaquio failed to pay and Macondray called upon the sheriff to take poss’n of the car. Eustaquio REFUSED to yield poss’n thereof, whereupon, Macondray brought the replevin sough and succeeded in getting poss’n of the car. It was then sold at public auction to Macondray.

Issue: w/n award of attorney’s fees and costs are in order. NO – interpretation of the phrase “any unpaid balance” - refers to the deficiency judgment to which the mortgagee may be entitled to when the proceeds from the auction sale are insufficient to cover the full amount of the secured obligation which include interest on the principal, attorney's fees, expenses of collection, and the costs. Were it the intention of the Leg. To limit the meaning tot eh unpaid balance of the principal, it would have so stated. – In all proceedings for the foreclosure of a chattel mortgage, executed on chattels which have been sold on the installment plan, the mortgagee is limited to the property mortgaged, and is not entitled to attorney’s fees and costs of suit.

Rescission in sale of immovables McLaughlin vs CA (1986) FACTS:  Mclaughlin and Flores entered into a contract of conditional sale of real property with the stipulated purchase price payable on installments.  Flores defaulted in the payment of the installments, so Mclaughlin filed a complaint for the rescission of the deed of conditional sale.  However, the suit was eventually compromised, with Flores agreeing on a scheduled payment of the balance of the purchase price.  Such compromise agreement also provided that in case of failure of the buyer to comply with the terms of payment, all payments previously made shall be forfeited in favor of the Mclaughlin as liquidated damages.  Flores still failed to pay on the dates provided in the compromise agreement, so Mclaughlin refused to accept further payment and eventually filed a motion for the writ of execution for the rescission of the contract.  Mclauglin also prayed for the forfeiture of all payments of the Flores previously made.  RTC/CA: ruled in favor of Flores. It held the right of Flores to prevent the rescission of the contract by his tender of the balance of the purchase price, based on the provisions of the Maceda Law. ISSUE: I. W/N Mclauglin can rescind the conditional contract of sale. HELD: I. CANNOT RESCIND. 1. Although there was no doubt that the buyer was no longer entitled to the benefits of the grace period under the Maceda Law, the court held that if the motion for the issuance of the writ of execution is considered as the notice of cancellation under the said law, Mclaughlin could cancel the contract only 30 days after the receipt of such notice. 2. The Court concluded that since the tender of payment of the balance of the purchase price was made within the 30 day period, this prevented the cancellation of the contract. 3. Although the Maceda law seem to require rescission and cancellation to be both notarial act, the case would hold notarial act as merely applicable to rescission, whereas “notice of cancellation” need not be by notarial act. 4. Even after the expiration of the grace period provided by the Law, the buyer still can prevent rescission or cancellation of the contract within the 30 day period when rescission or cancellation is to take effect.

5.

Buyers have a jurisprudential grace period which allows them to prevent the rescission or cancellation of their contracts even after they have received the demand for rescission or notice of cancellation, by paying-up the unpaid balance prior to the expiration of the 30-day period provided in the Maceda law.

Adelfa Properties vs CA (1995) FACTS:  Private respondents and their brothers, Jose and Dominador Jimenez, were the registered co-owners of a parcel of land.  Jose and Dominador Jimenez sold their share consisting of one-half of said parcel of land to Adelfa Properties.  Subsequently, a "Confirmatory Extrajudicial Partition Agreement" 4 was executed by the Jimenezes, wherein the eastern portion of the subject lot was adjudicated to Jose and Dominador Jimenez, while the western portion was allocated to herein private respondents.  An "Exclusive Option to Purchase" 5 was executed between Adelfa Properties and private respondents, under certain conditions.  Before petitioner could make payment it received summons for annulment of the deed of sale in favor of Household Corporation and recovery of ownership.  As a consequence, Adelfa informed PR that it would hold payment of the full purchase price and suggested that PR settle the case with their nephews and nieces.  Respondent Salud Jimenez refused to heed the suggestion of petitioner and attributed the suspension of payment of the purchase price to "lack of word of honor."  The RTC dismissed the civil case, and PR executed a Deed of Conditional Sale in favor of Emylene Chua over the same parcel of land.  Thus, Adelfa demanded refund of the 50% dp it has paid, but PR demanded the return of the certificate of title.  Adelfa failed to return the certificate, so PR filed for annulment of the contract.  RTC: agreement was an option contract, suspension of payment was a counter-offer, which was rejected. Exclusive Option to Purchase contract was cancelled (basically ruled in favor of PR)  CA: affirmed ISSUE: I. W/N the "Exclusive Option to Purchase" executed Adelfa Properties, Inc. and private respondents is an option contract. HELD: I. CONTRACT TO SELL, NOT OPTION CONTRACT NOR CONTRACT OF SALE 1. The distinction between the two is important for in contract of sale, the title passes to the vendee upon the delivery of the thing sold; whereas in a contract to sell, by agreement the ownership is reserved in the vendor and is not to pass until the full payment of the price. In a contract of sale, the vendor has lost and cannot recover ownership until and unless the contract is resolved or rescinded; whereas in a contract to sell, title is retained by the vendor until the full payment of the price, such payment being a positive suspensive condition and failure of which is not a breach but an event that prevents the obligation of the vendor to convey title from becoming effective. 2. There are two features which convince us that the parties never intended to transfer ownership to petitioner except upon the full payment of the purchase price. Firstly, the exclusive option to purchase, although it provided for automatic rescission of the contract and partial forfeiture of the amount already paid in case of default, does not mention that petitioner is obliged to return possession or ownership of the property as a consequence of non-payment. 3. Secondly, it has not been shown there was delivery of the property, actual or constructive, made to herein petitioner. The exclusive option to purchase is not contained in a public instrument the execution of which would have been considered equivalent to delivery.

Coronel vs CA –

– – – – – –

Romulo Coronel et al executed a document entitled "Receipt of Downpayment" in favor of Ramona Alcaraz. ○ It said, "Total amount P1.24M, P50k downpayment. Received from Ms. Ramona Alcaraz the sum of P50k. We bind ourselves to effect the transfer in our names from our deceased father the transfer certificate of title immediately upon receipt of the downpayment above-stated. On our presentation of the TCT already in our name, we will immediately execute the deed of absolute sale of said property and Miss Alcaraz shall immediately pay the balance of the P1.19M." On Jan 5, 1985, Ramona's mom paid already the P50k. On Feb 6, the property, originally registered in the name of Coronel's father, was transferred in their names. On Feb 18, the Coronels sold the property to Catalina Mabanag for P1.85M after Catalina had paid P300k. For this reason, the Coronels cancelled and rescinded the contract with Ramona by depositing the downpayment paid in the bank in trust for Ramona. Ramona and her mom filed a complaint for specific performance against the Coronels. But the Coronels still executed a Deed of Absolute Sale over the property in favor of Catalina. RTC: Coronel is ordered to execute in favor of Ramona a deed of absolute sale, and Ramona is to pay the balance. CA: Affirmed.

Issue: Contract to sell or contract of sale? Held: Conditional contract of sale. Consummation is subject only to the successful transfer of the certificate of title from the name of Coronel’s father to their names. DOCTRINE:



Under the definition of a contract of sale (consent + determinate subject matter + price), a contract to sell is NOT a contract of sale, because the first essential element is lacking. In a contract to sell, the prospective seller explicitly reserves the transfer of title to the prospective buyer, meaning, the happening of an event, which for present purposes we shall take as the full payment of the purchase price. What the seller agrees or obliges himself to do is to fulfill his promise to sell the subject property when the entire amount of the purchase price is delivered. In other words, the full payment of the purchase price partakes of a suspensive condition, the non-fullfillment of which prevents the obligation to sell from arising and thus, ownership is retained by the prospective seller without further remedies by the prospective buyer.



Stated positively, upon fulfillment of the suspensive condition (full payment of the purchase price), the prospective seller’s obligation to sell the subject property by entering into a contract of sale with the prospective buyer becomes demandable, as provided in Art 1479 (“A promise to buy and sell a determinate thing for a price certain is reciprocally demandable.”). A contract to sell may thus be defined as a bilateral contract (the prospective seller, while expressly reserving ownership of the property despite delivery thereof to the buyer, binds himself to sell the property exclusively to the prospective buyer upon fulfillment of the condition which is full payment of the purchase price. In a contract to sell, upon fulfillment of the suspensive condition (full payment), ownership will NOT automatically transfer to the buyer, although the property may have been previously delivered to him. The seller still has to convey title by entering into a contract of absolute sale.



In the present case: The agreement was NOT a contract to sell. – The seller made no express reservation of ownership or title to the subject parcel of land.





– – –

Plus, the circumstance which prevented the parties from entering into an absolute contract of sale pertained to the sellers themselves (remember the certificate of title was NOT in their names) and NOT the full payment of the purchase price. The seller had already agreed to sell the lot, but it just so happened that the certificate of title was still in the name of their father. As soon as the new certificate of title is issued in their names, they were committed to immediately execute the deed of absolute sale. Only then will the obligation of the buyer to pay the remainder of the purchase price will arise. Unlike in a contract to sell (where there is a condition of full payment by the buyer), the sellers here were the ones unable to enter into a contract of absolute sale by reason of the fact that the certificate was in the name of their father. It was the sellers who had the impediment. Besides, this condition was actually fulfilled when a new title was issued in their names. Art 1169 then applies: “In reciprocal obligations, neither party incurs in delay if the other does not comply or is not ready to comply in a proper manner with what is incumbent upon him.” There is thus no legal basis to rescind the contract of sale. And the sale to Catalina was a double sale. Catalina registered it, so the second paragraph of Art 1544 applies (“Should it be immovable property, the ownership shall belong to the person acquiring it who in good faith first recorded it in the Registry of Property”). Catalina could NOT have been in good faith because a notice of lis pendens had been annotated in February when Catalina registered the sale in April.

CA judgment affirmed. Coronel ordered to executed deed of sale in favor of Ramona.

PNB vs CA (1996) – Ong (buyer) and the spouses Robles (sellers) executed an Agreement of Purchase and Sale respecting 2 parcels of land in Quezon. ○ The conditions of the contract: Purchase price of P2M, to be paid: initial payment of P600k (P100k was already paid, and P500 shall be paid directly by the buyer to BPI to answer for the loan of the sellers from BPI of P500k and the interest that may have accrued). P1.4M shall be paid in four quarterly installments. The land shall be delivered upon execution of the document. – Ong took possession of the land together with the piggery, building, ricemill and house that was on it. He paid P100k, by depositing with UCPB, and deposited sums of money with BPI pursuant to the stipulations. – For the P1.4M balance, Ong issued 4 postdated Metrobank checks. However, they were dishonored due to insufficient funds. He promised to replace the checks but failed. What’s more, it turned out he did not pay to BPI the full amount. – The Robles demanded return of the properties, which went unheeded. They filed a case with the RTC for rescission and recovery of the properties with damages. While the case was pending, Ong constructed a fence of hollow blocks and expanded the piggery. ○ RTC: Ong is to return the land, and the Robles are to return the amount paid, but Ong is liable for damages. CA affirmed. Issue: Can the contract entered into by the parties be validly rescinded under Art 1191? Held: NO. Art 1191 does not apply. The contract is a contract to sell and not a contract of sale. Ong failed to pay the purchase price, so Robles had no obligation to deliver the land at all. – The contract was actually a contract to sell, not a contract of sale. In a contract of sale, the title to the property passes to the vendee upon the delivery of the thing sold, while in a contract to sell, ownership is, by agreement, reserved in the vendor and is not to pass to the vendee until full payment of the purchase price. In a contract to sell, the payment of the purchase price is a positive suspensive condition, the failure of which is not a breach, but a situation that prevents the obligation of the vendor to convey title from acquiring an obligatory force. – Here, the Robles’ bound themselves to deliver a deed of absolute sale and clean title covering the lands upon full payment of the price of P2M. This promise to sell was subject to the fulfillment of the suspensive condition of full payment of the price. Since Ong failed to pay fully, the non-fulfillment of the condition rendered the contract to sell ineffective and without force and effect. – The breach in Art 1991 is the obligor’s failure to comply with an obligation already existing. BUT in this case, failure to pay is not a breach but merely an event which prevents the vendor’s obligation to convey title from acquiring binding force. Hence, the agreement of the parties may be set aside, but not because of breach on the part of Ong. Rather, his failure to pay the price brought a situation which prevented the obligation of Robles from acquiring obligatory force. CA decision affirmed. BABASA V. CA Roque vs Lapuz

– – –

Roque and Lapuz entered into an agreement of sale covering lots in Quezon City. The price was to be payable in 120 monthly installments at the rate of P15-16 per square meter. Lapuez paid to Roque P150 as deposit and the further sum of P740 to complete payment of four monthly installments. A subdivision plan of the lots was approved. (It became Rockville Subdivision) Lapuz then requested Roque that he be allowed to substitute the lots with other lots, which are corner lots. He proposed this modification of their contract to sell because he found it difficult to

– – –

pay the monthly installments on the former lots. It was agreed that for the new lots, the uniform rate would be P17 per square meter payable in 120 monthly installments. But Lapuz failed to make any more payments. He kept asking for extensions instead. Roque made a demand for Lapuz to vacate the lots and to pay rentals. Both parties agreed that the period to pay for the lots is 10 years, but they disagree on the mode of payment (Lapuz claims that he could pay the purchase price at any time within 10 years with a gradual proportionate discount on the price, while Roque claims that there was an agreement for monthly installments. ○ CFI: Ruled in favor of Roque. CA: Affirmed, but modified by granting Lapuz 90 days to pay the balance of the price in the amount of P11M.

Issue: Is Lapuz entitled to the benefits of the third paragraph of Art 1191, for the fixing of a period within which he should comply with what is incumbent upon him? Held: NO. – Art 1191: “… The court shall decree the rescission claimed, unless there be just cause authorizing the fixing of a period.” – Art 1592: In the sale of immovable property, even though it may have been stipulated that upon failure to pay the price at the time agreed upon the rescission of the contract shall of right take place, the vendee may pay, even after the expiration of the period, as long as no demand for rescission of the contract has been made upon him either judicially or by a notarial act. After the demand, the court may not grant him a new term. – In the present case, there is no writing or document evidencing the agreement originally entered into, nor the modified agreement. The absence of a formal deed of conveyance is a strong indication that the parties did NOT intend immediate transfer of ownership and title, but only a transfer after full payment of the price. In this case, the lots were delivered to Lapuz who took possession. BUT the property is registered under the LRA, so it is the act of registration of the Deed of Sale which could legally effect the transfer of title of ownership to the transferee. Hence, the contract between the petitioner and the respondent was a contract to sell (where the ownership or title is retained by the seller and is not to pass until the full payment of the price, such payment being a positive suspensive condition and failure of which is not a breach, but simply an event that prevented the obligation of the vendor to convey title from acquiring binding force). – Jurisprudence (Luzon Brokerage) laid down the rule that Art 1592 does not apply to a contract to sell where title remains with the vendor until full payment of the price, as in the case at bar. Art 1191 is the applicable provision, where the obligee (Roque) elects to rescind or cancel his obligation to deliver ownership of the lots in question. – But Lapuz is not entitled to the benefits of paragraph 3. Having been in default, he is not entitled to the new period of 90 days. He refused to pay further installments, he also refused to ever sign a contract of sale, and he failed to pay anything after 26 YEARS! This is evident bad faith and malice, so he does not deserve the benefits of being granted an additional period. In favor of Roque. Lapuz is not given any additional period to pay.

ANGELES V. CALASANZ Parties may stipulate for cancellation of the contract by a party w/o judicial intervention / Breach must be substantial



– –





Ursula & Tomas Calasanz and Angeles & Juani entered into a contract to sell a piece of land in Cainta, rizal for 3,920. Angeles and Juani made a downpayment of 392 and were to pay in monthly installments. They paid such installments until July 1966, when their aggregate payment already amounted to 4,500+. On numerous occasions, Calasanz accepted delayed installment payments. Calasanz demanded for past due accounts then cancelled the contract for failure to meet the subsequent payments. Angeles filed a case to compel Calasanz to execute the final deed of sale since they have already paid 4,500+. Calasanz: they had a right to cancel the contract as Angeles violated par. 6 of the contract to sell when they failed to pay the installment for August for more than 5 months. And even without this, they had a right to rescind under Art. 1191. ○ Par. 6: that if Angeles failed to pay an installment, he would be given a grace period. And upon lapse of the 90 days from the expiration of the grace period, Calasanz may declare the contract cancelled… Angeles: such provision is contrary to law.

Was the contract to sell automatically and validly cancelled? NO – Par. 6 of the contract to sell is valid. There is nothing in the law that prohibits the parties from entering into an agreement that violation of the terms of the contract would cause its cancellation even without court intervention. However, the party who deems the contract violated may consider it rescinded, but proceeds at his own risk, as such is always subject to scrutiny and review by the proper court when the other party denies that the rescission is justified. – HOWEVER, the general rule is that rescission will not be permitted for a slight or casual breach, but only for such substantial or fundamental breach as would defeat the very object of the parties in making the agreement. – The breach was so slight and casual since Angeles already paid the monthly installments for almost 9 yrs. In only a short time, the entire obligation would have been paid. The principal obligation was 3,920 and they have paid an aggregate amount of 4,500+. – Plus, when Calasanz, instead of availing their right to rescind, have accepted and received delayed payments of installments, though Angeles have been in arrears beyond the grace period mentioned in pat. 6, they have waived and are now estopped from exercising their alleged right of rescission. – Lastly, the contract to sell was a contract of adhesion, which must be construed against Calasanz. DIGNOS V. CA No valid rescission for lack of notarial act ; slight breach

– – – –

1. –



1. – –



Dignos owned a parcel of land in Lapu-Lapu City and sold said lot to Jabil for 28K, payable in 2 installments w/ assumption of indebtedness w/ First Insular Bank of Cebu. Dignos then sold the same land to Cabigas for 35K. A Deed of Absolute Sale was executed in favor of Cabigas. Dignos then refused to accept Jabil’s payment for the balance of the purchase price. Jabil then filed the present suit. Whether the contract was a deed of absolute sale or a contract to sell? Absolute sale Although denominated as “Deed of Conditional Sale”, it is an absolute sale as there is no proviso to the effect that title is reserved in the vendor. Nor is there a stipulation giving the vendor the right to unilaterally rescind the contract the moment the vendee fails to pay w/in a fixed period. All elements of a valid contract of sale are present. Plus, there was already delivery of the land in question to Jabil. Whether the sale was rescinded when they sold the land to the Cabigas?  NO Dignos never notified Jabil by notarial act that they were rescinding the contract and neither did they file a suit in court to rescind the sale. The supposed letter by Amistad, claiming to be an emissary of Jabil, telling Dignos not to go to Jabil as he had no money is not sufficient. There is no showing that Amistad was properly authorized to make such extra-judicial rescission for the latter, who denied having sent Amistad. Plus, where time is not of the essence, a slight delay is not sufficient ground for the rescission of the agreement. Considering that the balance was only 4K, and was delayed for only one month, equity and justice mandate that Jabil be given addtl period w/in which to complete payment of the purchase price.

Topacio vs Court of Appeals





– –

– –



The spouses De Villa (parents-in-law of Topacio) were the former owners of a lot in QC. It was previously mortgaged to Ayala Investment and Development Corp to secure an obligation of P500k. For failure to pay, the mortgage was foreclosed and consequently, BPI acquired the property as highest bidder. Topacio wanted to buy the property. He made an offer for P900k, but was asked to improve it. Together, they arrived at P1.25M as the purchase price, with 30% downpayment and the balance payable in cash upon execution of the Deed of Sale. Topacio paid the initial payment of P375k. BPI wrote to Topacio and informed him that he had until January 4, 1986 to pay the balance of P875k. P. asked for extensions. BPI agreed to extend up to June 30. Topacio was unable to meet the deadline, so BPI wrote a letter to Topacio, where BPI declared himself free to sell the property to other buyers and that Topacio could claim his initial payment of P375k. Topacio merely asked for more extensions. While BPI kept telling Topacio that he could claim the P375k back (in the form of a cashier’s check), Topacio declined. But BPI mailed the check to him. The check remained with Topacio, uncashed. BPI then told Topacio that the property would be sold for P1.6M instead, so Topacio reminded him of the original agreement (P1.25M), but BPI refused.

– –

RTC: In favor of Topacio, finding that there is a perfected contract of sale which is still enforceable because BPI did not rescind either by judicial or notarial rescission. CA: Reversed. The contract is a contract to sell, not a contract of sale.

Issue: Contract to sell or contract of sale? Held: Contract of sale. – The payment by Topacio of P375k was the operative act that gave rise to a perfect contract of sale. It is considered earnest money (something of value to show that the buyer was really in earnest, and given to the seller to bind the bargain). It is considered part of the purchase price and proof of the perfection of the contract. – The parties agreed on the object (house and lot in White Plains), and the price and the manner of payment. – Nowhere in the transaction indicates that BPI reserved its title on the property, nor did it provide for any automatic rescission in case of default. So when Topacio failed to pay the balance of P875k despite several extensions, BPI could not validly rescind the contract w/o complying with the provision of Art 1592 or Art 1191 on notarial or judicial rescission respectively.

LUZON BROKERAGE CO V MARITIME BUILDING CO UP V. DE LOS ANGELES

PALAY, INC. vs JACOBO CLAVE & NAZARIO DUMPIT Palay, Inc., through its President, Albert Onstott executed in favor of Nazario Dumpit, a Contract to Sell a parcel of Land of the Crestview Heights Subdivision in Antipolo, Rizal. The sale price was P23,300.00 with 9% interest per annum, payable with a downpayment of P4,660.00 and monthly installments of P246.42 until fully paid. Paragraph 6 of the contract provided for automatic extrajudicial rescission upon default in payment of any monthly installment after the lapse of 90 days from the expiration of the grace period of one month, without need of notice and with forfeiture of all installments paid. Dumpit paid the downpayment and several installments. Six years later Dumpit wrote Palay offering to update all his overdue accounts with interest, and seeking its written consent to the assignment of his rights to a certain Lourdes Dizon. Palay informed him that his Contract to Sell had long been rescinded pursuant to paragraph 6 of the contract, and that the lot had already been resold. Dumpit filed a complaint to the National Housing Authority (NHA) who found the rescission void and ruled that Dumpit be reimbursed for the installments paid. Upon appeal, Presidential Executive Assistant Clave affirmed the decision of the NHA. W/N notice or demand is not mandatory under the circumstances and, therefore, may be dispensed with by stipulation in a contract to sell. Palay, Inc. contends that it was justified in canceling the contract to sell w/o prior notice or demand upon respondent in view of paragraph 6. SC: Judicial action for the rescission of a contract is not necessary where the contract provides that it may be revoked and cancelled for violation of any of its terms and conditions. There has to be at least a written notice sent to the defaulter informing him of the rescission. The act of a party in treating a contract as cancelled should be made known to the other. Extrajudicial rescission has legal effect where the other party does not oppose it. Where it is objected to, a judicial determination of the issue it still necessary. If the debtor impugns the obligation, it shall be subject to judicial determination. The resolution of Palay of the contract was ineffective and inoperative against private respondent for lack of notice of resolution. The indispensability of notice of cancellation to

the buyer was later underscored in RA 6551 “An Act to Provide Protection to Buyers of Real Estate on Installment Payments."

CHENG vs. GENATO (1998) (copied from Mon) 1. 2.

3. 4.

5. 6. 7.

8.

9.

Ramon Genato is the owner of 2 lots in Bulacan covered by a TCT. He entered into a contract with Da Jose spouses over the 2 lots. The contract was in a public instrument and was duly annotated. There was a condition stated in the annotation that full payment will be paid in within 30 days after payment of downpayment worth 50k. Da Jose spouses asked for an extension when stipulated time of payment came. Pending the effectivity of the extension period, without notice to the spouses, Genato executed an Affidavit to Annul the Contract to Sell (affidavit). But such was not annotated at the back of the titles right away. Subsequently, Cheng expressed interest in the lots. Genato showed the TCT’s and the annotations at the back and the Affidavit. Cheng issued a check for 50k upon the assurance that the contract with the Da Jose spouses will be annulled for which Genato issued a handwritten receipt. Genato deposited the check and Cheng kept on reminding him to register the affidavit. Genato then caused such registration. Genato met by coincidence in the registry the Da Jose spouses. The spouses were shocked to know about the affidavit because the extension they asked was still in effect. With that, Genato decided to continue the Contract he had with them. Genato advised Cheng of his decision of not pursuing the sale with him. Cheng sent a letter demanding of compliance as for him there was already a perfected contract of sale. Genato returned the check but was refused by Cheng. Nevertheless, the contract with the spouses still pushed through Cheng instituted a complaint for specific performance to compel Genato. LC held that the receipt issued by Genato to Cheng meant a sale and not just a priority or an option to buy. Also, there was a valid rescission by virtue of the affidavit and concluded that Cheng should be preferred over the spouses. However on appeal, the CA reversed the decision and that the prior contract to sell in favor of the spouses was not validly rescinded and that Cheng should pay damages for being in bad faith.

Main Issue: w/n the contract to sell with regard to the spouses was validly rescinded. NO! – Execution by Genato of the affidavit to annul the contract is not called for. Even with or without the affidavit, their non-payment to complete the full downpayment ipso facto avoids their contract to sell, it being subject to a suspensive condition – There can be no rescission of an obligation that is still non-existent, the suspensive condition not having occurred yet – When a contract is subject to a suspensive condition, its birth or effectivity can take place only if & when the event which constitutes the condition happens or is fulfilled. If the suspensive condition does not take place, the parties would stand as if the conditional obligation had never existed.





Nevertheless, Genato is not relieved from the giving of a notice (verbal or written) to the Da Jose spouses for his decision to rescind their contract. The act of a party in treating a contract as cancelled should be made known to the other. Such act is always provisional & is always subject to scrutiny and review by the courts in case the alleged defaulter brings the matter to the proper courts. Thus, Cheng’s contention that the contract to sell between Genato and spouses was rescinded due to Genato’s unilateral rescission finds no support in this case

Issue: w/n Cheng’s contract with Genato was not just a contract to sell but a conditional contract of dale which gave him better rights, thus precluding the application of 1544 – Cheng was inconsistent in characterizing the contract he allegedly entered into – In fact, both courts below correctly held that the receipt which was the result of their agreement is a contract to sell – Even if it be treated as conditional contract of sale, it did not acquire obligatory force since it was subject to suspensive condition that the earlier contract of the spouses be cancelled first – a condition never met, as Genato redeemed himself by maintaining the contract with the spouses – A reading of 1544 is not apropos to the case. The following circumstance must occur: ○ That 2 or more sales transactions in issue must pertain to exactly the same subject matter, and must be valid sales transactions ○ The 2 or more buyers at odds over the rightful ownership of the subject matter must each represent conflicting interests ○ The 2 or more buyers at odds over the rightful ownership of the subject matter must each have bought from the very same seller – These situations are obviously lacking. But even though, 1544 should still apply to the situation. FIRST IN TIME, FIRST IN RIGHT. Not only was the contract between the spouses first in time, it was also registered before the intrusion of Cheng – The rule of 1544 for the 2nd buyer to be able to displace the 1st buyer are: ○ That the 2nd buyer must show that he acted in good faith from the time of acquisition until title is transferred ○ That 2nd buyer must show continuing good faith until contract ripens into full ownership – The knowledge gained by Cheng of the 1st transaction defeats his rights even if he is the first to register the transaction since he can be considered in bad faith – Good faith must concur with registration for such prior right to be enforceable. The annotation made by the spouses on the titles more than satisy the requirement. Issue: w/n it was error to hold him liable for damages – No, Genato in bad faith when he filed suite for specific performance knowing fully well that his agreement with Genato did not push through Petition denied. TORALBA V DE LOS ANGELES

Conditions / warranties Romero vs CA (1995)

FACTS:  Romero was engaged in the business of production, manufacture and exportation of perlite filter aids, permalite insulation and processed perlite ore.  Petitioner and his foreign partners decided to put up a central warehouse in Metro Manila on a land.  Alfonso Flores and his wife offered a parcel of land to petitioner.  Flores spouses called on petitioner with a proposal that should he advance the amount of P50k which could be used in taking up an ejectment case against the squatters, pr would agree to sell the property for only P800/sqm. Petitioner expressed his concurrence.  A Deed of Conditional Sale was executed between petitioner and pr.  Pr then filed a complaint for ejectment against the squatters, which the court rendered in favor of the former.  However, the decision was handed down beyond the 60-day period (w/in to evict the squatters), as stipulated in the contract.  So Pr sought to return the P50k given by petitioner since “she could not get rid of the squatters”.  Then, Presidential Commission for the Urban Poor asked for a grace period of 45 days w/in which to evict the squatters.  Pr advised petitioner that the Deed of Conditional Sale had been rendered null and void by virtue of his failure to evict the squatters from the premises within the agreed 60-day period. She added she has "decided to retain the property."  Pr, prompted by petitioner's continued refusal to accept the return of the P50k, filed for rescission of the deed of "conditional" sale.  RTC: pr had no right to rescind the contract since it was she who "violated her obligation to eject the squatters from the subject property" and that petitioner, being the injured party, was the party who could, under Article 1191 of the Civil Code, rescind the agreement.  CA: reversed; It opined that the contract entered into by the parties was subject to a resolutory condition, i.e., the ejectment of the squatters from the land, the non-occurrence of which resulted in the failure of the object of the contract; that pr substantially complied with her obligation to evict the squatters; that it was petitioner who was not ready to pay the purchase price and fulfill his part of the contract. ISSUE: I. W/N the vendor may demand the rescission of a contract for the sale of a parcel of land for a cause traceable to his own failure to have the squatters on the subject property evicted within the contractually-stipulated period? HELD: I. VENDOR CANNOT DEMAND RESCISSION 1. It would be futile to challenge the agreement here in question as not being a duly perfected contract. A sale is at once perfected when a person (the seller) obligates himself, for a price certain, to deliver and to transfer ownership of a specified thing or right to another (the buyer) over which the latter agrees. 2. Under the agreement, pr is obligated to evict the squatters on the property. The ejectment of the squatters is a condition, the operative act of which sets into motion the period of compliance by petitioner of his own obligation, i.e., to pay the balance of the purchase price. Pr's failure "to remove the squatters from the property" within the stipulated period gives petitioner the right to either refuse to proceed with the agreement or waive that condition in consonance with Article 1545 of the Civil Code. 3. This option clearly belongs to petitioner and not to pr. Her action for rescission is not warranted. She is not the injured party. 4. The right of resolution of a party to an obligation under Article 1191 of the Civil Code is predicated on a breach of faith by the other party that violates the reciprocity between them. It is pr who has failed in her obligation under the contract. Petitioner did not breach the agreement.

Heirs of Pedro Escanlar vs CA (1997) FACTS:

   

    

 

Spouses Nombre and Cari-an died w/o a child. Nombre’s heirs include his nephews and grandnephews. Two parcels of land formed part of the estate of Nombre and Cari-an. The heirs of Cari-an (PR) executed a Deed of Sale in favor of Escanlar and Holgado (petitioners). Petitioners paid P50k as a form of downpayment, but was unable to pay the remaining balance (paid only 12 installments). Being former lessees, petitioners continued in possession of the said lots, and continued to pay rent. PR later sold the said lots to the spouses Chuas. PR then filed an action for cancellation of sale against petitioners, for failure to pay the balance. Petitioners however, sold their rights and interests over the said lots to Jayme, and turned over possession. RTC: 1. Sale to petitioners was nullified since it was not approved by the probate court. 2. Sale to the spouses Chuas, which was approved by the probate court, was upheld. CA: affirmed, questioned deed of sale (one with petitioners) is a contract to sell because it shall become effective only upon approval by the probate court and upon full payment of the purchase price.

ISSUE: I. W/N Sale was a contract of sale or contract to sell. HELD: I. CONTRACT OF SALE 1. PR as sellers did not reserve unto themselves the ownership of the property until full payment of the unpaid balance. 2. There is no stipulation giving the sellers the right to unilaterally rescind the contract the moment the buyer fails to pay w/in the fixed period. 3. Prior to the sale, petitioners were in possession of the subject property as lessees. Upon sale to them of the rights and interests, they remained in possession, not in the concept of lessees anymore but as owners now through symbolic delivery known as traditio brevi manu. 4. In a contract of sale, the non-payment of the price is a resolutory condition which extinguishes the transaction that, for a time, existed and discharges the obligations created thereunder. 5. The Deed of Sale, complying as it does with the essential requisites, is a valid one. However, it did not bear the stamp of approval of the court. 6. This, notwithstanding the contract’s validity was not affected. In other words, only the effectivity and not the validity of the contract is affected. 7. Petitioners are correct in saying that the need for approval by the probate court exists only where specific properties of the estate are sold and not when only ideal and invisible shares of an heir are disposed of. Power Commercial and Industrial vs CA (1997)



– –





Power Commercial and Industrial Development Corporation, an industrial asbestos manufacturer, entered into a contract of sale with the spouses Quiambao for a parcel of land in Makati. The contract stated that Power Commercial would pay P108k as downpayment and the balance of P295k upon the execution of the deed of transfer of the title over the property. Plus Power Commercial assumed the existing mortgage on the land. In full satisfaction, Power Commercial paid P79k to PNB. The Quiambaos mortgaged the land again to PNB to guarantee a loan of P145k. Power Commercial agreed to assume payment of the loan. The parties executed a Deed of Absolute Sale With Assumption of Mortgage, where Power Commercial agreed to pay in full the entire amount of the mortgage to the mortgagee bank PNB. It also stated, “We warrant that we are the lawful and absolute owners, free from any lien and/or encumbrance, and we hereby agree and warrant to defend its title and peaceful possession thereof in favor of Power commercial against any claims whatsoever of any and all third persons.” But PNB informed the Quiambaos that, because of Power Commercial failing to submit papers, the application for assumption of mortgage was considered withdrawn, and the outstanding balance of P145k was deemed fully due and demandable. PNB demanded payment within 15 days.





Power Commercial made some payments, and told PNB in a letter, “It was our understanding that this lot was free and clear of problems.. inasmuch as the previous owner was not able to keep his commitment, it will be necessary for us to take legal possession of this lot.” PNB replied, “This refers to the loan granted to Quiambao which was assumed by you. We request you to remit payments.” Power Commercial filed a case against the Quiambaos for rescission and damages. And in reply to PNB’s letter, Power Commercial demanded the return of the payments it made on the ground that the assumption of mortgage was never approved. Also, they claimed that there were squatters on the lot that they could not eject, and the Quiambaos were at fault for this. ○ While the case was pending, the mortgage was foreclosed, and bought by PNB during the public auction. ○ RTC: The failure of the Quiambaos to deliver actual possession to Power Commercial entitled Power Commercial to rescind the sale, and in view of such failure and of the denial of Power Commercial’s assumption of mortgage, PNB was obliged to return the payments made by Power Commercial. ○ CA: Reversed. It ruled that the deed of sale between them did not obligate Power Commercial to eject the lessees from the land in question as a condition of the sale, nor was the occupation thereof by said lessees a violation of the warranty against eviction. Hence, there was no substantial breach to justify rescission nor return of the payments. ○ Power Commercial contends that there was a substantial breach of the contract between the parties warranting rescission, and there was a mistake in payment made by Power Commercial, obligating PNB to return such payments.

ISSUE: Is the seller’s failure to eject the lessees from a lot that is the subject of a contract of sale with assumption of mortgage a ground for (1) rescission of such contract and (2) for a return by the mortgagee of the payments made by the buyer who assumed such mortgage? HELD: NO. – The alleged “failure” of the Quiambaos to eject lessees from the lot and to deliver actual and physical possession cannot be considered a substantial breach for two reasons: (1) Such failure was not stipulated as a condition (whether resolutory or suspensive) in the contract. (2) Its effects and consequences were not specified either. – The provision cited by Power Commercial (“We hereby warrant that we are the lawful and absolute owners..”) does not impose a condition or an obligation to eject the lessees from the lot. If the parties inteded to impose on the Quiambaos the obligation to eject tenants from the lot, it should have included a stipulation to that effect. Absent a stipulation therefore, the court cannot say that the parties intended to make its nonfulfillment a ground for rescission. – Power Commercial was well aware of the presence of the tenants, and even undertook the job of ejecting them. They actually filed suit to eject them. – There was no fulfillment of the reqs for breach of warranty against eviction: 1) Purchaser has been deprived of the whole or part of the thing sold; 2) this eviction is by final judgment; 3) the basis thereof is by virtue of a right prior to the sale made by the vendor; 4) the vendor has been summoned and made co-defendant in the suit for eviction at the instance of the vendee. – Power Commercial WAS deprived of ownership, BUT that was because it failed to pay the amortizations on the mortgage, causing the lot to be foreclosed. That’s entirely Power Commercial’s fault. It was under obligation to pay the amortizations on the mortgage because of the contract. Petition denied. CA decision affirmed. Nutrimix Feeds Corporation vs CA (2004) – –



The spouses Evangelista started to directly procure various kinds of animal feeds from Nutrimix. They were given a credit period of 30-45 days to postdate checks (this accommodation was made because the company president was close to the family). Initially, they were good paying customers, but eventually they failed to issue checks despite delivery. Their outstanding obligation became P766k. Their postdated checks were dishonored because the account was closed. Nutrimix filed a complaint for sum of money and damages. In their answer, the Evangelistas admitted their unpaid obligation but claimed that their animals had died

because of the feeds. The Evangelistas also filed a complaint for damages, for the untimely and unforeseen death of their animals supposedly effected by the feeds. ○ RTC: In favor of Nutrimix. The Evangelistas are to pay the amount of P766k to Nutrimix. ○ CA: Reversed. The Evangelistas were not obligated to pay Nutrimix, in view of its breach of warranty against hidden defects. Issue: Is Nutrimix liable for hidden defects? Held: NO. – The provisions on warrant against hidden defects are in Articles 1561 and 1566 in the Civil Code. – A hidden defect is one which is unknown or could not have been known to the vendee. Under the law, the requisites to recover on account of hidden defects are: 1) defect is hidden 2) defect must exist at time the sale was made 3) the defect must ordinarily have been excluded from the contract 4) the defect must be important (renders the thing unfit or considerably decreases fitness) 5) the action must be instituted within the statute of limitations – To be able to prove liability on the basis of breach of implied warranty, three things must be established: 1) that they sustained injury because of the product; 2) the injury occurred because the product was defective or unreasonably unsafe; 3) the defect existed when the product left the hands of the petitioner. ○ The defect must be present upon delivery or manufacture, or when the product was sold to the purchaser. – Nutrimix delivered the animal feeds in July, but the Evangelistas had them examined three months after their broilers and hogs had died. A difference of 3 months means the feeds could have already been contaminated by outside factors. – One of the witnesses, Dr. Garcia, testified that the feeds submitted to her contained very high level of aflatoxin, possibly caused by mold. – Several investigations (people from the Bureau of Plants Industry and the NBI) showed that the sample feeds yielded positive results for components in rat poison. – The Evangelistas brought sample feeds in sealed bags and chickens to a specialist of the Phil Nuclear Research Institute. Chickens that were fed with the feeds died. But these bags were in sealed bags; there is no proof they were from Nutrimix. – There is evidence that the Evangelistas combined different kinds of animal feeds and fed them to their animals. – The Evangelistas failed to prove that Nutrimix is guilty of breach. The buyer of the product has the burden of proving that the seller of the product breached its warranty. – It must be stressed that the remedy against violations of warrant against hidden defects is either to withdraw from the contract (accion redhibitoria) or to demand a proportionate reduction of the price (accion quanti minoris) with damages in either case. In any case, the Evangelistas had already admitted, both in their testimonies and pleadings, that they are indeed indebted to Nutrimix for the unpaid animal feeds delivered to them. For this reason alone, they should be held liable for their unsettled obligations to Nutrimix.

Extinguishment of Sale VILLARICA VS CA

– – – –

– –

the spouses Villarica sold to spouses Consunji a lot in Davao for 35K. The instrument of absolute sale was in the form of a deed poll, acknowledged by a notary public. The public instrument of absolute sale and TCT were delivered to Consunji. On the same day, Consunji executed another public instrument, whereby they granted the spouses Villarica an option to buy the same property within the period of one year for the price of P37,750. The cert of Villarica was cancelled and a new TCT was issued to Consunji. Consunji sold the land to Francisco, for whom a new TCT was issued. Villarica brought an action against the spouses Consunji and Jovito S. Francisco for the reformation of the instrument of absolute sale into an equitable mortgage as a security for a usurious loan of P28,000 alleging that such was the real intention of the parties. CFI: equitable mortgage; CA: reversed



Villarica: under Article 1604 in relation to Articles 1602 and 1603 of the Civil Code, the instrument of absolute sale, should be presumed as an equitable mortgage on the grounds that (1) the price of P35,000 was unusually inadequate; (2) the vendors remained in possession of the property sold; (3) the period of one year for repurchase granted in the instrument was extended for one month; (4) the vendors pay the taxes on the land sold.

Was the agreement really an equitable mortgage? NO 1.

2.

3. –



1.

The price was not inadequate. TC erred in saying that the market price at the time of the sale to COnsunji was 70K. It was, in fact, 35K. The vendors did not remain in possession of the land sold as lessees or otherwise. On their request in order to help them in the expenses of their children in Manila, the vendors were merely allowed by the vendees to collect the monthly rents of P300 for a certain period on the understanding that the amounts so collected would be charged against them. What was given was an option to buy, which is different and distinct from the right of repurchase which must be reserved by the vendor, by stipulation to that effect, in the contract of sale. The right of repurchase is not a right granted the vendor by the vendee in a subsequent instrument, but is a right reserved by the vendor in the SAME instrument of sale as one of the stipulations of the contract. Once the instrument of absolute sale is executed, the vendor can no longer reserve the right to repurchase, and any right thereafter granted the vendor by the vendee in a separate instrument cannot be a right of repurchase but some other right like the option to buy in the instant case. Hence, Exhibits "B" and "D" cannot be considered as evidencing a contract of sale with pacto de retro. Since Exh. "D" did not evidence a right to repurchase but an option to buy, the extension of the period of one year for the exercise of the option by one month does not fall under No. 3, of Article 1602 of the Civil CodE (When upon or after the expiration of the right to repurchase another instrument extending the period of redemption or granting a new period is executed.) the taxes paid were back taxes, which are supposed to be paid by the vendors.

Misterio vs Cebu State College of Science and Technology

– –



– – – –

1. 2.

the Provincial Board of Cebu granted the usufruct of 41 parcels of land, covering 104.5441 hectares of the Banilad Friar Lands Estate to the Sudlon Agricultural High School (SAHS). Asuncion Sadaya-Misterio executed a Deed of Sale of a parcel of land denominated as Lot No. 1064 of the Banilad Friar Lands Estate, in favor of the SAHS. The sale was subject to the right of the vendor to repurchase the property after the high school shall have (1) ceased to exist, or (2) shall have transferred its site elsewhere the Provincial Board of Cebu, through Resolution No. 491, donated the aforementioned 41 lots to SAHS, subject to two (2) conditions: (1) that if the SAHS ceases to operate, the ownership of the lots would automatically revert to the province, and (2) that the SAHS could not alienate, lease or encumber the properties Batas Pambansa (B.P.) Blg. 412 was enacted, which incorporated and consolidated as one school system certain vocational schools in the province of Cebu, including the SAHS, and which became an extension of the Cebu State College of Science and Technology (CSCST). The province of Cebu decided to recover what it donated earlier. Misterio’s heirs informed SAHS of their intention to exercise their right to repurchase the property as stipulated in the aforecited deed of sale Bonilla, Superintendent: that SAHS still existed and that it was just a change in name w/n the sale was void for SAHS had no juridical personality of its own, and thus, cannot acquire and possess any prop  NO, sale was valid w/n Misterio had the right to repurchase the property NO, prescribed

– –

that the SAHS had ceased to exist when B.P. Blg. 412 took effect, the appellate court ruled that the period for the petitioners to repurchase the property expired on June 1987, four years after the enactment of B.P. Blg. 412. The petitioners argue that the issue of whether the SAHS had ceased to exist had still yet to be resolved. The petitioners posit that unless and until judgment would be rendered stating that the SAHS has ceased to exist, the period to repurchase the property would not start to run. It is only from the finality of the said judgment that the right to repurchase the property may be exercised; hence, they still had thirty (30) days from the date of the promulgation of the CA decision within which to repurchase the property. NO ○ The four-year period for the petitioners to repurchase the property was not suspended merely and solely because there was a divergence of opinion between the petitioners, on the one hand, and the respondent, on the other, as to the precise meaning of the phrase “after the SAHS shall cease to exist” in the deed of sale. The existence of the petitioners’ right to repurchase the property was not dependent upon the prior final interpretation by the court of the said phrase.

Vda. De Zulueta REYES vs. HAMADA As a consequence of the extrajudicial foreclosure proceeding instituted by the PNB, certain real properties of the spouses Hamada, located in Baguio and mortgaged to the said institution, were sold at public auction and awarded to Arsenio Reyes being the highest bidder. At 10 in the evening of the last day of redemption, the mortgagors delivered to the city sheriff the checks covering the redemption amount, which was accepted by the said official. Purchaser Reyes refused to recognize the validity of such redemption and demanded the delivery of possession of the properties which remained with defendant mortgagors. Because of such refusal, Reyes filed a civil case. During the pendency of the case, Reyes filed recovery of rentals paid by tenants in the said lot. Issue: who is entitled to the rents? The rule is clear that if during the period of redemption the judgment-debtor is in possession of the property sold, he is entitled to retain it and to receive its fruits, the purchaser not being entitled to its possession; but if the property is in the possession of a tenant, it is only then that the purchaser is entitled to receive its rents or the reasonable value of its use and occupation. In such a case, the purchaser is accountable for the amount thus received to the judgment-debtor when he effects the redemption. Almeda v. Daluro Francisco v. Bolser Soriano vs Bautista (1962)



– –

– –

Spouses Bautista own a parcel of land in Rizal. In consideration of P1,800, they signed a mortgage document in favor of Soriano and De Jesus. This document stated that the land cannot be re-mortgaged again to other. Failure to pay would also allow for foreclosure. It also stated, “If the financing condition of the mortgagees will permit, (Soriano) may purchase said land absolutely on any date within the 2 year period of this mortgage at the agreed price of P3,900.” Possession was transferred to Soriano, who possessed it up until the date of this case. Soriano payed the sum of P450 to Bautista, who refused it and returned the payment. Soriano’s attorney informed Bautista that Soriano wanted to buy the property, but Bautista ignored it. Soriano filed a complaint, praying that they be allowed to consign or deposit with the clerk of court the sum of P1,650 as the balance of the purchase price. Bautista filed a complaint against Soriano, asking for the release of the mortgage.

Issue: Whether, having seasonably advised Bautista that they had decided to buy the land in question, Soriano is entitled to specific performance consisting of the execution by Bautista of the deed of sale. Held: YES.

– –



Bautista contends that, being mortgagors, they can not be deprived of the right to redeem the property, because such right is inherent in and inseparable from this kind of contract. But this premise is not correct. While the transaction is undoubtedly a mortgage, it carries the added special provision quoted. It renders the mortgagor’s right to redeem defeasible at the election of the mortgagees. There is nothing illegal or immoral in this. It is simply an option to buy. Bautista claims that they had already tendered the sum of P1,800 to redeem the mortgage. But that tender was ineffective. In the first place, it must have been made AFTER the option to purchase had been exercised by Soriano, and secondly, Bautstia’s offer to redeem could be defeated by Soriano’s preemptive right to purchase within the period of two years. Such right was availed of and Bautista was accordingly notified by letter. Offer and acceptance converged and gave rise to a perfected and binding contract of sale.

Bautista to execute a deed of sale covering the property in favor of Soriano.

View more...

Comments

Copyright ©2017 KUPDF Inc.
SUPPORT KUPDF